BRS – Pediatrics: Dermatology

BRS – Pediatrics: Dermatology

Source: BRS Pediatrics, 2019

I. General Concepts

A. Skin examination should be conducted in good light and should be complete, including evaluation of the scalp, hair, nails, eyes, mouth, palms, and soles. Examination should be both visual and tactile.
B. Description of skin disease requires identification of primary lesions and any secondary characteristics. Configuration and distribution of lesions should also be noted.

1. Primary lesions

a. Macules are <10 mm in size, flat, and nonpalpable, and represent a cutaneous color change. A patch is a large macule (≥10 mm).
b. Papules are epidermal or superficial dermal lesions <10 mm in size that are elevated above the skin surface. A plaque describes large or coalesced papules (≥10 mm).
c. Nodules are dermal lesions that are <10 mm in size and generally below the skin surface, although they may have an epidermal component that rises above the skin surface. A tumor is a large nodule (≥10 mm).
d. Vesicles are fluid-filled papules. A bulla is a large vesicle.
e. Pustules are purulent-filled papules.
f. Cysts are nodules filled with expressible material.
g. Wheals are cutaneous elevations caused by dermal edema.

2. Secondary characteristics include the following:

a. Scale: desquamation of the stratum corneum
b. Crust: dried exudate and debris
c. Pigmentary changes
d. Excoriations: linear erosions into the epidermis caused by fingernail scratches
e. Scars: thickened fibrotic dermis
f. Ulcers: absence of epidermis and some of the dermis
g. Atrophy: thinning of the epidermis or dermis
h. Fissures: linear cracks into the dermis

3. Configuration and distribution

a. Configuration of lesions may be described as linear, annular (i.e., ring shaped), arcuate (i.e., in half-circles), serpiginous (i.e., with a wavy or serpentine border), reticulated (i.e., netlike), grouped, or discrete (i.e., distinct and separate).
b. Distributions include generalized, flexural, acral (hands, feet, buttocks), dermatomal (confined to a dermatome), or other specific locations. Some skin lesions are blaschkoid, meaning they follow the lines of Blaschko, which are the lines of cutaneous development in utero.

C. Diagnostic procedures
1. Woods light can highlight pigmentary changes and some dermatophytes.
2. Scrapings

a. Fungus. Ten percent potassium hydroxide (KOH) can be added to a scraping of a scale or exudate to identify fungal hyphae.
b. Scabies. Examination under a microscope of a scraping of an unscratched lesion or burrow for mites, eggs, or feces may be diagnostic of scabies.
c. Herpes simplex virus (HSV). The base of a vesicle can be scraped for laboratory identification of the herpes virus.

3. Cultures may be obtained for bacteria, virus, fungus, and yeast.
4. Invasive techniques

a. Incision and drainage may be performed for diagnosis, to obtain cultures, or for therapy.
b. Biopsy

1. Shave or tangential biopsy for epidermal and superficial dermal lesions
2. Punch biopsy for epidermal, dermal, and superficial subcutaneous lesions
3. Excisional biopsy for complete lesion removal

5. Immunofluorescent staining of biopsied skin lesions may be useful in diagnosis of autoimmune and/or vasculitic disorders.

D. Management
1. General concepts

a. Absorption of topical agents through the skin of infants and small children is greater than in adults owing to the increased body surface area to weight ratio that children have as compared with adults. This is especially true for premature infants in whom there is also greater absorption through the skin owing to a thinner stratum corneum.
b. Therapeutic efficacy of a topical agent is related to both the active ingredient and the vehicle (e.g., cream, lotion, ointment).

2. Hydration of the skin is critical. Dry, irritated skin is a poor barrier and easily damaged. Moisturizers include the following:

a. Ointments contain little or no water and have maximal water-retaining properties,
making them useful for very dry skin.
b. Creams contain 20–50% water and are useful for skin of average dryness.
c. Lotions contain more water than creams and are useful for minimally dry skin or for large surface areas.
d. Solutions and alcohol-based gels are most useful for hair-bearing areas (e.g., scalp).

3. Thickened skin (hyperkeratosis) responds well to treatment with keratolytics, such as salicylic acid, urea, α-hydroxy acids, and retinoic acid.
4. Destructive therapies (e.g., for warts, molluscum contagiosum) include high-dose salicylic acid, cantharidin, podophyllin, 5-fluorouracil, cryotherapy, electrotherapy, and laser therapy.
5. Anti-infective agents include topical antibiotics, antifungals, antivirals, and antiparasitic agents.
6. Anti-inflammatory agents

a. Topical corticosteroids are categorized on the basis of their vehicle and potency. Effectiveness and potential side effects mirror steroid potency. The weakest steroid that will achieve the treatment goal should be used first. In general, only low- potency corticosteroids should be used on the face or groin because the epidermis is thinner in these areas, resulting in an increased risk of side effects.
1. Systemic side effects. Systemic steroid levels can be achieved if very potent steroids are used on damaged or thin skin for long periods of time. Rare side effects may include adrenal suppression, depressed growth, cataracts, glaucoma, and Cushing syndrome (see also Chapter 6, section IV.E.2).
2. Local side effects are more common and are also more likely to occur after long-term use of topical corticosteroids.

a. Acne (acne rosacea and steroid acne)
b. Hirsutism
c. Folliculitis
d. Striae (especially in the axilla or groin)
e. Hyper- or hypopigmentation
f. Skin atrophy
g. Ecchymoses and telangiectasias
h. Tachyphylaxis (insensitivity to the medication)

b. Other topical anti-inflammatory agents
1. Tacrolimus ointment and pimecrolimus cream are topical calcineurin inhibitors/immunomodulators used to treat atopic dermatitis, psoriasis, and other inflammatory skin conditions.
2. One to five percent sulfur, formulated with other medications, for acne
3. Tar, used for eczema and psoriasis
4. Antibiotics, used for anti-inflammatory properties to treat acne and perioral dermatitis.

II. Newborn Skin Diseases

See Chapter 4, section I.K.

III. Inflammatory Disorders
A. Contact dermatitis
1. Definition. Contact dermatitis is inflammation of the epidermis and superficial dermis secondary to direct contact with the skin by a sensitizing substance.
2. Categories
a. Allergic contact dermatitis
1. Etiology. Allergic contact dermatitis occurs as a direct T-cell–mediated response to an exogenous applied allergen. There must be an initial sensitization and then a rechallenge (which may be very small and is not dose- dependent) to elicit a reaction. Common causes include poison ivy, oak, or sumac; nickel found in jewelry, belt buckles, or snaps; topical lotions or creams; and perfumes or soaps.
2. Clinical features. Erythematous pruritic papules and vesicles occur in the area that came into contact with the allergen.
3. Management. Treatment involves topical corticosteroids and avoidance of the offending allergen.
b. Primary irritant contact dermatitis
1. Etiology. Primary irritant contact dermatitis is caused by caustic substances that irritate the skin, rather than an allergic reaction. No prior sensitization is required. The reaction is dose-dependent. The most common form is diaper dermatitis, a multifactorial disorder caused by prolonged contact with urine and fecal matter, friction, maceration, and proteases contained in the feces and urine. Secondary infection with Candida albicans may occur.
2. Clinical features of diaper dermatitis include erythema with papules on the skin that is exposed to the diaper surface, including the upper thighs, buttocks, and genitourinary area without involvement of the inguinal creases. Involvement of the inguinal creases, more-intense confluent erythema, and satellite lesions all suggest candidal superinfection.
3. Management of diaper dermatitis includes protecting the skin from urine and stool; application of skin moisturizers, barrier creams, and ointments (e.g., zinc oxide); and frequent diaper changes. Low-potency corticosteroids may be used for severe inflammation. Candidal infection may be treated with topical antifungal medication (e.g., nystatin, clotrimazole).
B. Atopic dermatitis (see Chapter 15, section III)
C. Seborrheic dermatitis
1. Epidemiology. Seborrheic dermatitis predominantly affects two age groups, infants and adolescents.
2. Etiology. The cause is unknown but is thought to be a hypersensitivity reaction to a saprophytic yeast (Pityrosporum ovale) that lives in areas that overproduce sebum.
3. Clinical features. Erythema with overlying “greasy” thick yellow-white scale and sometimes crusting in areas with high numbers of sebaceous glands, such as the scalp, face (including the eyebrows, nose, and beard area), chest, and groin
a. Infants may have dermatitis limited to the scalp, termed seborrheic capitis or cradle cap. Involvement of the face, upper chest, and flexor creases of the extremities may also occur.
b. Adolescents may have dermatitis in the nasolabial folds, pinnae, and scalp.
4. Management
a. Low-potency topical corticosteroids

b. Sulfur-, zinc-, or salicylic acid–based shampoos may be used, followed by light scrubbing with a brush to remove crusts. Loose scales may also be removed with mineral oil.
c. Topical antiyeast medications (e.g., clotrimazole) or shampoos (containing zinc pyrithione or ketoconazole) are used to eradicate Pityrosporum ovale.
D. Pityriasis rosea
1. Epidemiology. Pityriasis rosea is uncommon before 5 years of age but is extremely common during late childhood and adolescence.
2. Etiology. The cause is unknown, although it is thought to be an immune response to a virus.
3. Clinical features
a. Papulosquamous skin lesions that begin with a solitary, large 2- to 5-cm scaly, erythematous lesion (herald patch) that is usually located on the trunk or extremities. The herald patch is present for 1–30 days.
b. Approximately 1–2 weeks after the appearance of the herald patch, similar smaller oval pink scaling macules and papules erupt for approximately 3–6 weeks on the trunk and extremities, following skin lines in a “christmas tree” distribution on the trunk.
c. Lesions are pruritic in 50% of cases.
4. Management. Treatment may include topical or systemic antihistamines as well as topical corticosteroids or immunomodulators to help treat pruritus. Exposure to ultraviolet light may shorten the disease course.
E. Psoriasis
1. Epidemiology. Psoriasis occurs in 3% of children in the United States. Although it is more common in adults, 30% of patients develop signs and symptoms during childhood.
2. Etiology. Childhood-onset psoriasis is often a genetic condition. It is caused by immune dysregulation, causing epidermal proliferation and skin inflammation.
3. Clinical features
a. Distribution of skin lesions and severity are variable. If severe, psoriasis may be disfiguring.
b. Lesions are characterized by erythematous scaling papules and plaques often found on the scalp (nongreasy scale without hair loss), ears, elbows, knees, lumbosacral area, and groin. Lesions often have a classic silvery scale.
c. Lesions often demonstrate the Koebner phenomenon in which new lesions develop at sites of skin trauma.
d. Nail involvement is common and may include pits, distal thickening, lifting of the nail bed, and nail destruction.
e. Arthritis during childhood is uncommon.
4. Management. Treatment may include moderate- or high-potency topical corticosteroids, topical calcineruin inhibitors, ultraviolet light therapy, vitamin D analogues, topical salicylic acid, tar, retinoids, and anthralin (downregulates epidermal growth factor).
F. Miliaria rubra (heat rash)
1. Etiology. Heat rash is caused by disrupted sweat ducts near the upper dermis (often caused by occlusion or friction) that result in sweat being released onto the skin. The sweat on the skin produces an inflammatory response. Therefore, the more sweat produced and the more occlusion, the more likely heat rash will develop.
2. Clinical features. Small erythematous pruritic papules or vesicles occur in areas of occlusion or in areas that have been rubbed, such as the inguinal region, axilla, chest, and neck.
3. Management. Treatment is avoidance of occlusive clothing to decrease sweating.

Medications are unnecessary.

IV. Hypersensitivity Disorders
A. Urticaria (see Chapter 15, section VI)
B. Serum sicknesslike reaction may initially mimic urticaria, but it tends to produce skin lesions with a more annular (ring-shaped) appearance with a dusky or bruised-looking central component. It is associated with systemic signs and symptoms including fever, arthralgias, adenopathy, and evidence of organ injury. Viral triggers and medications, such as cephalosporins, are common causes.
C. Erythema multiforme (EM) and Stevens–Johnson syndrome (SJS)
1. Definition. EM is a hypersensitivity reaction to many potential stimuli, including drugs, viruses, bacteria, fungi, protozoa, and systemic disease.
2. Categories. There are two major categories of EM: EM minor and EM major. SJS is now considered to be a separate entity from EM. The classic skin lesion present in both EM and SJS is a target lesion, which is a fixed, dull red, oval macule with a dusky center that may contain a papule or vesicle. Table 19-1 summarizes the causes, clinical features, management, and prognosis of EM and SJS.
D. Toxic epidermal necrolysis is a severe reaction to medications (e.g., anticonvulsants, antibiotics, anti-inflammatory drugs) that results in widespread epidermal necrosis. Clinical features include sloughing of the epidermis (usually >30% skin loss) and severe mucous membrane involvement. Target lesions are usually not seen. Nikolsky sign (skin peels away with lateral pressure) is often present. Mortality is high (10–30%) as a result of the high incidence of complications due to sepsis, dehydration, and electrolyte abnormalities.

Table 19-1
Characteristic Features of the Types of Erythema Multiforme and Stevens–Johnson Syndrome

Erythema Multiforme Minor Erythema Multiforme Major Stevens–Johnson Syndrome
Major cause Herpes simplex virus Mycoplasma pneumoniae;
Medications Medications
Skin findings Symmetric target lesions; acral
distribution Typical symmetric target lesions;
acral and truncal distribution Widespread atypical, asymmetric
target lesions, blisters, and necrosis
Mucous membrane
findings Occurs in 25%; only one
surface involved (often mouth) At least two mucosal surfaces involved (often mouth and eyes) At least two mucosal surfaces involved (often mouth and eyes)
Systemic
findings Prodrome of low-grade fever,
arthralgias, myalgias Prodrome of low-grade fever,
arthralgias, myalgias Prodrome of high fever, cough,
malaise, headache, arthralgias
Management Supportive care Supportive care Supportive care
Acyclovir may prevent
recurrence Erythromycin or azithromycin if
M. pneumoniae is suspected Stop offending drug
Ophthalmology consultation
Stop offending drug Consider steroids, IVIG, burn unit
Prognosis Good; possible recurrence Good High morbidity and mortality (5%)
IVIG = intravenous immune globulin.

V. Infections of the Skin
A. Fungal (dermatophyte) infections can involve hair, skin, and nails.
1. Tinea capitis is a fungal infection of the scalp hair.
a. Etiology. Tinea capitis is most commonly caused by Trichophyton tonsurans (95%), acquired from human-to-human contact, and Microsporum canis (5%), often acquired from animals, such as cats and dogs.
b. Clinical features
1. Patchy hair loss in which the hairs break off at the scalp (black dot ringworm) or in which the broken hairs may be thickened and white
2. Infected areas may have pustules and scale.
3. Kerion (large red boggy nodule/plaque) may be present and represents a hypersensitivity reaction to the dermatophyte.
4. Occipital and posterior cervical lymphadenopathy are suggestive of tinea capitis.
c. Diagnosis. The basis of diagnosis is microscopic evaluation of hairs with 10% KOH to identify fungal hyphae or a positive fungal culture. Hairs fluoresce under Woods light if M. canis is the infecting organism.
d. Management. Treatment includes systemic oral antifungal therapy (e.g., griseofulvin or terbinafine) for 6–8 weeks. Topical antifungal agents are ineffective for infections of the hair. Topical 2.5% or 5% selenium sulfide shampoo may be used as an adjunct to reduce infectivity.
2. Fungal infection of the skin may include tinea corporis (infection on the body), tinea pedis (infection on the foot), tinea cruris (infection in the groin), and tinea faciei (infection on the face).
a. Etiology. Pathogens include M. canis, T. tonsurans, and other Trichophyton species.
b. Clinical features
1. Tinea corporis (“ringworm”) presents as a circular or annular scaly erythematous patch with partial central clearing and an “active” border, which means that the advancing edge or border of the skin lesion is more erythematous and/or scaly than the central portion.
2. Tinea pedis (athlete’s foot) presents most commonly in postpubertal adolescents with scaling, erythema, and maceration between the toes or on the plantar aspect of the foot. Vesicles may also be seen.
3. Tinea cruris usually presents as well-defined scaling and erythema in the groin and inguinal creases.
c. Diagnosis. Clinical features are often the basis of diagnosis. 10% KOH examination of skin scrapings for fungal hyphae or fungal culture can confirm the diagnosis.
d. Management. Treatment includes topical antifungal medications (e.g., terbinafine, ketoconazole, itraconazole).
3. Tinea unguium (onychomycosis) is a fungal infection of the nails characterized by thickening and yellow discoloration of one or several nails (usually toenails). Topical management is not very effective; treatment requires prolonged therapy and is often unsuccessful as onychomycosis tends to recur. Systemic medications, such as griseofulvin, terbinafine, and itraconazole, are therapies of choice and are given for long courses (12+ weeks).
4. Tinea versicolor is a common disorder seen more often in adolescents and young adults. It is caused by a yeast of the Malassezia species (rather than a fungus) that invades the stratum corneum.

a. Clinical features vary and include fine, scaly oval macules on the trunk, proximal arms, and face. Macules may be hypo- or hyperpigmented and become more prominent with sun exposure. Infection is usually asymptomatic.
b. Diagnosis is usually made by clinical examination but can also be made by identification of fungal hyphae or circular spores on KOH examination of skin scrapings (“spaghetti and meatballs” appearance) or by yellow-green fluorescence on Woods light evaluation. The Malessezia yeast does not grow well on culture.
c. Management. Treatment includes overnight application of 2.5% selenium sulfide weekly for 3–4 weeks, ketoconazole shampoo or cream, or systemic antifungal medications (ketoconazole, itraconazole).
B. Bacterial infections, including impetigo, cellulitis, erysipelas, scarlet fever, toxic shock syndrome, and staphylococcal scalded skin syndrome, are discussed in Chapter 7, section IX.
C. Viral infections
1. Viral exanthem refers to a skin rash associated with a viral infection. Any virus may cause an exanthem, and the rash may take many forms. An enanthem (describing involvement of the oral mucosa) may also be present.
a. Morbilliform: refers to a “measleslike” appearance with diffuse involvement of red macules on the skin
b. Papulovesicular: refers to a rash composed of papules and vesicles
2. Measles and rubella (see Chapter 7, sections XIII.C and D)
3. Erythema infectiosum (fifth disease)
a. Epidemiology. Fifth disease is most common in school-age children, although it can occur at any age.
b. Etiology
1. The cause is parvovirus B19.
2. Fifth disease is transmitted by respiratory secretions.
c. Clinical features
1. Fifth disease begins with upper respiratory symptoms (cough, fever, rhinorrhea) that are followed within 1–2 weeks by a bright red macular rash on the cheeks (“slapped-cheek” appearance) that lasts several days. Patients are generally no longer contagious when the characteristic facial rash appears.
2. A lacy, reticulate rash on the limbs and then the trunk follows the facial rash and generally lasts 3–5 days. In some patients, exercise, heat, or sunlight can induce the lacy rash to recur.
3. Arthralgias may be present, although they are more common in adults.
4. Parvovirus B19 infection may also cause aplastic crisis (especially in patients with hemoglobinopathies), prolonged anemia in immunosuppressed patients, and fetal hydrops or miscarriage in pregnant women.
d. Management. Treatment is supportive. However, intravenous immune globulin may be used treat prolonged anemia in immunosuppressed patients.
4. Roseola infantum (exanthem subitum)
a. Epidemiology. Roseola is most common in children between 6 months and 3 years of age.
b. Etiology. Roseola is most commonly caused by human herpes virus 6 and 7. Other causes include adenovirus, parvovirus B19, and echovirus 16.
c. Clinical features. Roseola begins with 3–5 days of high fever and irritability. Once the fever resolves, a usually asymptomatic pink papular eruption occurs on the trunk that generally fades within 24–48 hours.
d. Management. Treatment is supportive.

5. Gianotti–Crosti syndrome (papular acrodermatitis) most often occurs in children between the ages of 6 months and 12 years. Gianotti–Crosti is associated with hepatitis B, Epstein–Barr virus, and enterovirus and echovirus infections. Clinical features include the development of red or flesh-colored flat-topped papules on the extremities with accentuation over the elbows and knees. The cheeks and buttocks can also be involved, but the rash classically spares the trunk. Skin lesions may last for several weeks and may recur. Upper respiratory symptoms may precede the eruption. Treatment is supportive.
6. Varicella (chickenpox)
a. Epidemiology. The incidence of varicella has decreased as a result of routine early childhood immunization in many states. Varicella may occur at any age in unimmunized children and adolescents.
b. Clinical features
1. Intensely pruritic erythematous macules develop acutely after a 7- to 21-day incubation period. The macules develop central vesicles within 1–2 days. The classic lesion is described as a “dew drop on a rose petal” or a vesicle on a red background.
2. Crops of lesions appear during the course of 2–5 days. Hundreds of vesicles may be present, which will crust over.
3. Fever is common.
c. Management. Treatment includes antipyretics, management of bacterial superinfection, antihistamines for itching, and monitoring and treatment of complications. Acyclovir is generally administered intravenously for patients with varicella pneumonia and encephalitis, orally for those at high risk for complications, and topically in the eyes for those with ophthalmic involvement.
d. Complications (Table 19-2)
7. Herpes simplex virus infection (HSV-1 and HSV-2)
a. Pathophysiology
1. Neonatal infection is generally acquired during passage through the birth canal of a mother with primary HSV infection. Two-thirds of neonatal infections are caused by HSV-2 and one-third are caused by HSV-1.
2. Gingivostomatitis is the most common HSV infection during infancy and childhood and is almost always caused by HSV-1.
b. Clinical features
1. Characteristic lesions are grouped vesicles on an erythematous base.
2. Gingivostomatitis presents most often in young infants with grouped vesicles and ulcers on the lips, in the corners of the mouth, and on the tongue. Pain on swallowing, drooling, and fever may be present. Infection lasts 1–2 weeks.
3. Neonatal HSV commonly presents in the first week of life with variable signs and symptoms. Neonates may have only a few vesicles at the site (often the scalp) that was in contact with the infecting maternal lesion, or they may present with signs and symptoms of sepsis, including apnea, lethargy, irritability, and seizures. Serious sequelae include meningoencephalitis, hepatitis, sepsis, shock, and death.
4. Herpetic whitlow describes HSV-1 infection of the thumb or fingers that is usually secondary to thumb- or finger-sucking by a child with an oral HSV lesion.
5. HSV resides in the dorsal root ganglion after initial infection, and therefore, recurrent HSV infection may occur. Recurrent HSV lesions are more mild and less symptomatic than primary infection and generally occur on the lip. Fever, illness, ultraviolet light exposure, emotional stress, and trauma may all

reactivate the latent virus.
c. Diagnosis. HSV may be diagnosed by identification of epidermal giant cells on microscopic evaluation of a Tzanck preparation, by detection of HSV antigen on direct fluorescent antibody (DFA) testing, or by culture of the base of the lesion. Polymerase chain reaction (PCR) technology is commonly used to identify HSV in the cerebrospinal fluid.
d. Management
1. Neonatal HSV infection is a medical emergency and requires immediate hospitalization and treatment with intravenous acyclovir.
2. Cutaneous and oral HSV may be treated with oral acyclovir, although treatment must be started promptly to alter the disease course. Oral acyclovir, when given daily, may also prevent recurrent infection.
8. Hand–foot–mouth disease and herpangina
a. Etiology. Hand–foot–mouth disease and herpangina are caused by infection with enterovirus, usually coxsackievirus types A16 (most common), A2, A5, A6, and A10. It is usually seen in children less than 5 years of age.
b. Clinical features include vesicles, papules, or pustules on the palms, soles, or fingertips and shallow ulcers or erosions on the soft palate or tongue. The rash also sometimes affects the buttocks. If only oral lesions are present, the disorder is termed herpangina. Fever may occur with both forms. In some cases, shedding of finger- and toenails, called onychomadesis, is observed a few weeks after the infection.
c. Management. Treatment is supportive.
9. Warts
a. Etiology. The cause is human papillomavirus.
b. Clinical features. Warts may occur on any skin surface and appear as irregularly shaped discrete flesh-colored papules that may be smooth or rough. Warts often increase in size, are contagious, and may spread to adjacent skin. Condylomata acuminata is the term used to describe multiple external warts in the genital area (see Chapter 3, section VII.B.6).
c. Management. Most warts resolve spontaneously within 1–2 years. Treatment to remove or destroy warts may include liquid nitrogen, salicylic acid, cantharidin, podophyllin, laser therapy, and surgical excision. Newer immune therapies are also used to treat warts, which include injections with candida antigen and topical agents used to stimulate the immune system (e.g., squaric acid, imiquimod). Recurrence after any treatment is high.
10. Molluscum contagiosum
a. Etiology. The cause is a poxvirus.
b. Clinical features
1. Small asymptomatic flesh-colored papules with central umbilication are characteristic.
2. Lesions may be present anywhere on skin with hair follicles, although the proximal extremities and trunk are most commonly involved.
3. Lesions are contagious.
4. Human immunodeficiency virus (HIV) infection may be associated with extensive eruptions of molluscum.
c. Management. Treatment is often observation with expected resolution over months to years without therapy. Removal can be accomplished by curettage or application of cantharidin, podophyllin, trichloroacetic acid, liquid nitrogen, or salicylic acid.
D. Ectoparasites

1. Louse infestation may involve the scalp (head lice), body (body lice), or groin (pubic lice).
a. Etiology. Causative organisms include Pediculus humanus, the cause of head and body lice, and Phthirus pubis, the cause of pubic lice. The louse is a small six-legged insect that attaches to the skin and ingests blood.
b. Epidemiology. Louse infestations are associated with crowded living conditions and sharing of hats, clothes, combs, and hairbrushes.
c. Clinical features
1. Head lice are associated with itching. The nits (eggs) may be seen as oval white bodies attached to the hair shaft. The louse may be found on the scalp.
2. Body lice are associated with papules and pustules on the trunk with excoriations.
3. Pubic lice are associated with lice or nits in the groin and black-crusted papules or blue macules (macula cerulea).
d. Management
1. Head lice are treated with 1% permethrin shampoo and a comb to remove the nits. Five percent permethrin, malathion, ivermectin, and therapies meant to suffocate the lice are sometimes used for resistant lice.
2. Body and pubic lice are treated with topical agents that contain malathion, permethrin, or a pyrethrin.
2. Scabies
a. Etiology. Scabies is caused by the mite Sarcoptes scabiei.
b. Clinical features. Pruritic papules or vesicles are most commonly located on the abdomen, dorsum of the hands, groin, axilla, flexor surfaces of the wrists, and interdigital spaces. Infants may have facial and neck involvement. Itching is severe, and S-shaped burrows may be seen.
c. Diagnosis. Microscopic examination of a scraping from an unscratched burrow demonstrating the mite, eggs, or mite feces is diagnostic.
d. Management. Treatment includes an overnight application of 5% permethrin lotion or 1% lindane (adolescents and adults only). Scabies is highly contagious, and therefore, all household contacts should be treated. Treatment should be repeated 7–10 days after the first application of the antiscabetic to catch the newly hatched mites. Itching may persist for up to 30 days after treatment. All bed sheets, pillowcases, and clothing should be washed in hot water.

Table 19-2
Complications of Varicella Infection

Bacterial superinfection (often Staphylococcus aureus)
Necrotizing fasciitis (often Group A Streptococcus)
Scarring
Reye syndrome (associated with simultaneous ingestion of salicylates/aspirin)
Pneumonia
Encephalitis
Acute cerebellar ataxia
Hepatitis
Herpes zoster (reactivation of virus)
Infection during pregnancy:
Teratogenic effects (“congenital varicella syndrome” due to maternal infection during the first 20 weeks of gestation: zigzag scarring of the skin, shortened or malformed extremities, central nervous system damage, and eye abnormalities such as
cataracts or chorioretinitis)
“Neonatal varicella” is severe varicella infection in a neonate due to peripartum maternal infection within 1 week of delivery or
postnatal exposure. May be fatal.

VI. Pigmentary Disorders
A. Hypopigmentation. Causes include the following:
1. Postinflammatory hypopigmentation may follow any skin inflammation (e.g., atopic dermatitis) and generally resolves over months to years.
2. Pityriasis alba is thought to be related to atopic dermatitis. It is characterized by hypopigmented, dry, scaly patches, most commonly on the cheeks. Treatment includes moisturizers, sun protection, and low-potency topical corticosteroids or immunomodulators.
3. Vitiligo is a complete loss of skin pigment in patchy areas that is caused by autoimmune melanocyte destruction. Treatment with high-potency topical steroids, topical immunomodulators, and/or ultraviolet light may sometimes produce repigmentation.
4. Oculocutaneous albinism is caused by a genetic defect in melanin synthesis. Clinical features include white skin and hair, blue eyes, and other eye findings, such as photophobia and nystagmus. There is no treatment.
B. Neurocutaneous disorders. Table 19-3 summarizes the clinical features of tuberous sclerosis and neurofibromatosis.
C. Nevocellular nevi are pigmented lesions that may be congenital or acquired.
1. Congenital nevi. These black, brown, tan, or flesh-colored papules or plaques are first detected between birth and 6 months of age. They occur in 1–2% of neonates. All congenital nevi may have an increased risk of malignancy, but giant nevi (>20 cm in diameter) have a 6–7% lifetime risk of development of malignant melanoma. Management of giant nevi often includes excision, if possible, or careful observation.
2. Acquired nevi (moles). Peak ages of development are 2–3 years and 11–18 years of age. Moles are characterized as well-demarcated brown or black papules that increase in size and number during puberty or pregnancy and following sun exposure. Most acquired nevi in childhood are junctional nevi. The risk of malignant transformation is much lower for acquired than that for congenital nevi. Management is careful observation.

Table 19-3
Clinical Features of Two Neurocutaneous Syndromes

Tuberous Sclerosis Neurofibromatosis Type 1 (NF-1)*
Inheritance Autosomal dominant Autosomal dominant
Skin findings Ash-leaf spots (hypopigmented macules seen
best under Woods light) Café-au-lait spots (Figure 19-1)
Axillary or inguinal freckling
Angiofibromas on nose or face (adenoma sebaceum) Plexiform neurofibroma or skin
neurofibromas (soft flesh-colored nodules)
Shagreen patch (thickened orange
peel appearance)
Ungual fibromas
CNS findings Seizures (95%), including infantile spasmsIntracranial calcifications
Cortical or subependymal tubers Optic glioma (usually present by
3 years of age)Intracranial calcifications CNS neurofibromas
Systemic findings Renal cysts Lisch nodules (iris hamartoma)
Cardiac rhabdomyomas (leading cause of neonatal cardiac tumors) Osseous lesions (present by 1 year of age): sphenoid dysplasia or thinning of long bone
cortex
Retinal astrocytoma or hamartoma Scoliosis
Mental retardation Hypertension
Learning problems

*Neurofibromatosis type 2 accounts for 10% of all cases of neurofibromatosis and is characterized by bilateral acoustic neuromas. Skin findings of café-au-lait spots and neurofibromas are less common findings as compared with neurofibromatosis type 1.
CNS = central nervous system.
Note: Diagnosis of neurofibromatosis type 1 requires two of the following seven clinical findings: six or more café-au-lait spots (≥5 mm in children or 15 mm in adults); two or more neurofibromas or one plexiform neurofibroma; freckling in the axilla or groin; optic glioma; two or more Lisch nodules; characteristic osseous lesion; and first-degree relative with neurofibromatosis type 1.

FIGURE 19.1 In pediatric patients with neurofibromatosis type 1, café-au-lait spots usually are larger than 55 mm in diameter, the edges are well defined, and the intensity of the coloration is uniform.
Reprinted with permission from McMillan JA, Feigin RD, DeAngelis C, Jones MD. Oski’s Pediatrics: Principles and Practice. Philadelphia: Lippincott Williams & Wilkins, 2006.

VII. Disorders of the Hair
A. Alopecia areata
1. Etiology. The cause is thought to be autoimmune lymphocyte-mediated injury to the hair follicle.
2. Epidemiology. Alopecia areata affects 1 in 1000 persons.
3. Clinical features
a. Complete hair loss occurs as sharply demarcated areas of alopecia without inflammation or scaling. The hair loss is described as occurring suddenly, and the underlying skin is smooth and soft. The hair loss can occur on the scalp or other hair-bearing areas such as the beard, eyebrows, eyelashes, etc.
b. Pitting of the nails occurs in 40% of patients.
c. Subtypes of alopecia areata include alopecia totalis (loss of all scalp hair) and
alopecia universalis (loss of all body and scalp hair).
4. Management. Most patients have regrowth of hair within months to years without any treatment. Hair regrowth may be stimluated with the use of topical or injected corticosteroids, topical minoxidil, and topical irritants (anthralin, squaric acid). Wigs and counseling may be necessary owing to the psychological consequences of the significant hair loss.
B. Tinea capitis is a common cause of hair loss [see section V.A.1].
C. Traumatic alopecia is also a common cause of childhood hair loss and may be one of the following two types:
1. Trichotillomania is hair loss that occurs as a result of conscious or unconscious pulling or twisting of hair. Clinical features include irregularly bordered areas of hair loss in which hairs are broken off at different lengths. The scalp may show perifollicular petechiae and excoriations. Eyelashes and eyebrows may also be involved. The cause is unknown, although it may be associated with anxiety. Management includes stress management or counseling and investigation into precipitating events.
2. Traction alopecia is hair loss caused by constant traction or friction and may be the result of tight hair braids, curlers, vigorous scalp massage, or constant rubbing. Clinical features include patchy and jagged, irregularly shaped areas of alopecia especially along the hair line, with thinned, small hairs, and broken hairs. Management is to stop the inciting trauma.
D. Telogen effluvium is the second most common type of alopecia after male pattern baldness. It is caused by any acutely stressful event (e.g., pregnancy, surgery, acute illness, high fever, trauma) that converts hairs from a growing phase (anagen) to a final resting phase (telogen). Clinical features include complaints of generalized excessive hair loss (hair loss > 100 hairs per day compared to normal hair loss of 50–100 hairs per day) 1–3 months after the precipitating event. Hair loss continues for 3–4 months, and then spontaneous regrowth occurs.
E. Other conditions that cause hair loss include hypothyroidism, diabetes mellitus, hypopituitarism, nutritional disorders (e.g., hypervitaminosis A, marasmus, zinc deficiency also known as “acrodermatitis enteropathica”), medications (e.g., warfarin, heparin, chemotherapy, cyclophosphamide, isotretinoin), ectodermal dysplasias, and hair shaft structural defects.

VIII. Acne Vulgaris
Acne is the most common skin disease, and its sequelae may include scarring and disfigurement with resultant adverse effects on psychosocial development, such as depression and other emotional problems.

A. Pathophysiology. Acne is caused by a combination of the following processes:
1. Excessive shedding and cohesion of cells that line the sebaceous follicles located on the chest, back, and face
2. Increased production of sebum by sebaceous glands under the influence of androgens. Obstruction of sebum outflow from the follicle leads to the formation of comedones.
3. Inflammation as a result of the proliferation of the bacteria, Propionibacterium acnes, and the host immune response to the bacteria
B. Clinical features
1. Acne usually begins 1–2 years before puberty.
2. Noninflammatory (comedonal) acne is characterized by open comedones (blackheads) and closed comedones (whiteheads).
3. Inflammatory acne is characterized by erythematous papules, pustules, nodules, and cysts.
4. Most patients have a combination of comedomal and inflammatory acne.
C. Management. Treatment is individualized based on the disease severity and on the location and type of lesions.
1. Benzoyl peroxide, retinoids (Retin-A) (particularly useful for comedonal acne), sulfur, and salicylic acid are effective topical agents for acne.
2. Antibiotics (oral or topical) are used in combination with the aforementioned medications for more inflammatory acne.
3. Oral contraceptives are helpful in treating acne in teenage girls and young adult women.
4. Systemic isotretinoin (Accutane) is highly effective for all types of acne, including nodular and cystic acne. Females must be tested for pregnancy before and during treatment, and must use effective birth control during treatment to prevent pregnancy because of the risk of teratogenic effects associated with the use of systemic isotretinoin.

IX. Vascular Skin Lesions
A. Nevus simplex/flammeus, often called “stork bites,” are congenital capillary malformations present in about 25–50% of newborns that are located on the midline upper face, scalp, and nuchal area. The lesions usually fade over the first 1–2 years of life.
B. Infantile hemangiomas are common vascular skin lesions seen in approximately 5% of infants.
1. Clinical features
a. Hemangiomas often appear shortly after birth and exhibit a growth phase lasting up to 5 months, followed by slow involution (shrinkage and loss of color) over several years.
b. Hemangiomas can present anywhere on the skin (as well as in internal organs such as the liver) as red papules and plaques, deep blue nodules, or both.
2. Management. Most hemangiomas do not require treatment, but treatment can be considered for those that are likely to cause disfigurement, as in the case of facial lesions, or obstruction of vital structures, as can be seen in the periocular location, or when they arise in the airway. Treatments may involve the use of topical and oral β-blockers to prevent growth and speed involution, or laser therapy.
C. Port wine stains (capillary malformations) are present at birth and do not proliferate or involute. They can present anywhere on the skin and are associated with certain syndromes when found in specific locations (e.g., Sturge–Weber Syndrome when located on the face in the distribution of the first or second branches of the trigeminal nerve). Port wine stains can be lightened with laser treatments.
D. Pyogenic granulomas are proliferations of capillaries on the skin, which often arise in response to some sort of trauma. They are most commonly seen during the first 5 years of life and often present as red friable glistening papules or nodules that bleed easily. Surgical removal is the treatment of choice, but laser can be used for small lesions.

Review Test
1. A 1-week-old female infant has several vesicles on her scalp. She also has a 1-day history of fever (temperature up to 38.2°C [100.8°F]) and irritability. She is breastfed and is eating slightly less vigorously than usual. The mother’s pregnancy was uncomplicated. Which of the following statements regarding the presumptive diagnosis is correct?
A. Herpes simplex virus type 1 is the most likely causative pathogen.
B. The infection was likely acquired after birth.
C. A Tzanck smear of the base of one of the vesicles may demonstrate epidermal giant cells.
D. Oral acyclovir should be started immediately.
E. Oral antibiotics to cover staphylococcal and streptococcal infection should be started promptly.
2. A 9-year-old girl is brought to the office by her parents. She has a 1-month history of two large well-demarcated areas of hair loss in the parietal scalp. On examination, no inflammation or scaling of the skin is apparent, and the underlying skin is smooth and soft. Which of the following statements regarding the likely diagnosis is correct?
A. The fingernails are likely to be normal.
B. Autoimmune destruction of the hair follicle is likely the cause of this disorder.
C. Oral griseofulvin should be prescribed after cultures.
D. The child and her parents should be counseled to loosen her braids and ponytails when styling her hair.
E. The hair loss is likely to be permanent.
3. A 6-year-old boy presents with a 1-month history of patchy alopecia in the occipital region of the scalp. Examination reveals a well-circumscribed area of hair loss in which all the hairs appear to be broken off at the scalp surface. Occipital lymph nodes are prominent. Which of the following statements regarding the likely diagnosis is correct?
A. This infection is contagious, and brushes, combs, and hats should not be shared.
B. Topical management with clotrimazole is the initial appropriate treatment.
C. Exposure to dogs or cats is the most likely cause of infection.
D. Hairs are likely to fluoresce under Woods light examination.
E. Oral antifungal therapy should be administered for 2 weeks.
4. A 5-year-old boy presents with a 6-week history of scaling, nongreasy erythematous plaques in the occipital region of the scalp and in the inguinal region. The lesions have a “silvery scale” appearance. Which of the following statements regarding the likely disorder is correct?
A. Low-potency corticosteroids are likely to be effective in treating this condition.
B. It would be unusual for other family members to also have this disorder.
C. New lesions may develop at any site of skin trauma.
D. Nail changes would not be expected with this condition.
E. Arthritis is likely to occur in this patient.
5. A 10-year-old girl presents with a history of malaise and a headache that was followed 4 days later with a body rash. Examination reveals a 3-cm scaly erythematous plaque on the upper arm and oval, red macules and papules on the back that follow skin lines. Which of the following management steps is the least appropriate?
A. Therapy with topical corticosteroids for pruritus
B. Natural ultraviolet exposure
C. Topical clotrimazole
D. Oral antibiotics
E. Reassurance only
6. A 4-month-old male infant is brought to the office by his parents for a routine health

maintenance examination. On examination, you note significant hypopigmentation in the inguinal area. Medical history is remarkable for suspected diaper dermatitis treated with zinc oxide and high-potency corticosteroids for 3 weeks. Which of the following statements regarding this patient and his clinical findings is most correct?
A. Topical antifungal therapy should now be prescribed to treat suspected fungal superinfection.
B. Zinc oxide should be immediately discontinued.
C. Atrophy of skin in the inguinal area may also be present.
D. The child should be referred to genetics to consider the diagnosis of tuberous sclerosis.
E. Ultraviolet light therapy should be prescribed.
7. A 4-year-old boy with a history of atopic dermatitis presents for evaluation of his skin. Examination shows that the skin on his upper and lower extremities is very dry and irritated. You would like to suggest a moisturizer in addition to topical corticosteroids. Which of the following types of moisturizers is most appropriate?
A. Lotion
B. Cream
C. Ointment
D. Solution
E. Gel
8. A 13-year-old boy is brought to the office by his parents because of concerns regarding his acne. Examination reveals scattered open and closed comedones on the forehead and nose. Which of the following statements regarding this condition is correct?
A. Topical benzoyl peroxide is an appropriate first-line treatment.
B. Oral isotretinoin is an appropriate first-line treatment.
C. Oral clindamycin is an appropriate first-line treatment.
D. Sebum plays little role in the pathophysiology at this stage of acne.
E. Reassurance only should be given because the patient’s skin findings are currently of little consequence.

For statements 9–13, the response options are the same. You will be required to select one answer for each statement in the following set.

A. Papule
B. Macule
C. Pustule
D. Nodule
E. Vesicle
F. Cyst
G. Wheal
H. Target lesion
I. Plaque

Match the disorder with its most characteristic skin lesion.

1. A 3-year-old boy with herpes simplex virus infection involving the upper and lower lips.
2. A 7-year-old boy with suspected vitiligo involving both hands.
3. A 15-month-old girl with molluscum contagiosum on the cheeks and forehead.
4. A 6-year-old boy with erythema multiforme major.
5. A 10-year-old girl with suspected Gianotti–Crosti syndrome.

For statements 14 and 15, the response options are the same. You will be required to select one

answer for each statement in the following set.

A. Neurofibromatosis type 1
B. Neurofibromatosis type 2
C. Tuberous sclerosis

For each patient, select the most likely associated neurocutaneous syndrome.

1. A 3-year-old boy with seizures and Lisch nodules on ophthalmologic examination.
2. A 12-month-old infant with infantile spasms.

Answers and Explanations
1. The answer is C [V.C.7.c–d]. This patient’s clinical presentation is most consistent with neonatal herpes simplex virus (HSV) infection. Diagnosis is by identification of (1) the virus by culture or (2) the viral antigen by rapid testing techniques. Infection may also be diagnosed by identification of epidermal giant cells on a Tzanck preparation. Two-thirds of HSV infections acquired during the neonatal period are caused by HSV-2, and this infection is most often acquired during passage through the birth canal of a mother infected with the virus. Neonatal HSV infection is a medical emergency that requires prompt admission and management with intravenous acyclovir. Oral antibiotics are not useful.
2. The answer is B [VII.A]. This patient’s presentation with well-demarcated hair loss with-out scalp inflammation is likely caused by alopecia areata. The cause of alopecia areata is thought to be an autoimmune lymphocyte-mediated injury to the hair follicle. Associated findings include nail pitting in 40% of patients. The clinical presentation is not consistent with tinea capitis, which would present with localized areas of hair loss associated with scalp inflammation, scaling, and broken hairs. Thus, griseofulvin is not indicated in this case. This presentation is also not consistent with traction alopecia due to hair care or styling practices, which is characterized by patchy and jagged, irregularly shaped areas of alopecia, especially along the hair line, containing small, thin, and broken hairs. In the case of alopecia areata, regrowth of hair within months to years occurs in the majority of patients.
3. The answer is A [V.A.1]. Clinical features of alopecia with hairs broken off at the scalp and occipital lymphadenopathy suggest tinea capitis, a fungal infection of the hair. The most common causal pathogen is Trichophyton tonsurans, acquired from human-to-human contact, including by sharing hats, brushes, and combs. Topical antifungal therapy is not effective for tinea capitis. Dogs and cats are a source of infection with Microsporum canis, which currently causes only 5% of tinea capitis infections. Infection with M. canis is characterized by thickened, white broken hairs, and only hairs infected with M. canis fluoresce under Woods light, whereas hairs infected with Trichophyton tonsurans do not fluoresce. Treatment of tinea capitis involves 6–8 weeks of oral antifungal therapy.
4. The answer is C [III.E.3]. Skin lesions with a silvery scale appearance suggest psoriasis. Lesions of psoriasis may also demonstrate the Koebner phenomenon, in which new lesions develop at sites of skin injury. Corticosteroids of moderate to high potency are the most effective treatment agents. Psoriasis is inherited in an autosomal dominant fashion, and therefore, other family members may also have the disorder. Nail involvement is often seen in individuals with psoriasis. Psoriatic arthritis is uncommon during childhood.
5. The answer is C [III.D.3–4]. This patient’s clinical presentation and examination findings are consistent with pityriasis rosea. Pityriasis rosea may be an immune response to a viral infection, although its true cause is unknown. Pityriasis rosea resolves without medications, although antihistamines and topical corticosteroids may relieve any associated itching. Exposure to ultraviolet light and short courses of oral antibiotics (for anti-inflammatory properties) may shorten the disease course. Clotrimazole is not an appropriate therapy for pityriasis rosea.
6. The answer is C [I.D.6.a]. The hypopigmentation in the inguinal region is most likely a side effect of the high-potency corticosteroid therapy. Only low-potency corticosteroids should be applied to the groin because the epidermis of the groin is very thin and absorption of corticosteroids is higher in this area. Local side effects of high-potency topical steroids include pigmentation changes, skin atrophy, acne, folliculitis, and telangiectasias. Neither antifungal medications nor ultraviolet light is indicated. Discontinuation of zinc oxide will not affect this patient’s skin findings. Although tuberous sclerosis is associated with

hypopigmented skin lesions, the history in this case is more suggestive of localized topical steroid use as the cause of the hypopigmentation. In the case of suspected tuberous sclerosis, the individual should have more than one hypopigmented skin lesion, a history of seizures or infantile spasms, and/or a family history of the disorder.
7. The answer is C [I.D.2]. Adequate hydration of the skin is crucial in many dermatologic conditions, especially atopic dermatitis. Ointments are especially useful for very dry skin because they have maximal water-retaining properties and would be the best choice for this patient. Lotions are helpful for minimal dryness, and creams are useful for average dryness. Both solutions and gels are most useful for hair-bearing surfaces.
8. The answer is A [VIII.C]. This patient’s skin findings are consistent with acne. The open and closed comedones are characteristic of noninflammatory acne, and a topical agent is most appropriate. Benzoyl peroxide is very effective for both noninflammatory and inflammatory acne and is an appropriate initial treatment for this patient. Neither oral antibiotics nor isotretinoin is a first-line treatment for noninflammatory acne. Oral antibiotics are most useful in inflammatory acne. Systemic isotretinoin is helpful in all types of acne; however, it is indicated for severe cystic or nodular acne. Obstruction to sebum outflow from the follicle leads to comedone development and therefore is important in the pathophysiology. Acne should never be dismissed as of no consequence, even in the early stages, because it may be associated with future scarring and may have effects on a child’s psychosocial development.
9. The answers are E, B, A, H, and A, respectively [V.C.7.b, VI.A.3, V.C.10.b.(1), IV.C.2, V.C.5, and Table 19-1]. Herpes simplex virus is characterized by grouped vesicles on an erythematous base. Vitiligo is a depigmentation disorder characterized by hypopigmented macules. Molluscum contagiosum is characterized by flesh-colored papules with central umbilication. Erythema multiforme major is characterized by target lesions. Erythematous or flesh-colored papules are characteristic of Gianotti–Crosti syndrome.
10. The answers are A and C, respectively [Table 19-3]. The 3-year-old boy has neurofibromatosis type 1, which is characterized by café-au-lait spots, axillary freckling, and neurofibromas in the skin and other organs. Seizures may occur because of central nervous system involvement that may include intracranial calcifications, neurofibromas, and optic glioma. Characteristic eye findings include Lisch nodules or iris hamartomas. The 12-month-old infant has tuberous sclerosis, which is associated with characteristic skin findings including ash-leaf spots, angiofibromas, and shagreen patch. Almost all children with tuberous sclerosis have seizures because of central nervous system involvement, and the condition is frequently associated with infantile spasms.

C H A P T E R 2 0

Emergency Medicine

Calvin G. Lowe

I. Infant and Child Cardiopulmonary Resuscitation (CPR)
A. General concepts
1. The most common cause of cardiac arrest in a child is a lack of oxygen supply to the heart secondary to a pulmonary problem (e.g., choking, suffocation, airway or lung disease, drowning), respiratory arrest, or shock. Cardiac arrest can often be prevented if assisted breathing is initiated promptly.
2. Heart disease is an uncommon cause of cardiac arrest in infants and children.
3. Chances for survival increase dramatically if CPR and advanced life support are begun quickly.
4. If a patient is found unresponsive and not breathing, then CPR should be initiated.
B. Sequence of CPR is C–A–B: Circulation–Airway–Breathing. In 2010 (and subsequently – reaffirmed), the American Heart Association recommended that CPR for infants and children be initiated with chest compressions rather than rescue breaths (C–A–B rather than A–B–C). When approaching an unresponsive victim, the rescuer should shout for nearby help and activate the emergency response system. The rescuer should look for breathing or gasping and simultaneously check for a pulse.
1. Circulation
a. Chest compressions are administered for asystole or bradycardia.
b. Rescuers should compress the chest at least one-third of the anterior–posterior diameter of the chest.
1. 1½ inches in most infants
2. 2 inches (5 cm) in most children
c. Chest compressions are delivered at a rate of 100/minute.
2. Airway
a. After the first set of compressions are given, the vicitim’s airway is opened.
b. The victim’s tongue is the most common cause of airway obstruction.
c. The airway may be opened by the head tilt method, which lifts the tongue from the back of the throat, or by the jaw-thrust method if the child has suspected neck or cervical spine injury. (Cervical spine injury should be suspected in face, head, or neck trauma.)
3. Breathing
a. After the airway is opened, assessment for breathing is performed by the look, listen, and feel method, in which the rescuer looks for a rise and fall in the chest, listens for exhaled air, and feels for exhaled airflow.
b. Rescue breathing must be performed if spontaneous breathing is absent.
C. CPR cycles
1. Ratio of chest compressions for one-rescuer CPR is 30 compressions:2 breaths.
2. Ratio of compressions of two-rescuer CPR is 15 compressions:2 breaths.
3. After 2 minutes of compressions/breaths or five cycles, a reassessment of the patient for a pulse or spontaneous respirations should be performed.

II. Shock
A. General concepts
1. Definition. Shock is a clinical state characterized by inadequate delivery of oxygen and metabolic substrates to meet the metabolic demands of tissues.
2. Shock may be present with normal or decreased blood pressure.
B. Classification. Shock may be classified by the degree of compensation and by the cause.
1. Shock classification by the degree of compensation includes compensated, decompensated, or irreversible.
a. Compensated shock is characterized by normal blood pressure and cardiac output with adequate tissue perfusion but maldistributed blood flow to essential organs.
b. Decompensated shock is characterized by hypotension, low cardiac output, and
inadequate tissue perfusion.
c. Irreversible shock is characterized by cell death and is refractory to medical treatment.
2. Shock may also be classified on the basis of the cause.
a. Hypovolemic shock is the most common cause of shock in children and is caused by any condition that results in decreased circulating blood volume, such as hemorrhage or dehydration (e.g., from acute gastroenteritis). The amount of volume loss determines the success of compensatory mechanisms, such as endogenous catecholamines, in maintaining blood pressure and cardiac output. Volume losses greater than 25% result in decompensated shock.
b. Septic shock occurs secondary to an inflammatory response to invading microorganisms and their toxins, and it results in abnormal blood distribution. There are two clinical stages:
1. Hyperdynamic stage is characterized by normal or high cardiac output with
bounding pulses, warm extremities, and a wide pulse pressure.
2. Decompensated stage follows the hyperdynamic stage if aggressive treatment has not been initiated. It is characterized clinically by impaired mental status, cool extremities, and diminished pulses.
c. Distributive shock is associated with distal pooling of blood or fluid extravasation, and it is typically caused by anaphylactic or neurogenic shock or as a result of medications or toxins.
1. Anaphylactic shock is characterized by acute angioedema of the upper airway, bronchospasm, pulmonary edema, urticaria, and hypotension because of extravasation of intravascular fluid from permeable capillaries (see Chapter 15, section I).
2. Neurogenic shock, typically secondary to spinal cord transection or injury, is characterized by a total loss of distal sympathetic cardiovascular tone, with hypotension resulting from pooling of blood within the vascular bed.
d. Cardiogenic shock occurs when cardiac output is limited because of primary cardiac dysfunction. Causes include dysrhythmias (e.g., supraventricular tachycardia), congenital heart disease (e.g., any lesion that impairs left ventricular outflow), and cardiac dysfunction after cardiac surgery. Clinical features are the signs and symptoms of congestive heart failure (CHF; see Chapter 8, section I.D).
C. Diagnosis
1. Recognition of shock may be difficult because of the presence of compensatory mechanisms that prevent hypotension until 25% of intravascular volume is lost. Therefore, the index of suspicion for shock must be high.

2. Historic features that may suggest the presence of shock include the following:
a. Severe vomiting and diarrhea
b. Trauma with hemorrhage
c. Febrile illness, especially in an immunocompromised patient
d. Exposure to a known allergic antigen
e. Spinal cord injury
3. Physical examination
a. Blood pressure may be normal in the initial stages of hypovolemic and septic shock.
b. Tachycardia almost always accompanies shock and occurs before blood pressure changes in children.
c. Tachypnea may be present as a compensatory mechanism for severe metabolic acidosis.
d. Mental status changes may indicate poor cerebral perfusion.
e. Capillary refill may be prolonged with cool and mottled extremities.
f. Peripheral pulses may be bounding in early septic shock.
4. Laboratory studies should include the following:
a. Complete blood count (CBC) to assess for blood loss and infection
b. Electrolytes to assess for metabolic acidosis and electrolyte abnormalities
c. Blood urea nitrogen and creatinine to evaluate renal perfusion and function
d. Lactate to assess adequacy of tissue perfusion
e. Calcium and glucose to assess for frequently encountered metabolic derangements
f. Coagulation factors to evaluate for disseminated intravascular coagulation (DIC), which may accompany shock
g. Toxicology screens to evaluate for a poisoning, which could cause shock
h. Blood and urine cultures to evaluate for infections that could cause shock
D. Management
1. Initial resuscitation includes:
a. Supplemental oxygen
b. Early endotracheal intubation to secure the airway and decrease the patient’s energy expenditure
c. Vascular access with appropriate fluid resuscitation. Fluids should initially include a 20 mL/kg bolus of normal saline or lactated Ringer solution.
2. To restore intravascular volume, intravenous crystalloid or colloid solutions should generally be used before administration of inotropic and vasopressor agents.
3. Inotropic and vasopressor medications (e.g., dobutamine, dopamine, epinephrine) are indicated if the blood pressure increase in response to fluids is inadequate.
4. Metabolic derangements, such as metabolic acidosis, hypocalcemia, or hypoglycemia, should be treated.
5. Other considerations include administration of broad-spectrum antibiotics for septic shock, blood products for hemorrhage, and fresh-frozen plasma for DIC.

III. Trauma
A. General concepts
1. Trauma is the leading cause of death in the United States in children older than 1 year.
2. Motor vehicle accidents are the leading cause of trauma.
3. Anatomic and physiologic differences between a child and an adult account for the child’s unique response to trauma.
a. Head injuries are common because a child’s head comprises a larger percentage of total body mass.
b. The neck of a child is shorter and supports a relatively greater weight.
c. The rib cage of a child is more pliable, leading to greater energy transmitted to internal organs, such as the spleen and liver.
d. The growth plates in the bones of a growing child result in a relatively weak epiphyseal–metaphyseal junction. Ligaments are stronger than the growth plate; therefore, with injury, the growth plate is at the highest risk for fracture.
B. Primary survey. This rapid initial assessment of the patient should be performed within 5– 10 minutes of arrival in the emergency department. The primary survey can be recalled using the mnemonic ABCDE: Airway maintenance, Breathing and ventilation with 100% oxygen,
Circulation and control of hemorrhage, Disability assessment using the Glasgow coma score (GCS, which is used to assess the extent of neurologic impairment based on physical examination (Table 20-1), and Exposure/Environmental control, in which the patient is undressed completely to facilitate examination and then warmed to prevent hypothermia.
C. Adjuncts to the primary survey
1. Electrocardiographic (ECG) monitoring is mandatory for all patients.
a. Dysrhythmias may indicate cardiac injury.
b. Pulseless electrical activity may indicate cardiac tamponade, tension pneumothorax, or profound hypovolemia.
2. A urinary catheter should be placed to monitor urine output, and a nasogastric tube to reduce abdominal distension.
3. Diagnostic studies typically include radiographs of the cervical spine, chest, and pelvis, and computed tomographic (CT) scans of the head and abdomen.
D. Secondary survey. This head-to-toe evaluation includes a thorough physical examination.
E. Specific injuries in the pediatric trauma patient
1. Head trauma
a. Seizures are common after head trauma but are self-limited.
b. Infants are at risk for bleeding in the subgaleal and epidural spaces because of open fontanelles and cranial sutures. However, these open structures may also allow infants to be more tolerant of expanding intracranial masses.
c. Intracranial bleeding may occur in the epidural space, subdural space, or within the brain parenchyma itself after even mild head trauma without skull fracture or loss of consciousness.
1. Epidural hematoma is bleeding between the inner table of the skull and the dura. It is associated with tearing of the middle meningeal artery typically following a high-risk injury (direct blow to the side or back of the head, or fall from a significant height).
a. Clinical features are the signs and symptoms of increased intracranial pressure (ICP; see Table 20-2 and section III.E.1.d). A classic history could include an initial loss of consciousness, a lucid interval for several hours, followed by abrupt decompensation and coma.

b. Diagnosis is by head CT, which shows a lenticular density representing blood within the epidural space.
c. Management is immediate surgical drainage.
d. Prognosis is generally good if surgery can be performed rapidly.
2. Subdural hematoma is blood beneath the dura. It is associated with tearing of the bridging meningeal veins by direct trauma or shaking. It is more common than epidural hematoma and is seen most commonly in infancy.
a. Clinical features include seizures and signs and symptoms of increased ICP (see Table 20-2 and section III.E.1.d). Subdural hematomas are bilateral in 75% of cases, and symptoms develop more slowly than with an epidural bleed.
b. Diagnosis is by head CT, which shows a crescentic density representing blood in the subdural space.
c. Management includes neurosurgical consultation and usually surgical drainage.
d. Prognosis is variable and may be poor if the underlying brain is also injured.
3. Intracerebral hematoma is bleeding within the brain parenchyma. Frontal and temporal lobes are most often affected, usually on the opposite side of the impact injury (contrecoup injury). Management is surgical drainage if the hematoma is accessible.
d. Increased ICP
1. Clinical features
a. Headache is the first symptom.
b. Pupillary changes and altered mental status are the first signs.
c. Table 20-2 lists the clinical features of increased ICP.
2. Complications. Increased ICP may lead to cerebral herniation, most commonly transtentorial or uncal herniation, in which the temporal lobe or uncus is displaced into the infratentorial compartment. Clinical features of herniation include the following:
a. Bradycardia, which is an early sign of herniation in children younger than 4 years
b. Fixed and dilated ipsilateral pupil
c. Contralateral hemiparesis
d. Pupils will eventually become bilaterally fixed and dilated.
e. Bilateral hemiparesis will also eventually occur.
f. Cushing triad, a late sign, is characterized by bradycardia, hypertension, and irregular breathing.
3. Management. The goal of treatment is to prevent secondary brain injury and includes the following:
a. Mild hyperventilation with 100% oxygen to lower PaCO2 to 30– 35 mm Hg, which in turn mildly vasoconstricts cerebral vessels.
Aggressive hyperventilation can lead to worsening cerebral ischemia.
b. Elevation of the head to 30°–45°, which encourages venous drainage
c. Diuretics (e.g., mannitol, 3% hypertonic normal saline)
d. Neurosurgical consultation
2. Spinal cord injury. Injury to the spinal cord may occur in children. Even in the presence of serious injury, radiographs of the cord may be normal. Note that spinal cord injury without radiographic abnormality (“SCIWORA”) occurs more commonly in children than in adults.

3. Chest trauma
a. A child’s soft and pliable chest wall allows transmission of forces to the lung parenchyma.
b. Tension pneumothorax may occur and is life-threatening.
1. Clinical features include distended neck veins, decreased breath sounds, hyperresonance to percussion, displaced trachea, pulseless electrical activity, and shock.
2. Management is emergent chest decompression by needle thoracotomy. Waiting for radiographic confirmation of the diagnosis can lead to a patient’s death.
4. Abdominal trauma is common because of underdeveloped abdominal musculature.
a. Duodenal hematoma often occurs secondary to injury to the right upper quadrant, commonly from a bicycle handle bar. Clinical features include abdominal pain and vomiting. Bowel obstruction is found on radiographic evaluation.
b. Lap belt injuries from a motor vehicle accident include a chance fracture (flexion disruption of the lumbar spine), liver and spleen lacerations, and bowel perforation.
c. Spleen, liver, and kidneys are often injured by blunt trauma.

TABLE 20-1
Glasgow Coma Scale (GCS)*

Verbal Patient GCS Score Nonverbal Patient (Child)
Eye opening
Spontaneously 4 Spontaneously
Response to voice 3 Response to voice
Response to pain 2 Response to pain
No response 1 No response
Best motor response
Obeys commands 6 Normal movements
Localizes pain 5 Localizes pain
Flexion withdrawal 4 Flexion withdrawal
Decorticate posturing 3 Flexion abnormal
Decerebrate posturing 2 Extension abnormal
No response 1 No response
Best verbal response
Oriented/appropriate 5 Cries normally, smiles, and coos
Disoriented conversation 4 Cries
Inappropriate words 3 Inappropriate crying and screaming
Incomprehensible words 2 Grunts
No response 1 No response
*This scale is used to assess the level of neurologic impairment on the basis of a patient’s physical examination (eye opening, motor response, and verbal response). A GCS of 13–15 indicates mild head injury; a GCS of 9–12 indicates moderate head injury; and a GCS < 8 indicates severe head injury.

Table 20-2
Symptoms and Signs of Elevated Intracranial Pressure

Symptom Sign
Headache Papilledema
Vomiting Cranial nerve palsies
Stiff neck Stiff neck
Double vision Head tilt
Transient loss of vision Retinal hemorrhage
Episodic severe headache Percussion of skull—“cracked pot sound”
Gait disturbance Obtundation
Dulled intellect Unconsciousness
Irritability Progressive hemiparesis

IV. Burns
A. Epidemiology. More than 40% of deaths from burns occur in children younger than 4 years.
1. Scalding injuries from hot liquids are the most common type of burn.
2. Not all burns are accidental, and child abuse should be considered (see section VI.B.2.c).
B. Classification of burns is on the basis of degree (depth of skin injured) and body surface area
(BSA).
1. First-degree burns involve only the epidermis and are characterized by red, blanching, painful skin that heals without scarring (e.g., sunburn).
2. Second-degree burns (partial-thickness burns) involve the entire epidermis and part of the dermis.
a. Superficial partial-thickness burns involve the entire epidermis and outer portion of the dermis. Burns are moist, painful, and red. They blister but usually do not scar.
b. Deep partial-thickness burns involve destruction of the entire epidermis and lower portion of the dermis. Burns are pale white. They may blister and they heal with scarring.
3. Third-degree burns or full-thickness burns involve the complete destruction of the epidermis, dermis, and part of the subcutaneous tissue. Burns are dry, white, and leathery to the touch, and skin grafts are needed. Because nerve endings are burned, the victim is usually insensitive to pain.
4. BSA burned is expressed in percentage. The Lund–Browder classification may be used to help measure the burned area. Although the “rule of 9’s” is used to measure percent BSA burned in adolescents and adults (each arm = 9%, each leg = 18%, anterior trunk = 18%, posterior trunk = 18%, head and neck = 9%), this rule overestimates burns in a child because of a child’s relatively larger head and smaller legs. Another estimate of percent BSA burned used in children uses the size of the patient’s palm to measure the burned area; the palm is approximately equivalent to 1% BSA. First-degree burns are not included in the calculation of the BSA burned.
C. Management
1. Initial resuscitation should include the ABCs.
a. Endotracheal intubation should be performed in any victim suspected of inhaling hot gases, which may burn the upper airway and lead to progressive edema and airway obstruction.
b. Assess oxygenation by pulse oximetry. Administer 100% oxygen and assess for carbon monoxide inhalation (see section VIII.D.6).
c. Intravenous access should be obtained through nonburned skin.
2. Fluid resuscitation is critical, because large volumes of fluid may be lost from burned skin and leaky capillaries.
a. Lactated Ringer solution is the isotonic crystalloid fluid of choice in burn resuscitation.
b. The Parkland formula calculates the volume of fluid replacement within the first 24 hours.
1. The replacement volume is approximately 4 mL/kg body weight per percent BSA burn.
2. Half of the replacement volume is given over the first 8 hours postburn, and the remaining half over the next 16 hours.
3. Skin care depends on the degree of burn.

a. First-degree burns require only moisturizers and analgesics.
b. Second-degree burns require appropriate analgesics (e.g., opiates) and debridement of dead skin to prevent infection. Bullae (large blisters), if intact, are generally not removed because the skin of the bullae forms a barrier to infection and prevents fluid loss. Bullae that have ruptured should be removed.
c. Third-degree burns require skin grafting and hydrotherapy. Escharotomy (i.e., surgical removal of a constricting scar) may be needed if the burn restricts blood flow or chest expansion.
d. Antibiotics, usually topical 1% silver sulfadiazine, are applied to second- and third- degree burns to decrease the risk of infection.
4. Hospitalization is required for partial-thickness burns >10% BSA, full-thickness burns
>2% BSA, burns to specific areas of the body (e.g., face, perineum, hands, feet, burns overlying a joint, or circumferential burns), suspected inhalation injury, and suspected nonaccidental trauma (i.e., inflicted burn).

V. Drowning
A. Definition. Drowning is the process of experiencing respiratory impairment from submersion/immersion of liquid.
1. Nonfatal drowning refers to a person who is rescued at any time, and the process of drowning is interrupted.
2. Fatal drowning refers to a person who dies any time as a result of drowning.
3. Terms such as “near drowning,” “dry or wet drowning,” “secondary drowning,” or “active and passive drowning” are no longer used.
B. Epidemiology. Submersion-related injuries are the leading cause of death among children ages 1– 4 years in the United States. There is a bimodal age distribution in childhood.
1. Older infants and toddlers, who may wander into unfenced pools or tip over into water containers (e.g., toilets and buckets)
2. Adolescents, most commonly males, whose submersion injury is typically associated with alcohol or drug ingestion
C. Pathophysiology
1. Victims may suffer asphyxia from aspirating liquid or from laryngospasm.
2. Both fresh and salt water drowning result in denaturing of surfactant, alveolar instability and collapse, and pulmonary edema.
3. The end result is decreased pulmonary compliance, increased airway resistance, increased pulmonary artery pressures, and impaired gas exchange.
D. Clinical features
1. Respirations may be absent or irregular, and the victim may cough up pink, frothy material.
a. Physical examination may reveal rales, rhonchi, and wheezes.
b. Pneumonia from aspiration of fluid containing mouth flora may develop after 24 hours.
c. Slow deterioration of pulmonary function (e.g., hypoxemia and hypercarbia) may occur during the first 12–24 hours.
2. Neurologic insult (hypoxic central nervous system [CNS] injury) is directly related to the length and severity of the hypoxia. The victim may appear alert initially or may be agitated, combative, or comatose.
3. Cardiovascular abnormalities include dysrhythmias and myocardial ischemia.
4. Hematologic abnormalities include hemolysis and DIC.
5. Renal failure may occur.
E. Management. Treatment is the same regardless of whether the drowning occurred in salt or fresh water.
1. Initial resuscitation includes the ABCs, cervical spine immobilization (because of the possibility of coexistent head trauma), and removal of wet clothing to reduce heat loss.
2. Intubation and mechanical ventilation with high positive end-expiratory pressures (PEEPs) are indicated for patients with respiratory failure.
3. Rewarming of body core with warm saline gastric lavage, bladder washings, or peritoneal lavage should be performed if needed. In severely hypothermic patients, resuscitation should continue until the patient is rewarmed to 32°C (89.6°F).
4. Attention should be paid to fluid and electrolyte imbalance.
F. Prognosis. In general, children have a better outcome from drowning because their primitive dive reflex shunts blood to vital organs, such as the heart, brain, and liver. However, prognosis is poor for the following victims:
1. Children younger than 3 years

2. Submersion time > 5 minutes
3. Resuscitation delay > 10 minutes
4. CPR required
5. Abnormal neurologic examination or seizures
6. Arterial blood pH < 7

VI. Child Abuse
A. General concepts
1. In most states, health care personnel have a legal obligation to report suspected child abuse or neglect to appropriate protective service or law enforcement agencies.
2. The index for suspicion of abuse should be high, especially in situations in which injuries found on examination are unaccounted for or are inconsistent with the caregiver’s history or the child’s developmental abilities.
3. Child abuse includes physical abuse, psychological abuse, neglect, and sexual assault.
B. Physical abuse
1. Epidemiology
a. Any child is at risk for abuse. The risk of abuse is greatest, however, in children with the following characteristics:
1. Age younger than 4 years, especially younger than 1 year
2. Mental retardation, developmental delay, severe handicaps, hyperactivity, or challenging temperament (including colic or frequent tantrums)
3. History of premature birth, low birth weight, neonatal separation from parents, or multiple births
4. Chronic illness
b. Child abusers come from all socioeconomic, cultural, and ethnic groups. Risk factors for an abusive caregiver include the following:
1. Low self-esteem, social isolation, depression, or history of substance abuse
2. History of abuse as a child
3. History of mental illness
4. History of violent temperament
5. Family dynamics that include single parenthood, unemployment, poverty, marital conflicts, domestic violence, poor parent–child relationships, and unrealistic expectations of the child
2. Clinical features
a. Bruises
1. Bruises on fleshy or protected areas, such as the face, neck, back, chest, abdomen, buttocks, and genitalia, are often consistent with inflicted injury. In contrast, bruises on exposed areas, such as the shins, knees, elbows, and forehead, are typically from noninflicted trauma.
2. Patterns of bruising may help determine the type of object used to inflict the trauma (e.g., distinctive marks are left by belt loops, buckles, hangers, and hands).
b. Human bites may be found anywhere on the body, including the genitalia and buttocks of infants.
c. Burns often have distinguishable patterns.
1. Accidental burns have an irregular, splashlike configuration. In contrast, nonaccidental burns typically have a clear line of demarcation (e.g., “stocking” or “glovelike” pattern, suggesting submersion injury).
2. Objects used to burn may be branded to the skin (e.g., irons and cigarettes).
d. Fractures that are inconsistent with the history or with the child’s developmental ability may be secondary to abuse (see Chapter 17, section V.G). The following fractures are considered highly suggestive of abuse:
1. Metaphyseal fractures (“bucket handle” or corner fractures), which are caused by torsional force on the limb (i.e., pulling and twisting) or by violent

shaking
2. Fractures of the posterior or first ribs, sternum, scapula, and vertebral spinous process
3. Multiple fractures in different stages of healing
e. Head injuries, caused by trauma, asphyxiation, or shaking, are the leading cause of death and morbidity from child abuse. Shaken baby syndrome may occur in a child younger than 2 years of age who is violently shaken. (This syndrome is termed “shaken impact syndrome” if the child is thrown after the shaking.) Retinal hemorrhages, subdural hematomas, metaphyseal fractures, and significant brain injury are characteristic.
f. Visceral injuries are the second leading cause of death from child abuse and include rupture and injury of the intestinal tract, liver, and spleen.
3. Diagnosis
a. History is critical in differentiating inflicted from noninflicted trauma. Child development should correlate with the nature of the injury. Delays in seeking medical attention, implausible histories, and histories that change or are inconsistent among caregivers are suspicious for abuse.
b. Physical examination should focus both on acute injuries and on identifying old lesions that may be secondary to abuse. If shaken baby syndrome is suspected, a dilated ophthalmoscopic evaluation for retinal hemorrhages should be performed (see Chapter 18, section V.A).
c. Accessory tests should include a skeletal survey to evaluate for old or healing fractures and head and abdominal CT scans to evaluate for acute injuries.
4. Management. Child protective services or law enforcement agencies must be notified if there is a suspicion of abuse. Hospitalization may be required if medically indicated or until a safe location for the child has been identified.
C. Sexual abuse
1. General concepts
a. Unlike physical abuse, there are typically no overt physical signs of trauma.
b. Perpetrators are often known to the child before the abuse.
2. Epidemiology. Eighty percent of sexual abuse occurs in females.
3. Diagnosis
a. History is critical to confirm abuse.
1. Obtaining a history of abuse from a young child is difficult. Ideally, the history should be obtained with open-ended questions from an interviewer trained in sexual abuse evaluation.
2. Sexually abused children typically present with multiple nonspecific complaints, including abdominal and urogenital symptoms.
3. Sexual behavior in young children raises red flags for abuse.
b. Physical examination should be performed after rapport with the patient has been established.
1. Signs of trauma should be noted.
2. Genital and perianal examination should be performed last and should include inspection of the hymen, vagina, and perianal areas (penis, scrotum, and perianal areas in males), with notation of any discharge, injury, or bleeding.
3. Physical examination is normal in most victims.
c. Laboratory studies to collect forensic evidence should be performed if the abuse occurred within 72 hours of presentation, and should include cultures or serologic testing for sexually transmitted infections (STIs), including human

immunodeficiency virus (HIV). If appropriate, testing for pregnancy and assessment of vaginal fluid for spermatozoa should also be performed.
d. Management
1. Safety of the child should be the highest priority in determining placement.
2. Child protective services or social services must be notified and should arrange follow-up and support.
3. Pregnancy may be prevented with high-dose oral contraceptives (morning- after pills).
4. Antibiotics are often prescribed to empirically treat STIs.

VII. Sudden Infant Death Syndrome (SIDS)
A. Definition. SIDS is the death of an infant younger than 1 year whose death remains unexplained after a thorough case investigation that includes autopsy, death scene evaluation, and review of the clinical history.
B. Epidemiology
1. SIDS is the most common cause of death in children younger than 1 year.
2. Incidence is approximately 1 in 2500 live births. Peak incidence is at 2–4 months of age.
3. The typical victim is found dead in the morning in bed after being put to sleep at night.
4. Risk factors associated with SIDS may be found in Chapter 9, section IV.E.2.b.
C. Management
1. Resuscitation should be attempted on all patients because of the difficulty in ascertaining the period of time the infant has been apneic and pulseless.
2. If resuscitation is unsuccessful, the child’s body is referred to the medical examiner for autopsy. Postmortem examination may demonstrate intrathoracic petechiae (the most common autopsy finding in 80% of cases, but one whose cause is unknown), pulmonary congestion or edema, small airway inflammation, and evidence of hypoxia.

VIII. Poisonings
A. General concepts
1. Epidemiology
a. Sixty percent of all poisonings occur in children younger than 6 years.
b. Ninety percent of poisonings are accidental. The majority of poisonings occur at
home when the child’s caregiver is distracted.
c. Most poisons are ingested, although poisons may also be inhaled, spilled on the skin or into the eyes, or injected intravenously.
d. Mortality is < 1%.
2. Etiology. The most common toxic exposures involve commonly used household products.
a. Cosmetics and personal care products (most common toxic exposure)
b. Cleaning agents
c. Cough and cold preparations
d. Vitamins, including iron
e. Analgesics (e.g., acetaminophen, nonsteroidal anti-inflammatory drugs, aspirin)
f. Plants (6–7% of all ingestions)
g. Alcohols (e.g., ethanol) and hydrocarbons (e.g., gasoline, paint thinner, furniture polish)
h. Carbon monoxide (see section VIII.D.6)
i. Prescription medications
B. Evaluation
1. Consider poisoning in patients presenting with nonspecific signs and symptoms, such as seizures, severe vomiting and diarrhea, dysrhythmias, altered mental status or abnormal behaviors, shock, trauma, or unexplained metabolic acidosis.
2. History obtained from caregivers typically identifies the poison.
a. Information about the toxin should include the type or name of toxin, toxin concentration (if known), and the route of exposure.
1. Potential poison dose is calculated for the worst-case scenario. Toxicity is typically on the basis of the amount ingested per kilogram of body weight.
2. Consider multiple agents in adolescents.
b. Information about the environment should include location of victim when discovered, and medications, plants, vitamins, herbs, and chemicals in the home. Time of occurrence, if known, is very important.
3. Physical examination should be comprehensive and may provide additional clues to the identity of the toxin. Figure 20-1 shows the link between some typical physical findings and associated toxins.
4. Laboratory studies
a. Screening laboratory tests include serum glucose, serum and urine toxicology screens, and electrolytes.
1. Anion gap [Na+ − (Cl− + HCO3−)] should be calculated.
2. Causes of an increased anion gap (>16) may be recalled using the mnemonic AMUDPILES (alcohol, methanol, uremia, diabetic ketoacidosis, paraldehyde, iron and isoniazid, lactic acidosis, ethylene glycol, salicylates).
b. Radiographic imaging of the abdomen may reveal radiopaque substances. These may be recalled using the mnemonic CHIPE (chloral hydrate and calcium, heavy metals, iodine and iron, phenothiazines, enteric-coated tablets).
C. General management principles

1. The ABCs are the initial management priority.
2. If the patient has altered mental status, administer dextrose for hypoglycemia and
naloxone for possible opiate overdose.
3. A poison control center may be consulted to assist with management.
4. Gastric decontamination
a. Activated charcoal has a very large adsorptive surface area that binds toxins and minimizes their absorption.
1. Activated charcoal should be considered for all poisonings. However, it is ineffective for some poisons such as iron, lithium, alcohols, ethylene glycol, iodine, potassium, and arsenic. In addition, activated charcoal interferes with visualization during endoscopy and therefore should not be used for caustic ingestions.
2. Evidence suggests that activated charcoal improves clinical outcomes,
especially if given within 1 hour after an ingestion.
b. Whole-bowel irrigation (WBI) is rapid, complete emptying of the intestinal tract accomplished using polyethylene glycol (an osmotic agent) and an electrolyte solution (to prevent electrolyte imbalance). Preliminary studies show that WBI may be effective for ingestions of iron and other heavy metals and sustained-release medications.
c. Antidotes exist for a relatively small number of compounds (Table 20-3).
D. Specific poisonings
1. Acetaminophen. This drug is one of the most common medications ingested by children and adolescents. Doses >150 mg/kg are associated with toxicity.
a. Pathophysiology
1. Hepatic damage, the major sequelae of toxicity, is directly related to the depletion of glutathione, a cofactor used during the metabolism of acetaminophen by the cytochrome P-450 system.
2. Toxic intermediates produced when glutathione is depleted bind directly to hepatocytes, causing hepatocellular necrosis.
b. Clinical features. There are four stages of acetaminophen poisoning (Table 20-4).
c. Management
1. Activated charcoal
2. Obtain serum acetaminophen level 2–4 hours after ingestion. The level should be plotted on the Matthew–Rumack nomogram to determine the potential for hepatitis.
3. If the nomogram predicts hepatitis, the antidote, N-acetylcysteine (NAC), a glutathione precursor, is indicated.
a. NAC is given orally as a 140 mg/kg loading dose and is followed with 70 mg/kg every 4 hours for 17 doses.
b. NAC may be given intravenously as an infusion for more than 21 hours.
c. NAC is hepatoprotective if given within 8 hours of ingestion. It may still be helpful up to 72 hours after ingestion.
2. Salicylates. Salicylate poisoning has decreased as acetaminophen’s usage has increased; however, salicylates remain as an ingredient in many over-the-counter compounds, such as Pepto-Bismol, Ben-Gay, and oil of wintergreen. Doses >150 mg/kg are associated with toxicity.
a. Pathophysiology
1. Salicylates directly stimulate respiratory centers. This causes hyperventilation that may overcompensate for metabolic acidosis produced by

the salicylate (it is a weak acid), resulting in a respiratory alkalosis.
2. Salicylates uncouple oxidative phosphorylation, producing lactic acidosis
and enhancing ketosis.
b. Clinical features. Common signs and symptoms include fever, diaphoresis, and flushed appearance; tinnitus; vomiting; headache; lethargy, restlessness, coma, and seizures; hyperpnea; and dehydration.
c. Laboratory findings
1. Respiratory alkalosis with an anion-gap metabolic acidosis is the most common acid–base disturbance.
2. Hyperglycemia, followed later by hypoglycemia
3. Hypokalemia
d. Management
1. Activated charcoal is effective and may be readministered every 4 hours in severe poisonings.
2. Obtain serum salicylate level at least 6 hours after ingestion. The level should then be plotted on the Done nomogram to assess for potential toxicity.
3. Alkalinization of urine with sodium bicarbonate to a urine pH > 7 and large- volume intravenous fluids enhance renal excretion of salicylates.
4. Dialysis may be required for life-threatening ingestions.
3. Iron
a. Epidemiology
1. Iron is one of the most common and potentially fatal childhood poisonings. As little as 20 mg/kg of iron is toxic.
2. Adult-strength ferrous sulfate tablets and iron in prenatal vitamins are the most common sources of accidental iron ingestion.
b. Pathophysiology
1. Direct damage to the gastrointestinal tract leading to hemorrhage
2. Hepatic injury and necrosis
3. Third spacing and pooling of blood in the vasculature leading to hypotension
4. Interference with oxidative phosphorylation
c. Clinical features. There are four stages of iron toxicity (Table 20-5).
d. Management
1. Activated charcoal does not bind to iron. However, if a polyingestion is suspected, activated charcoal should be given.
2. Hypovolemia, blood loss, and shock should be anticipated and treated.
3. WBI should be considered for life-threatening ingestion.
4. Serum iron level should be obtained 2–6 hours after ingestion.
5. Intravenous deferoxamine, an iron-binding ligand, should be given if any of the following condition exists:
a. If serum iron levels > 500 µg/dL, or if > 300 µg/dL and acidosis, hyperglycemia, or leukocytosis are present
b. If severe gastrointestinal symptoms are present
c. If more than 100 mg/kg of iron is ingested
6. Before the serum iron level is known, a test dose of deferoxamine may be administered. If the patient’s urine then turns red or pink (vin rose) (the color of chelated iron), the challenge is considered positive, indicating a clinically significant iron ingestion. Intravenous deferoxamine should then be continued.
4. Lead
a. Sources of lead include ingestion of lead-based paint chips, water carried by

outdated lead pipes, improperly glazed or foreign-made ceramic food or water containers, and pica (compulsive eating of nonnutrient substances such as dirt, paint, and clay). (See Chapter 1, section IV.J.)
b. Clinical features. Lead poisoning is typically a chronic ingestion; however, children may also present with acute lead intoxication.
1. Abdominal complaints include colicky pain, constipation, anorexia, and vomiting.
2. CNS complaints include listlessness, irritability, seizures, and decreased consciousness with encephalopathy.
3. Peripheral blood smear may show microcytic anemia, basophilic stippling,
and red blood cell precursors.
4. Radiopacities may be seen on abdominal radiographs, and dense metaphyseal bands may be seen on radiographs of the knees and wrists (lead lines).
c. Diagnosis. An elevated lead level or elevated erythrocyte protoporphyrin is the basis of diagnosis.
d. Management. Treatment for significant toxicity includes dimercaprol, British anti- lewisite (BAL), or calcium disodium ethylenediaminetetraacetic acid (EDTA).
5. Caustic agents. These are acids or alkalis with corrosive potential.
a. Pathophysiology
1. Acids (e.g., toilet bowl cleaner) cause coagulation necrosis that leads to superficial damage to the mouth, esophagus, and stomach. More severe injury results from compounds that have a pH <2.
2. Alkalis (e.g., oven and drain cleaners, bleach, laundry detergent) cause liquefaction necrosis that produces deep and penetrating damage, most commonly to the mouth and esophagus. More severe injury results from compounds that have a pH >12.
b. Clinical features
1. Immediate burning sensation with intense dysphagia, salivation, retrosternal chest pain, and vomiting
2. Obstructive airway edema (especially with acid ingestion)
3. Gastric perforation and peritonitis may follow acid ingestion.
4. Esophageal perforation with mediastinitis may follow alkali ingestion.
c. Management. Treatment initially includes the ABCs.
1. No attempt should be made to neutralize the caustic agent, because the combination of acid and alkali will generate an exothermic reaction and worsen any burn.
2. Activated charcoal is contraindicated because it interferes with endoscopy.
3. Endoscopy is performed to assess the degree of damage.
4. Household bleach has less corrosive potential and generally does not require treatment.
6. Carbon monoxide poisoning
a. Epidemiology. Carbon monoxide (CO) is a by-product of incomplete combustion of carbon-containing material. Excessive exposure may occur from fires, tobacco, faulty home heaters, car exhaust, and industrial pollution. CO is odorless, tasteless, and colorless.
b. Pathophysiology. CO interferes with oxygen delivery and utilization.
1. Carbon monoxide displaces oxygen from the hemoglobin molecule, forming carboxyhemoglobin (CO-Hb), which can no longer carry oxygen. The bond between CO and hemoglobin is more than 200 times stronger than the bond

between oxygen and hemoglobin.
2. The oxygen–hemoglobin dissociation curve is shifted to the left. This leads to tighter binding of the remaining oxygen bound to hemoglobin and impaired release of oxygen to tissues.
3. Carbon monoxide also interferes with cellular oxidative metabolism.
c. Clinical features depend on the CO-Hb level.
1. Low levels are associated with nonspecific symptoms such as headache, flulike illness, dyspnea with exertion, and dizziness.
2. High levels are associated with visual and auditory changes, vomiting, confusion and later syncope, slurred speech, cyanosis, myocardial ischemia, coma, and death.
3. Classic physical examination findings, although uncommon, include cherry red skin (venous blood carries more oxygen than normal as a result of impaired release of oxygen to tissues) and retinal hemorrhages. Tachycardia and tachypnea may be present.
4. Young children (<8 years) have more symptoms at lower CO-Hb levels. Young children are also more likely to have gastrointestinal symptoms (e.g., vomiting and diarrhea) instead of neurologic symptoms.
5. Delayed permanent neuropsychiatric syndrome, consisting of memory loss, personality changes, deafness, and seizures, may occur in some victims up to 4 weeks after CO exposure.
d. Diagnosis is made by measuring the CO-Hb level. It is important to remember that CO-Hb levels are not always indicative of the degree of CO exposure and may even be low in victims with significant intoxication. Other abnormal findings include anion-gap metabolic acidosis, low oxygen saturation (however, PaO2 may be normal), and evidence of myocardial ischemia on ECG or elevated cardiac enzymes.
e. Management
1. One hundred percent oxygen is administered to displace CO from hemoglobin.
2. If available, hyperbaric oxygen more rapidly displaces CO from hemoglobin as compared with oxygen alone and also improves oxygen delivery to tissues.
3. Hospitalization is indicated for CO-Hb levels > 25%, CO-Hb levels > 10% during pregnancy, history or presence of neurologic symptoms, or presence of metabolic acidosis or ECG changes.

FIGURE 20.1 Selected physical examination findings and their associated toxins.

Table 20-3
Selected Toxins and Their Antidotes

Toxin Antidote
Acetaminophen NAC
Anticholinergic agents Physostigmine
Benzodiazepines Flumazenil
Black widow spider envenomation Antivenin Latrodectus mactans
Carbon monoxide Oxygen
Coral snake envenomation (Eastern U.S. or Texas coral
snake) Antivenin Micrurus fulvius
Cyanide Cyanide antidote kit (contains amyl nitrite, sodium nitrite, sodium
thiosulfate)
Hydroxocobalamin (vitamin B12)
Digitalis glycosides Digoxin-specific Fab antibodies
Heavy metals (e.g., mercury, manganese, copper, gold,
nickel, zinc, lead, arsenic) d-Penicillamine (for lead, mercury, arsenic, copper)
Dimercaprol (British anti-lewisite [BAL] in oil) for all the heavy metals
and lewisite (chemical weapon)
DMSA (for lead and possibly mercury, arsenic, other metals)
EDTA, calcium (for lead, nickel, zinc, manganese)
Inducers of dystonia Diphenhydramine
Benztropine
Inducers of methemoglobinemia Methylene blue
Iron Deferoxamine
Isoniazid Pyridoxine (vitamin B6)
Methanol; ethylene glycol Ethanol, fomepizole
Narcotics Naloxone
Organophosphates; carbamate pesticides Atropine
Pralidoxime (for organophosphates)
Pit viper snake bite (rattlesnake, water moccasin,
copperhead envenomation) Crotalidae polyvalent, antivenin, Crotalidae polyvalent Fab antibodies
β-Blockers; calcium-channel blockers Glucagon
Sulfonylurea oral hypoglycemic agents Octreotide
Glucagon
EDTA = ethylenediaminetetraacetic acid; DMSA = dimercaptosuccinic acid; NAC = N-acetylcysteine.

Table 20-4
Stages of Acetaminophen Toxicity

Stage Time After
Ingestion Signs and Symptoms
1 30 minutes–
24 hours Asymptomatic, or vomiting and diarrhea
2 24–72 hours Gastrointestinal symptoms resolve; at 36 hours, hepatic transaminases begin to increase
3 72–96 hours Hepatic necrosis, jaundice, hypoglycemia, lactic acidosis, hepatic encephalopathy, coagulopathy,
and renal failure
4 4 days–2 weeks Resolution of symptoms, progressive liver damage requiring liver transplantation, or death
Table 20-5
Stages of Iron Toxicity

Stage Time After Ingestion Signs and Symptoms
1 1–6 hours Abdominal pain, vomiting, diarrhea, GI bleeding
Shock from bleeding and vasodilation
Fever and leukocytosis
2 6–12 hours Resolution of stage 1 symptoms
3 12–36 hours Metabolic acidosis

Circulatory collapse
Hepatic and renal failure
DIC
Neurologic deterioration
4 2–6 weeks Late sequelae includes pyloric or intestinal scarring with stenosis
DIC = disseminated intravascular coagulation; GI = gastrointestinal.

IX. Mammalian Bites
A. Epidemiology
1. Dogs (80% of bites), cats, rodents, other wild or domesticated animals, and humans (2– 3% of bites) may all cause a bite injury.
2. Most bites occur in boys during the spring and summer months.
B. Dog bites
1. Clinical features
a. Bites range in severity from scratches, punctures, and lacerations to severe soft tissue injury. Note that the jaw pressure of a dog may exceed 200–450 pounds per square inch.
b. Young children are typically bitten on the head and neck, whereas older children are bitten predominantly on the extremities.
c. Secondary infections may result from anaerobic and aerobic organisms, such as
Staphylococcus aureus, Pasteurella multocida, and Streptococcus species.
2. Management. Treatment includes meticulous and prompt local wound care.
a. Copious wound irrigation
b. Wounds on the face or large wounds should be sutured if less than 12 hours old. Facial wounds <24 hours old can be sutured because the face has increased vascularity, which assists with healing and minimizes the risk of infection.
c. Wounds at high risk for infection include those on the hand, wrist, and foot, and small puncture wounds.
d. Antibiotic prophylaxis, such as amoxicillin–clavulanic acid, should be administered.
e. Tetanus prophylaxis should be given if needed.
1. Tetanus toxoid vaccine should be given if the most recent tetanus vaccine was given greater than 5 years ago.
2. Tetanus immune globulin may also be recommended if the date of the last tetanus vaccine is unknown.
C. Cat bites
1. Clinical features
a. Puncture wounds to the upper extremity are most common.
b. Victims have a high risk of infection due to P. multocida.
c. Cat scratch disease (regional lymphadenitis) may also develop (see Chapter 7, section XVII.B).
2. Management. Treatment is similar to the management of dog bites. Because the injury is typically a puncture wound, adequate irrigation is often difficult.
D. Human bites
1. Clinical features
a. Wound is typically located on the trunk or face in young children. If the wound occurred during a fistfight, it is typically located at the metacarpophalangeal (MP) joint. Wounds to the MP joint are extremely serious, because infection may penetrate the avascular fascial layers, resulting in deep infection and tendonitis.
b. Infection rate is high.
1. Mixed bacterial infection is often present. Pathogens include Streptococcus viridans, S. aureus, and anaerobic bacteria such as Bacteroides, Peptostreptococcus, and Eikenella corrodens.
2. Other systemic infections, such as hepatitis B, HIV, and syphilis, may also be transmitted.

c. Management. Treatment includes copious wound irrigation, closure of large lacerations, and antibiotics (e.g., amoxicillin–clavulanic acid).

X. Biologic Poisonings (Venoms)
A. Black widow spider (Latrodectus species)
1. General concepts
a. The black widow spider is characterized by a red or orange hourglass marking on the ventral surface.
b. Only the female spider is dangerous, and the female spider only bites if provoked.
c. The web is located in dark recesses, such as closets, woodpiles, and attics.
d. Black widows are found in temperate regions throughout the world, which include the United States, southern Europe and Asia, Australia, Africa, and much of South America.
2. Clinical features result from the venom, a potent neurotoxin.
a. The bite causes few local symptoms, except for burning or a sharp pinprick sensation.
b. Pathognomonic signs and symptoms include severe hypertension and muscle cramps.
c. Nonspecific symptoms, such as headache, dizziness, nausea, vomiting, anxiety, and sweating, may also occur.
3. Management
a. Local wound care, including wound irrigation and tetanus prophylaxis (if needed), is important.
b. Benzodiazepines and narcotics may relieve muscle cramps.
c. Latrodectus antivenin is given for signs and symptoms suggesting severe envenomation.
B. Brown recluse spider (Loxosceles species)
1. General concepts
a. The brown recluse spider (also called the fiddleback spider) is characterized by a
brown violin-shaped marking on the dorsum of the thorax.
b. The spider only bites if provoked.
c. The web is located in dark recesses.
d. The known geographic range of the brown recluse spider extends from Nebraska to Ohio and across the south from Texas to Florida.
2. Clinical features result from the venom, a cytotoxic compound containing tissue- destructive enzymes.
a. The bite results in little initial pain. However, 1–8 hours later, a painful itchy papule that increases in size and discolors during the course of 3–4 days develops at the site of the bite.
b. Some patients develop a necrotic and ulcerated deep lesion at the bite site.
c. Systemic reactions may occur 24–48 hours after the bite with fever, chills, weakness, vomiting, joint pain, DIC, hemolysis, and renal failure from myoglobinuria.
3. Management. Treatment includes local wound care and tetanus prophylaxis, if needed. Treatment of a necrotic ulcer is controversial but may include steroids, skin grafting, dapsone, and hyperbaric oxygen. There is no antivenin.
C. Pit viper snakes (family Crotalidae) account for more than 95% of all snakebites. Rattlesnake, cottonmouth, and copperhead snakes are members of the Crotalidae family.
1. Pathophysiology
a. Bite location and amount of venom injected determine the severity of envenomation. Head and trunk bites are most severe.

b. Venom is a complex mixture of proteolytic enzymes.
2. Clinical features
a. Local findings include puncture marks and progressive severe swelling and ecchymosis.
b. Systemic effects include paresthesias of the scalp, periorbital fasciculations,
weakness, diaphoresis, dizziness, nausea, and a metallic taste in the mouth.
c. Coagulopathy, thrombocytopenia, hypotension, and shock may also develop.
3. Management. Treatment involves local wound care, tetanus prophylaxis if needed, immobilization of the bitten extremity, and immediate transport to the nearest emergency department.
a. Incision and suction are not recommended.
b. Tourniquets, ice, and direct pressure on the wound may cause more injury.
c. Crotalidae polyvalent antivenin should be considered for all bites.
1. Children require more antivenin because they receive proportionally more venom per kilogram body weight.
2. Antivenin is most effective if given within 4–6 hours of the bite.
3. Complications of antivenin are common and include serum sickness and anaphylaxis.
d. Crotalidae polyvalent immune Fab is also available for envenomation and is safe, more potent, and very effective.
D. Coral snakes (family Elapidae) account for 1–2% of all snakebites. Coral snakes may be identified by their stripe pattern using the mnemonic “red next to yellow, kill a fellow; red next to black, venom lack.”
1. Clinical features result from the neurotoxic venom and include mild local swelling and tenderness and severe systemic symptoms, such as paresthesias, vomiting, weakness, diplopia, fasciculations, confusion, and respiratory depression.
2. Management should be aggressive and includes antivenin (available only for the Eastern U.S. and Texas coral snake), local wound care, and supportive care.

Review Test
1. You are summoned urgently to the waiting room of the emergency department by a clerk after she witnessed a 3-year-old boy collapse. You are concerned that he may be in cardiac arrest. Which of the following is the correct order of steps for the initial management of a child found in full cardiopulmonary arrest?
A. Open the airway, check for a pulse, administer rescue breaths, and start chest compressions.
B. Check for a pulse, open the airway, start chest compressions, and administer rescue breaths.
C. Check for a pulse, start chest compressions, open the airway, and administer rescue breaths.
D. Open the airway, administer rescue breaths, start chest compressions, and check for a pulse.
E. Check for breathing and a pulse, and start chest compressions.
2. A 2-year-old boy sustains severe head trauma in a three-story fall. Which of the following statements regarding head injury in a patient of this age is correct?
A. Epidural hematoma has a crescentic density on head computed tomography.
B. The prognosis is better if he has a subdural hematoma rather than an epidural hematoma.
C. Cushing triad is an early sign of increased intracranial pressure.
D. The occipital lobe is the most common site of intracerebral hematoma.
E. Bradycardia is an early sign of herniation.
3. A 1-year-old girl has been involved in a motor vehicle accident, and you suspect she has sustained head injury. On examination, she is unconscious but she opens her eyes to pain and also has abnormal flexion of her extremities to pain. She does not cry but rather grunts with stimulation. What is her Glasgow coma score?
A. 4
B. 5
C. 6
D. 7
E. 8
4. A 19-month-old male infant has a burn on his right hand that occurred 3 hours ago. On examination, the burn appears to be in a “stocking glove” distribution. The involved area is moist, painful, and red and contains two moderate-sized intact blisters. Which of the following statements regarding this burn is correct?
A. This patient has suffered a superficial partial-thickness burn.
B. This burn will most likely scar.
C. Child abuse is unlikely based on the burn’s characteristics.
D. Blisters should be ruptured and debrided.
E. Hospitalization is not needed.
5. A 2-year-old girl is found submerged in a lake. She is suspected to have been under the water for more than 15 minutes. Paramedics find her apneic and pulseless. On arrival at the hospital, her core body temperature is 82.4°F (28°C). Which of the following statements regarding the management, expected clinical findings, and prognosis in this patient is correct?
A. Management is dependent on whether the lake contains fresh or salt water.
B. Because the child was found apneic and pulseless, resuscitative efforts should not be attempted.
C. Cervical spine immobilization is unnecessary.

D. Pulmonary function would be expected to improve during the next 18 hours.
E. The prognosis is poor.
6. A 17-year-old girl is brought by her parents to the emergency department after a possible suicide attempt. She discloses that she swallowed 100 aspirin tablets 4 hours ago. Which of the following acid–base relationships would most likely be found on an arterial blood gas study of this patient?
A. Metabolic alkalosis and respiratory acidosis
B. Metabolic alkalosis and respiratory alkalosis
C. Metabolic acidosis and respiratory acidosis
D. Metabolic acidosis and respiratory alkalosis
E. Respiratory alkalosis only
7. A 2-year-old girl is brought to the emergency department by her parents. She swallowed an unknown amount of industrial-strength drain cleaner from her father’s plumbing van
30 minutes ago. Which of the following statements regarding the clinical findings and management of this incident is correct?
A. The drain cleaner will cause coagulation necrosis
B. Activated charcoal should be administered.
C. Nasogastric tube for lavage should be avoided.
D. Lower intestinal perforation is a possible complication.
E. The drain cleaner should be neutralized with acetic acid.
8. A 14-year-old boy is involved in a fistfight with another teenager at school. He is brought to the emergency department 2 hours later with a laceration on the dorsum of his right hand over the knuckle secondary to a human bite. Which of the following statements regarding this human bite is correct?
A. If infection develops, it is usually secondary to both anaerobes and aerobes.
B. If irrigated appropriately, human bites are unlikely to become infected.
C. This wound most likely involves an interphalangeal joint.
D. Antibiotics should not be prescribed until an infection develops because of the risk of selection of resistant organisms.
E. If infection develops, it is most likely caused by Pasteurella multocida.
9. A 5-year-old girl is brought to a rural emergency department after being bitten on the leg by a rattlesnake. The bite occurred 30 minutes ago. No antivenin is available at the hospital, and transfer to another hospital is pending. Which of the following should be performed immediately?
A. Local wound care, leg immobilization, and supportive care only.
B. Incise the wound and apply suction to remove the venom.
C. Apply direct pressure to the wound.
D. Apply a tight tourniquet proximal to the wound to prevent venom from spreading.
E. Rub ice over the fang marks.

The response items for statements 10 and 11 are the same. You will be required to select one answer for each statement in the set.

A. Black widow spider
B. Brown recluse spider

For each of the following patients, select the most likely spider bite.

1. A 5-year-old girl with severe muscle cramps and hypertension.
2. A 10-year-old boy with fever, chills, vomiting, and disseminated intravascular coagulation 24 hours after a suspected bite. Five days later, a necrotic, ulcerated skin lesion is evident.

The response items for statements 12–14 are the same. You will be required to select one answer for each statement in the set.

A. Distributive shock
B. Septic shock
C. Cardiogenic shock
D. Hypovolemic shock
E. Neurogenic shock

For each patient, select the most likely type of shock.

1. A 10-month-old female infant presents with hypotension. An electrocardiogram reveals supraventricular tachycardia.
2. A 10-year-old boy presents with a 2-day history of high fever, chills, vomiting, diarrhea, and weakness. Examination reveals hypotension with bounding pulses and warm extremities.
3. A 5-year-old boy presents with acute onset of wheezing, urticaria, stridor, and hypotension.
4. A 2-year-old girl is brought to the emergency department after ingesting her mother’s prenatal vitamins 60 minutes ago. Which of the following statements regarding the clinical findings, diagnosis, and management of this type of poisoning is correct?
A. Activated charcoal is effective.
B. Basophilic stippling is seen on peripheral blood smear.
C. A test dose of deferoxamine should be the first step in management.
D. An abdominal radiograph may reveal the ingested poison.
E. Shock is uncommon.

Answers and Explanations
1. The answer is E [I.B]. The CABs (Circulation, Airway, Breathing) should begin every resuscitation of a child. If the victim is unresponsive, the rescuer should shout for help and activate the emergency response system. While looking for breathing, the rescuer should simultaneously feel for a pulse. The airway should be opened by either a chin lift–head tilt maneuver or by a jaw-thrust maneuver if the patient has suspected neck or cervical spine injury. If the patient is bradycardic or asystolic, chest compressions are administered.
2. The answer is E [III.E.1.d]. In children younger than 4 years, bradycardia is often the initial sign of cerebral herniation. Epidural hematomas have a lenticular appearance on head computed tomography, whereas subdural hematomas generally have a crescentic appearance. Patients with subdural hematomas generally have a worse prognosis compared with those with epidural hematomas, because of the bleeding directly on the surface of the brain parenchyma. Cushing triad (in which bradycardia is seen with hypertension and an irregular breathing pattern) is a late finding of increased intracranial pressure. The frontal and temporal lobes are the most common sites of intracerebral bleeding.
3. The answer is D [Table 20-1]. The Glasgow coma score (GCS) is used to assess the level of neurologic impairment on the basis of the patient’s physical examination. It includes assessment of three components: eye opening, motor response, and verbal response. This scoring system has been modified for the nonverbal pediatric patient. This patient has a GCS score of 7; she opens her eyes to pain (2), has abnormal extremity flexion to pain (3), and grunts with stimulation (2). A GCS of less than 8 signifies severe head injury.
4. The answer is A [VI.B.2.c, IV.B.2.a, and IV.C]. This patient’s burn is classified as a superficial partial-thickness burn on the basis of the presence of pain, blisters, and erythema. Superficial partial-thickness burns involve the epidermis and outer dermis and do not scar. Child abuse must be considered for well-demarcated burns (e.g., “stocking glove” distribution) that suggest submersion injury. If intact, blisters should not be ruptured because the risk of infection and loss of fluid from the skin would increase. All patients with burns to the hands, feet, perineum, face, and skin overlying joints should be hospitalized for treatment.
5. The answer is E [V.D–F]. A poor prognosis is associated with age younger than 3 years, submersion times greater than 5 minutes, and the need for cardiopulmonary resuscitation, all of which are present in this patient. In general, however, the outcome of drowning in children is better than that in adults, because children have a primitive dive reflex that preferentially shunts blood to vital organs such as the brain, heart, and liver. Management is the same regardless of the type of water in which the patient was submerged. Despite the apnea and asystole, this patient should be resuscitated, and efforts at resuscitation should continue until the core body temperature reaches at least 89.6°F (32°C). Head and neck trauma should always be suspected, and therefore cervical spine immobilization is important. Pulmonary function tends to deteriorate during the 12–24 hours after submersion injury.
6. The answer is D [VIII.D.2.d]. Salicylates are weak acids and cause an anion-gap metabolic acidosis. However, salicylates also directly stimulate the respiratory centers in the brainstem. The stimulation of the respiratory center causes hyperventilation, which overcompensates the metabolic acidosis, producing a respiratory alkalosis. Therefore, the most common acid–base finding in a patient who has ingested a toxic amount of salicylates is a metabolic acidosis and a respiratory alkalosis.
7. The answer is C [VIII.D.5.b–c]. Drain cleaners are alkalis, which typically cause severe deep burns to the mouth and esophagus by liquefaction necrosis. In contrast, acids cause coagulation necrosis that produces superficial damage. Because the esophagus may be injured, a nasogastric tube for suction or lavage is contraindicated. Activated charcoal should be

avoided as well because it interferes with endoscopy. Neither acids nor alkalis cause lower intestinal perforation. Neutralization with an acid produces an exothermic reaction that results in further injury.
8. The answer is A [IX.D]. Human bites have a very high infection rate despite appropriate wound cleaning. Infection is typically a mixed infection consisting of both aerobes and anaerobes. The hand, specifically the metacarpophalangeal joint, is most commonly bitten in a fistfight. Antibiotics should always be prescribed. Infection with Pasteurella multocida occurs in cat and dog bites, but not in human bites. Other infections such as syphilis, human immunodeficiency virus, and hepatitis B also may be transmitted by human bites.
9. The answer is A [X.C.3]. Management of a rattlesnake bite involves the immediate administration of snake antivenin. If antivenin is unavailable, the victim should be transported to a facility that has antivenin. Local wound care (including tetanus prophylaxis if needed) and immobilization of the affected extremity are all that are needed until antivenin becomes available. Incision and suction are not recommended, and direct pressure, tourniquets, and ice may aggravate the injury.
10. The answers are A and B, respectively [X.A and X.B]. The black widow spider (Latrodectus species) generally produces few local symptoms; however, muscle cramps and systemic hypertension are pathognomonic. The brown recluse spider (Loxosceles species) produces significant local signs and symptoms in some patients, including a deep necrotic ulcerated skin lesion. In some patients, systemic symptoms, such as disseminated intravascular coagulation, vomiting, fever, chills, hemolysis, and joint pain, may occur 24–48 hours after the bite.
11. The answers are C, B, and A, respectively [II.B.2.d, II.B.2.b and II.B.2.c]. The 10-month-old female infant with supraventricular tachycardia has cardiogenic shock, which is generally caused by cardiac dysrhythmias. The 10-year-old boy has signs and symptoms of infection. Shock associated with infection may include septic shock and hypovolemic shock. However, bounding pulses and warm extremities are found only in the initial hyperdynamic stage of septic shock. The 5-year-old boy has signs and symptoms of anaphylaxis. Shock associated with anaphylaxis is distributive shock, which may also be secondary to spinal cord injury and drug poisoning.
12. The answer is D [VIII.B.4.b, VIII.D.3.d and Table 20-5]. Prenatal vitamins contain iron, a cause of childhood poisoning. Iron tablets are radiopaque and may be seen on imaging studies of the abdomen. There are four clinical stages of iron ingestion, including the first stage in which gastrointestinal bleeding can occur along with shock and vasodilation. Management of suspected iron ingestion includes whole-bowel irrigation if poisoning is severe, and treatment with deferoxamine, an iron chelator. Activated charcoal is not effective for iron ingestion. Basophilic stippling is seen in lead intoxication. A test dose of deferoxamine may be very helpful in the management of this patient, but it would not be the first step in management.

Comprehensive Examination

1. At a routine health maintenance visit, the parents of a well-appearing 2-week-old female infant have no concerns other than jaundice, which they have noted since the infant was 1 week of age. The patient is alert and feeding well. Laboratory evaluation reveals a total
bilirubin level of 14.2 mg/dL and a direct bilirubin level of 6.2 mg/dL. Which of the following studies is most likely to be diagnostic of this patient’s condition?
A. Hepatic ultrasound
B. Complete blood count and blood culture
C. Coombs testing
D. Levels of thyroxine and thyroid-stimulating hormone (TSH)
E. Osmotic fragility test
2. The parents of a 15-month-old boy bring him to the office for a routine health maintenance visit. Which of the following is a contraindication to administering the diphtheria, tetanus, and acellular pertussis (DTaP) vaccine at this time?
A. Signs and symptoms of an upper respiratory illness with a temperature up to 38.3°C (101°F)
B. A history of intravenous immune globulin infusion for immune thrombocytopenic purpura at 9 months of age
C. A 4-year-old sibling in the household receiving chemotherapy
D. A history of moderate swelling at the injection site after the 6-month DTaP vaccination
E. Uncontrolled epilepsy
3. A 6-year-old girl with a 3-week history of malaise, decreased appetite, and intermittent tactile fevers is brought to see you by her parents for evaluation of a “purplish rash” around her eyes. On further questioning, you learn that the child also has less energy, which the parents attribute to the “flu.” Joint swelling, abdominal pain, oral ulcers, cough, nausea, vomiting, and diarrhea are absent. Physical examination reveals an afebrile, tired-appearing child with a periorbital violaceous rash that crosses the nasal bridge. There is no evidence of arthritis, although the patient has an erythematous hypertrophic rash over the knuckles. Which of the following laboratory or diagnostic studies is most consistent with the most likely diagnosis?
A. Abnormal electromyography findings
B. Increased serum immunoglobulin levels against Epstein–Barr virus
C. Positive antinuclear antibody
D. Positive urine culture for cytomegalovirus
E. Low creatine kinase levels
4. A 13-year-old previously healthy boy is injured when his scooter collides with a parked car. On arrival at the emergency department, he is unconscious, and you suspect head injury. On examination, he does not open his eyes to voice or to pain. He has decerebrate posturing of the extremities to pain, and he moans with stimulation (you are unable to understand any of his words). Which of the following is this patient’s Glasgow coma score (GCS)?
A. 4
B. 5
C. 6
D. 7
E. 8
5. A 5-year-old boy has a 2-week history of persistent daily fevers and a 1-week history of pain in his right arm. Physical examination reveals a very pale child with a markedly enlarged liver

and spleen and generalized lymphadenopathy. His right arm is nontender to palpation with normal range of motion. On the basis of his clinical presentation and examination, you suspect acute lymphocytic leukemia (ALL). Which of the following statements about the presenting features of ALL is most accurate?
A. The age at presentation is unusual.
B. Bone pain is a common presenting symptom.
C. The disease is more common in females.
D. Pallor is an uncommon sign at presentation.
E. White blood cell count is almost always high (˃50,000 cells/mm3).
6. A 7-year-old boy presents to the emergency department with swelling in his knee. His parents state that the swelling developed very rapidly over 30 minutes after their son bumped his knee into a door. On physical examination, you note that his knee is very swollen with what appears to be blood. Based on his presentation, you consider an evaluation for hemophilia A. Which of the following statements regarding this diagnostic possibility is correct?
A. Activated partial thromboplastin time (aPTT) is prolonged.
B. Males and females are equally affected.
C. Management includes replacement of factor IX.
D. Bleeding time is prolonged.
E. The cause is a deficiency of a vitamin K–dependent coagulation factor.
7. A 3-year-old boy has a 2-week history of viral upper respiratory symptoms and a 2-day history of swelling around his eyes. Physical examination reveals normal vital signs and moderate periorbital edema, mild scrotal edema, and mild edema of the feet. Urinalysis reveals 3+ proteinuria. Which of the following statements regarding the likely diagnosis is most accurate?
A. Serum cholesterol level is low.
B. He is predisposed to hemorrhage.
C. Should the patient become febrile, management involves administration of empiric antibiotics to cover encapsulated organisms.
D. Referral for a diagnostic renal biopsy is warranted.
E. The patient is not likely to respond to corticosteroid therapy and will likely require treatment with cyclophosphamide or cyclosporin.
8. A 16-year-old girl presents with a 1-week history of gray-white vaginal discharge with a strong “fishy” odor. On physical examination, you confirm the fishy odor and note minimal vulvar inflammation. Which of the following is correct regarding her likely condition?
A. Treatment of her sexual partners is necessary.
B. Wet-mount saline preparation will demonstrate motile protozoa.
C. The infection is caused by a change in the normal vaginal flora.
D. Topical antiyeast medication should be prescribed.
E. Oral doxycycline and intramuscular ceftriaxone should be administered after obtaining appropriate cultures.
9. A 7-year-old girl is brought to the emergency department with a 5-day history of fever, malaise, and anorexia. Her past medical history is significant for aortic stenosis. Physical examination reveals a loud systolic ejection murmur at the cardiac base that radiates to the carotids and a systolic ejection click. Splenomegaly and a petechial eruption on the patient’s palms and soles are also present. Laboratory studies reveal a white blood cell count of 19,000 cells/mm3 and an erythrocyte sedimentation rate of 78 mm/hour. Which of the following statements regarding the most likely diagnosis is correct?
A. Abnormal rheumatoid factor is unusual.
B. Transthoracic echocardiogram is the most sensitive test for detecting vegetations.
C. Gram-negative rods are the most common causative agents.

D. Ophthalmologic evaluation is useful in diagnosis.
E. Antibiotics should be administered promptly without waiting for blood cultures to be drawn.
10. A 6-month-old male infant is referred to you for evaluation of failure to thrive. The parents report that their child has always been a “poor feeder” and they deny any vomiting, diarrhea, or frequent infections. Physical examination reveals generalized weakness with particularly poor head control owing to weakness of the neck flexor muscles. One of the diagnostic possibilities is juvenile myasthenia gravis. Which of the following statements regarding juvenile myasthenia gravis is most accurate?
A. Most patients with this condition have fasciculations.
B. This condition is most likely related to ingestion of honey.
C. Bilateral ptosis is the most common presenting sign of this condition.
D. Deep tendon reflexes are diminished.
E. The Tensilon test demonstrates increased weakness.
11. A 6-year-old girl has a history of daily fevers to 102°F (38.9°C) for the past 2 weeks. Preliminary workup and physical examination have not revealed a diagnosis. Which of the following statements regarding this patient’s fever of unknown origin (FUO) is correct?
A. The FUO is likely caused by a rare illness.
B. The FUO is most likely caused by a rheumatologic disorder.
C. Hospitalization is not indicated unless the patient becomes more ill.
D. The FUO is likely to resolve without a diagnosis having been made.
E. The FUO is likely caused by an unusual presentation of a common infection.
12. You are called to evaluate a 1-day-old infant born at 32 weeks’ gestation who develops respiratory distress at 12 hours of life. Which of the following diagnoses is the most likely cause of this infant’s respiratory distress?
A. Meconium aspiration syndrome
B. Bronchopulmonary dysplasia
C. Respiratory distress syndrome
D. Neonatal sepsis
E. Intraventricular hemorrhage
13. A 7-month-old male infant is born at 29 weeks’ gestation. While hospitalized in the neonatal intensive care unit, he develops necrotizing enterocolitis and undergoes a resection of the majority of his small intestine. He is now dependent on total parenteral nutrition for his calories and growth. Which of the following is a complication associated with the treatment of his disorder?
A. Gastrointestinal bleeding
B. Gallstones
C. Chronic constipation
D. Pancreatic insufficiency
E. Inflammatory bowel disease
14. A 9-year-old girl is brought to the emergency department after falling off her bicycle. She complains of pain in her right leg. Radiography reveals a tibial fracture that extends from the epiphysis, through the physis, and into the metaphysis. Which of the following is the Salter– Harris grade of this fracture?
A. Grade I
B. Grade II
C. Grade III
D. Grade IV
E. Grade V
15. A 6-month-old male infant has an asymmetrically shaped head. A computed tomography scan

of the skull reveals premature closure of a cranial suture. Which of the following is the most likely cause of this condition?
A. Perinatal asphyxia
B. Genetic syndrome
C. Positional, from sleeping on the back
D. Unknown
E. Congenital muscular torticollis

The response options for statements 16 and 17 are the same. You will be required to select one answer for each statement in the following set.

A. Strawberry hemangioma
B. Nevus flammeus
C. Erythema toxicum neonatorum
D. Pustular melanosis
E. Milia
F. Cutis marmorata
G. Acrocyanosis
H. Neonatal acne

For each patient, select the skin lesion described or associated with the clinical presentation.

1. A 6-month-old male infant has seizures. Computed tomography of the head reveals intracranial calcifications.
2. A 5-day-old female infant has papules and pustules on her trunk. Microscopic evaluation of the fluid within a lesion reveals eosinophils.
3. A 16-year-old boy with homocystinuria is on a methionine-restricted diet. He is also taking folic acid and pyridoxine. Which of the following medications should you add to his medical regimen?
A. Propranolol
B. Vitamin C
C. d-Penicillamine
D. Betaine
E. Zinc

The response options for statements 19–21 are the same. You will be required to select one answer for each statement in the following set.

A. Turner syndrome
B. Prader–Willi syndrome
C. Kallmann syndrome
D. Laurence–Moon–Biedl syndrome
E. McCune–Albright syndrome

For each patient, select the most likely diagnosis.

1. A 3-year-old girl has bilateral femur fractures and Tanner stage 2 breast development.
2. A 15-year-old girl has delayed puberty and a decreased sense of smell.
3. A 6-month-old infant has hypotonia, small hands and feet, and small gonads. Weight and height are at the fifth percentile.
4. An 18-month-old boy is brought to your clinic by authorities for an examination. Neighbors called the police after they noticed that the child had multiple bruises and appeared

malnourished and unkempt. The police are concerned about possible child abuse. Which of the following findings, by itself, is most suggestive of child abuse?
A. Bruises on the knees, shins, and elbows in different stages of healing
B. Metaphyseal (“corner”) fracture of the distal left humerus
C. Burns to the right arm that have an irregular, splashlike configuration
D. Nondisplaced spiral fracture of the left tibia
E. Displaced supracondylar fracture of the right elbow
5. An 11-month-old male infant is brought to your office by his mother. She is concerned because her son coughs and wheezes day and night for 2 weeks after every cold, and he seems to have a cold every month. He uses no medication for these symptoms. He is otherwise well with normal growth and development. The patient’s father smokes cigarettes but always away from his son. Which of the following statements about this patient’s condition is correct?
A. Asthma is very unlikely because of his young age.
B. A trial of prophylactic inhaled cromolyn sodium may be beneficial.
C. Smoking cigarettes in a separate room should not affect the patient’s symptoms.
D. An albuterol inhaler should be prescribed for prevention of his symptoms.
E. Inspiratory and expiratory radiographs should be performed to rule out a foreign body.
6. A 9-month-old male infant is brought to the emergency department in cardiac arrest. His parents found him unconscious, and cardiopulmonary resuscitation was initiated immediately. Which of the following is the most common cause of cardiac arrest in a child?
A. Poisoning
B. Cardiac dysrhythmia secondary to heart disease
C. Seizures
D. Hypoxia
E. Trauma
7. During a routine health maintenance examination, an obese 13-year-old girl is noted to have acanthosis nigricans. Your counseling of the patient includes a discussion of the association between acanthosis nigricans and type 2 diabetes mellitus. Which of the following is also associated with type 2 diabetes mellitus?
A. Human leukocyte antigen (HLA) haplotypes DR3 and DR4
B. Environmental triggers, such as coxsackievirus
C. Islet cell antibodies
D. Peripheral tissue resistance to insulin
E. Insulin antibodies
8. A 7-month-old male infant is brought to your office with a 1-month-long history of fussiness, intermittent vomiting, and nonbloody, foul-smelling stools. He has not gained weight since his 6-month health maintenance examination. His diet consists of formula and wheat cereal that was introduced at 6 months of age. Which of the following is the most likely diagnosis?
A. Lactase deficiency
B. Celiac disease
C. Crohn disease
D. Gastroesophageal reflux disease
E. Cow’s milk protein intolerance
9. You are called urgently to the newborn nursery to evaluate a “blue” infant. On administration of 100% oxygen, there is no significant increase in the patient’s PaO2. Which of the following is the most likely diagnosis?
A. Neonatal pneumonia
B. Tetralogy of Fallot
C. Respiratory distress syndrome
D. Truncus arteriosus

E. Meconium aspiration syndrome
10. A 3-year-old girl is diagnosed with cervical adenitis and is discharged home with a 7-day course of amoxicillin–clavulanic acid. Five days into therapy, she presents for reevaluation with no decrease in the size of the enlarged cervical node. The child’s fevers have also been persistent. Physical examination is remarkable for a febrile toddler with conjunctivitis, pharyngitis, right anterior cervical adenopathy (the node is 2 cm in diameter), and an erythematous macular rash on the chest and back. Which of the following courses of management would be most appropriate at this time?
A. Send the patient home on oral penicillin with a presumptive diagnosis of scarlet fever.
B. Perform a tuberculin skin test and discharge the patient on oral dicloxacillin, with follow- up in 48 hours.
C. Admit the patient for a rule-out sepsis workup that includes evaluation of the blood, urine, and cerebrospinal fluid for bacterial infection.
D. Admit the patient for high-dose intravenous immune globulin therapy.
E. Admit the patient for high-dose corticosteroid therapy.

The response items for statements 29 and 30 are the same. You will be required to select one answer for each statement in the following set.

A. Dilated cardiomyopathy
B. Hypertrophic cardiomyopathy
C. Restrictive cardiomyopathy

For each patient, select the associated cardiomyopathy.

1. A female newborn is born at full term by cesarean section weighing 9 pounds 8 oz. Her mother developed diabetes during pregnancy. The newborn is admitted to the neonatal intensive care unit because of tachypnea. On examination, a harsh systolic ejection murmur is heard.
2. A 12-year-old boy presents to the emergency department after an episode of chest pain that occurred during baseball practice. Electrocardiography reveals evidence of myocardial infarction. Because of this unusual presentation, you suspect he may have anomalous origin of the left coronary artery from the pulmonary artery.
3. A 3-year-old girl is brought to the office with a 2-week history of melena. Physical examination and vital signs are normal. A complete blood count reveals a hemoglobin of
6 g/dL. Which of the following is the next most appropriate step to evaluate the cause of her anemia and bleeding?
A. Computed tomography of the abdomen
B. Meckel scan
C. Colonoscopy
D. Chest and abdominal radiography
E. Upper endoscopy
4. A 3-year-old boy is brought to the office for evaluation of a rash. The parents report that
1 week earlier he had the “stomach flu,” characterized by fever, abdominal pain, and foul- smelling loose stools with occasional blood streaks. The gastrointestinal symptoms resolved spontaneously. For the past 2 days, he has been less active than usual and somewhat irritable with a decreased appetite. On physical examination, the patient is irritable but consolable, with mild periorbital edema, pale mucous membranes, and a petechial eruption on the abdomen. Laboratory studies reveal a leukocyte count of 12,000 cells/mm3, a hemoglobin count of 7.5 g/dL, and a platelet count of 34,000/µL. The serum creatinine is 1.4 mg/dL. Which of the following statements regarding this patient’s most likely diagnosis is most accurate?

A. The thrombocytopenia is most likely caused by antibody-mediated destruction.
B. Antibiotic therapy is not indicated.
C. The anemia is most likely secondary to blood losses associated with his prior episodes of bloody diarrhea.
D. The prognosis is poor.
E. The most common pathogen associated with this condition in North America is Shigella dysenteriae type 1.
5. The parents of an 18-month-old girl return to see you for a follow-up of their child’s anemia. You prescribed iron (5 mg/kg per day) when you discovered that the girl had a microcytic, hypochromic anemia with a hemoglobin (Hgb) of 10.1 g/dL on routine anemia screening. Today, her Hgb is 10 g/dL despite iron therapy for the past 3 months. She is asymptomatic. Which of the following is an appropriate course of management at this time?
A. Represcribe iron and suggest that it be given daily with orange juice.
B. Represcribe iron and suggest that it be given daily with whole cow’s milk.
C. Consider referral for a red blood cell transfusion if the Hgb does not respond to iron within the next month.
D. Order an Hgb electrophoresis and iron level to rule out other causes of anemia before restarting iron.
E. Prescribe folic acid.
6. The parents of a 3-month-old female infant are concerned about their child’s vomiting. For the past 6 weeks, their daughter has spit up after each feed. She takes both breast milk and a cow’s milk–based formula and spits up after either is given. She does not seem bothered by the spitting up, and her parents deny that she has diarrhea, fever, cough, apnea, cyanosis, or discomfort. She weighed 7 pounds 4 oz at birth and now weighs 11 pounds 1 oz. Physical examination is normal. Which of the following is the most appropriate next step in management?
A. Provide education and reassurance that her spitting up is common and normal and will resolve without treatment.
B. Order a pH probe study to document gastroesophageal reflux.
C. Change her diet to a hydrolyzed amino acid formula.
D. Prescribe an H2-blocker as a therapeutic trial.
E. Order a barium upper gastrointestinal study to evaluate the esophagus, stomach, and duodenum.
7. The mother of a 2-year-old girl calls you urgently, concerned that her child fell in the backyard and traumatically avulsed her maxillary incisor. Which of the following is the most appropriate management at this time?
A. The mother should wrap the tooth in a clean, dry towel and bring the tooth to the dentist within 24 hours.
B. The mother should store the avulsed tooth in milk and seek emergency dental care immediately.
C. The mother should scrub the tooth, rinse it with water, and replace it in the socket.
D. The mother should replace the tooth in the socket and seek emergency dental care immediately.
E. No intervention is required.
8. A 3-year-old boy is brought to the office with a 1-week history of upper respiratory symptoms and the acute onset of a limp. Physical examination is notable for a petechial eruption on the thighs. Laboratory analysis reveals a normal platelet count. Which of the following statements regarding the most likely diagnosis is most accurate?
A. This patient has a multiorgan system vasculitis associated with increased serum IgE levels.

B. Recurrences of this condition are rare.
C. Steroids are contraindicated for the treatment of this condition.
D. The rash in this condition typically begins on the palms and soles.
E. The renal manifestations of this condition may not become clinically apparent for up to 3 months after the initial presentation.
9. A 4-year-old previously healthy boy is brought to the emergency department after being bitten by a dog 18 hours ago. He has two small puncture wounds on his left wrist. Which of the following regarding the dog bite in this patient is correct?
A. Wound irrigation is the most appropriate initial treatment.
B. The wound should be sutured.
C. Infection, if it develops, is most commonly caused by Bartonella henselae.
D. Antibiotics should be withheld until infection is clinically present.
E. Tetanus immunization is not needed for dog bites.

The response items for statements 38–41 are the same. You will be required to select one answer for each statement in the following set.

A. Topical tobramycin drops
B. Topical acyclovir drops
C. Baby shampoo scrubs of the eyelids
D. Topical mast cell stabilizer drops
E. Oral erythromycin
F. Oral acyclovir
G. Intravenous penicillin

For each patient, select the most appropriate initial management.

1. A 7-day-old male infant with bilateral conjunctival discharge.
2. An 8-year-old girl with burning, crusting, and scales at the eyelash base.
3. A 9-month-old female infant with purulent conjunctival discharge.
4. A 3-year-old boy with conjunctival redness, itching, and watery discharge.
5. A 6-year-old boy is brought to the office with complaints of sore throat, mild fatigue, and headache for 2 days. On physical examination, he is febrile to 38.3°C (101°F) and is well hydrated, comfortable, and without distress. White exudates on his tonsils and anterior cervical lymphadenopathy are present. Which of the following is the most important initial management step?
A. Initiate antibiotic therapy for “strep throat.”
B. Order a monospot test.
C. Obtain a throat culture.
D. Recommend supportive care with acetaminophen and throat lozenges.
E. Obtain a lateral neck radiograph.
6. You are called to the emergency department to evaluate a 3-year-old girl referred for evaluation of a 2-day history of fever, emesis, and progressive lethargy. Computed tomography of the head demonstrates marked dilation of the ventricular system consistent with communicating hydrocephalus. Which of the following is the most likely cause of this condition?
A. Brain atrophy
B. Chiari type II malformation
C. Bacterial meningitis
D. Aqueductal stenosis
E. Dandy–Walker malformation

The response options for statements 44 and 45 are the same. You will be required to select one answer for each statement in the following set.

A. Vitamin A deficiency
B. Vitamin B1 deficiency (thiamine)
C. Vitamin B6 deficiency (pyridoxine)
D. Vitamin B12 deficiency (cobalamin)
E. Vitamin C deficiency
F. Vitamin D deficiency
G. Vitamin K deficiency

For each patient, select the most likely vitamin deficiency.

1. A 2-year-old boy is a “picky” eater and has anemia, swollen gums, and a deep abrasion to the right leg that has taken almost 3 weeks to heal.
2. A 4-year-old girl has dry eyes and difficulty seeing in the dark.
3. A 17-year-old boy presents with dysuria and increased urinary frequency. He has had several sexual partners and indicates that he uses condoms “most of the time.” Which of the following statements regarding the likely diagnosis is correct?
A. Trimethoprim–sulfamethoxazole should be given for a presumed urinary tract infection.
B. Evidence of greater than five white blood cells per high-power field on a Gram stain of his urethral secretions is sufficient for a presumptive diagnosis.
C. This diagnosis is more common in females.
D. The causative agents almost always cause symptoms that lead to the diagnosis.
E. Obtaining a culture from a urethral swab is the only method that provides a definitive diagnosis.
4. A 7-year-old boy is brought to the office with a 3-day history of painful swelling anterior to his left ear. On physical examination, a small amount of pus can be expressed from Stensen duct. Which of the following statements regarding the most likely diagnosis is correct?
A. Placement of a skin test for tuberculosis should be considered.
B. Supportive care for probable mumps infection is indicated.
C. Epstein–Barr virus or cytomegalovirus infection is the most likely cause of the infection.
D. Surgery should be immediately consulted to drain the infection.
E. Antibiotics to cover Streptococcus pneumoniae are indicated.
5. The parents of a 6-week-old male infant bring him to the office for an evaluation because of concerns about their infant’s color. They note that he turns “purple” whenever he cries and is very fussy. He is otherwise well. He feeds vigorously, and his weight is 1 pound 3 oz above his birth weight. He is pink and happy when not crying. On physical examination, a systolic murmur is heard at the upper left sternal border. Which of the following is the most likely diagnosis?
A. Transposition of the great arteries
B. Tetralogy of Fallot
C. Tricuspid atresia without a ventricular septal defect
D. Total anomalous pulmonary venous connection
E. Truncus arteriosus

The response items for statements 49–53 are the same. You will be required to select one answer for each statement in the following set.

A. Seborrheic dermatitis
B. Contact dermatitis

C. Tinea corporis
D. Pityriasis rosea
E. Miliaria rubra
F. Psoriasis
G. Fifth disease

For each patient, select the most likely diagnosis.

1. An 8-year-old boy has oval scaly erythematous patches on the upper extremity with partial central clearing.
2. A 15-year-old boy has red greasy scales and crusts in the nasolabial skin folds.
3. A 13-year-old girl has scaling papules and plaques. When she injures her skin, new lesions develop at the site of the trauma.
4. A 6-month-old female infant has small erythematous pruritic papules on the chest and neck in the summertime.
5. A 3-year-old boy has erythematous papules in a “Christmas tree” distribution on the trunk.
6. An 18-month-old boy with a history of myelomeningocele repair at birth and subsequent ventriculoperitoneal shunt placement is brought to the office for a routine health maintenance visit. Medical history is also significant for an extensive stay in the neonatal intensive care unit, which was complicated by sepsis. Since the neonatal period, he has been relatively well, although his mother notes occasional rashes described as “red welts.” Considering this patient’s medical history, this patient is most likely to be allergic to which of the following?
A. Penicillin
B. Cow’s milk protein
C. Dust mites
D. Latex
E. Wool
7. At birth, a newborn has thrombocytopenia, enlarged liver and spleen, hearing loss, heart murmur, and bilateral cataracts. His mother recalls a brief viral illness during her first trimester of pregnancy. Congenital infection with which of the following organisms is the most likely cause of this infant’s symptoms?
A. Toxoplasmosis
B. Rubella
C. Varicella
D. Herpes simplex virus
E. Cytomegalovirus
8. An infant is brought to the office for a routine health maintenance visit. The infant can sit with support, vocalize with mixed vowel and consonant sounds, and has just learned to transfer objects from hand to hand. Which is the most likely age of this infant?
A. 4 months of age
B. 6 months of age
C. 8 months of age
D. 9 months of age
E. 10 months of age
9. A 4-year-old girl is brought to the office with a persistent cough. Her mother is concerned because she has had a wet-sounding cough for the past 4–5 months without improvement. Past medical history is significant for three episodes of pneumonia in the past 18 months; each episode was treated with antibiotics. Her mother is also concerned that she has been underweight for the past 12 months. On physical examination, the patient is afebrile, and her respiratory rate is 24 breaths/minute. Her weight and height are both less than the fifth percentile. Lung examination reveals diffuse crackles and wheezes. Which of the following

tests would reveal her most likely underlying diagnosis?
A. Chest radiograph
B. Sputum culture
C. Sweat chloride assessment
D. Bilateral decubitus radiographs
E. Cold agglutinins

The response options for statements 58–60 are the same. You will be required to select one answer for each statement in the following set.

A. Viral meningitis
B. Bacterial meningitis
C. Tuberculous meningitis
D. Partially treated bacterial meningitis
E. Brain abscess
F. Fungal meningitis

For each patient, select the most likely diagnosis.

1. A 14-month-old boy has fever and irritability. Cerebrospinal fluid (CSF) analysis reveals a white blood cell (WBC) count of 7500 cells/mm3 with a polymorphonuclear leukocyte predominance, a low glucose, and a high protein.
2. A 5-year-old girl has had fevers for 2 weeks and is now confused and sleepy. Cerebrospinal fluid (CSF) analysis reveals a white blood cell (WBC) count of 200 WBCs/mm3 with a lymphocyte predominance, a very low glucose, a very elevated protein, and a negative Gram stain.
3. A 9-year-old boy presents with headache, photophobia, and 1 day history of fever. Cerebrospinal fluid (CSF) analysis reveals a white blood cell (WBC) count of 350 WBCs/mm3 with a polymorphonuclear leukocyte predominance, a normal glucose, a normal protein, and a negative Gram stain.
4. A 6-week-old male infant is brought to the office in September with a 2-day history of cough and increased work of breathing. Past medical history is significant for vaginal delivery without complications at 38 weeks. At 10 days of age, he was treated with erythromycin ointment for conjunctivitis. On physical examination, the patient is afebrile. He is coughing rapidly and has a respiratory rate of 54 breaths/minute. Diffuse wheezes are present throughout the lung fields along with mild subcostal retractions. Which of the following is the most likely cause of the patient’s signs and symptoms?
A. Mycoplasma pneumoniae
B. Chlamydia trachomatis
C. Respiratory syncytial virus
D. Streptococcus pneumoniae
E. Asthma
5. A 13-year-old boy has a 3-hour history of severe pain in his right scrotum. On physical examination, his right testicle is diffusely tender and swollen. The right cremasteric reflex is absent. Which of the following statements regarding this patient’s likely diagnosis is correct?
A. Radionuclide imaging will demonstrate increased uptake to the right testicle.
B. Definitive diagnosis can be made on the basis of relief of pain when the right testicle is elevated.
C. Management includes oral doxycycline and oral cefixime for presumptive epididymitis.
D. Surgery consultation within the next 12 hours is indicated for possible surgical intervention.

E. Surgical intervention will involve both testicles.
6. A 7-year-old boy is brought to the office with a 6-month history of daytime enuresis. Which of the following findings would be most important in prompting you to order imaging studies?
A. Positive family history of enuresis in the patient’s father, who wet the bed until he was 12 years of age
B. Onset of symptoms when the parents separated
C. Abnormal anal wink reflex
D. Constipation without encopresis
E. Frequent voiding of small amounts of urine

The response items for statements 64–66 are the same. You will be required to select one answer for each statement in the following set.

A. Acute dactylitis
B. Sequestration crisis
C. Acute chest syndrome
D. Hyperhemolytic crisis
E. Aplastic crisis

For each patient with sickle cell disease, select the type of sickle cell crisis.

1. A 3-year-old boy has pallor, jaundice, and fatigue. He has an increased reticulocyte count, increased bilirubin level, and decreased hemoglobin (Hgb).
2. A 4-year-old girl has pallor and fatigue. Her reticulocyte count and hemoglobin (Hgb) levels are decreased.
3. A 2-year-old boy has pallor, fatigue, tachycardia, and hypotension. His reticulocyte count is high, and his hemoglobin (Hgb) level is low.
4. A 4-month-old female infant is brought to the office for a routine health maintenance visit. Which of the following motor skills accurately describes the expected developmental abilities of this infant?
A. When the infant is pulled from a supine to a sitting position, her head lags behind her shoulders.
B. In the prone position, the infant can raise her head slightly but cannot push up with her arms.
C. In the prone position, the infant can push up using her arms and hold her head up.
D. When pulled from a supine to a sitting position, the infant anticipates the movement and leads with her head.
E. In the prone position, the infant can roll to a supine position and can then get into a sitting position.
5. A 15-month-old boy is brought to the office for a routine health maintenance visit. His parents are concerned that he has not yet started to walk. Other motor milestones and verbal milestones are on target. On examination, you note normal strength and movement of all extremities; however, his thigh folds are asymmetric, and there appears to be a limb length discrepancy of 1 inch. Which of the following is the most likely diagnosis?
A. Transient synovitis
B. Developmental hip dysplasia
C. Legg–Calvé–Perthes disease
D. Femoral anteversion
E. Slipped capital femoral epiphysis

The response items for statements 69–71 are the same. You will be required to select one answer

for each statement in the following set.

A. Wilms tumor
B. Neuroblastoma
C. Lymphoma
D. Pheochromocytoma
E. Rhabdomyosarcoma

For each patient, select the most likely malignancy.

1. A 3-year-old girl presents with the acute onset of an unsteady gait. On physical examination, she has random jerking eye movements in all directions, myoclonus, and ataxia with ambulation.
2. A 18-month-old girl presents for a routine health maintenance evaluation. On physical examination, she has a left-sided abdominal mass, absence of her bilateral irides, and hypertrophy of her right thigh and right leg.
3. A 5-month-old female infant diagnosed with an abdominal malignancy has a localized tumor with metastases to her bone marrow and right femur. Without treatment, her tumor regresses.
4. A 23-day-old male infant has a 2-day history of streaks of blood and mucus with each stool. His parents report no fever, vomiting, or ill contacts. Which of the following is the most likely cause of his symptoms?
A. Juvenile polyp
B. Meckel diverticulum
C. Infectious enterocolitis
D. Allergic colitis (protein intolerance)
E. Inflammatory bowel disease

The response options for statements 73 and 74 are the same. You will be required to select one answer for each statement in the following set.

A. Head ultrasound
B. Dental evaluation
C. Ophthalmologic evaluation
D. Thyroid screen
E. Flexion and extension cervical spine radiographs

For each patient, select the most appropriate screening test to be used to evaluate for a complication of the condition.

1. A short 16-year-old girl with delayed puberty, a webbed neck, and shield chest.
2. A 4-year-old boy with fragile bones, easy bruisability, and blue sclerae.
3. You are called to the emergency department to evaluate a severely ill 5-year-old child who collapsed while at kindergarten. On physical examination, you find a comatose child whose respirations are irregular with no particular pattern. This patient’s respiratory pattern is most suggestive of which of the following?
A. Cerebellar injury
B. Cerebral stroke
C. Narcotic ingestion
D. Impending brain death
4. A 3-year-old boy recently adopted from an international adoption agency is referred to you for evaluation of short stature. The details of the patient’s past medical history are unknown. Physical examination reveals an alert child whose weight is at the third percentile and whose

height is well below the third percentile for age (50th percentile for a 16-month-old child). The patient’s upper-to-lower body segment ratio is high (i.e., above the expected ratio for age).
Which of the following causes might explain this patient’s short stature?
A. Malnutrition
B. Child neglect
C. Growth hormone deficiency
D. Fetal alcohol syndrome
E. Rickets
5. A previously healthy 4-year-old boy is admitted to the hospital for a 5-day history of fever and severe right leg pain. There is no history of trauma. On examination, the boy is nontoxic; however, his right tibia above the ankle is warm, erythematous, and indurated. You suspect he may have osteomyelitis. Which of the following statements regarding this diagnosis is correct?
A. A plain radiograph demonstrates periosteal elevation and should be the initial diagnostic study.
B. The infection was most likely acquired by hematogenous seeding.
C. The most likely pathogen is Salmonella typhi.
D. Blood culture will be positive in almost all patients with osteomyelitis.
E. An oral first-generation cephalosporin should be started for outpatient treatment of osteomyelitis because the child is nontoxic.
6. You are performing an initial physical examination on a term newborn in the delivery room. The presence of which of the following leads you to suspect an underlying congenital anomaly?
A. Umbilical hernia
B. Hypospadias
C. Single umbilical artery
D. Nevus simplex on the right upper eyelid
E. Diastasis recti
7. A 17-year-old boy presents to your office for a routine health maintenance evaluation. He has been well with the exception of some fatigue for the past few months and vague, nonspecific discomfort in his abdomen. On physical examination, the spleen is significantly enlarged, extending down to the left lower quadrant of his abdomen. On the basis of his symptoms and the splenomegaly, chronic myelogenous leukemia is in your differential diagnosis. Which of following statements regarding the most likely type of chronic myelogenous leukemia in this patient is correct?
A. Further examination will likely reveal an eczematous-like facial rash and petechiae.
B. The Philadelphia chromosome is present on cytogenetic analysis of leukemic cells.
C. Radiation therapy is effective.
D. The illness is very likely to be fatal within months.
E. The white blood cell count will likely be ˂30,000 cells/mm3.
8. A 6-year-old girl who lives in Vermont develops fever, headache, muscle and joint aches, and several enlarged lymph nodes. On physical examination, an erythematous ringlike skin rash with a clear center is evident on her trunk. Which of the following would be the most appropriate treatment at this time?
A. Oral doxycycline
B. Oral amoxicillin
C. Intravenous penicillin
D. Intravenous ceftriaxone
E. No antibiotic treatment
9. An 18-month-old female infant is brought to the office with a 3-day history of nasal congestion

and rhinorrhea. Today she developed a fever with temperature up to 38.3°C (101°F) and a barky cough. She is eating well and has remained active. On initial evaluation in the emergency department, she has a normal respiratory rate with no respiratory distress, is playful, and has no stridor. However, when you begin your examination, she begins to cry and immediately you hear inspiratory stridor. As she calms down, the stridor resolves completely. The remainder of the examination is normal. Which of the following should be your management approach?
A. Supportive care only
B. A 10-day course of oral corticosteroids
C. Nebulized albuterol
D. Nebulized racemic epinephrine
E. Nebulized ribavirin
10. A 3-year-old boy has a focal seizure involving his left arm and left leg. Computed tomography (CT) of the head reveals a calcified cyst within the brain parenchyma. Which of the following statements regarding the most likely diagnosis is correct?
A. Surgical excision of the cyst is indicated.
B. Anticonvulsant therapy alone is indicated.
C. Ingestion of contaminated poultry is the likely cause.
D. Ova and parasite stool examination is a reliable and sensitive diagnostic technique.
E. The cyst noted on CT scan is evidence of active infection.
11. A 13-year-old girl who resided outside of the United States for 6 months visited a physician while abroad for knee and ankle pain. She was diagnosed with systemic lupus erythematosus (SLE). She now has returned to the United States, and she is brought to your office for a second opinion. She currently has no symptoms other than mild joint pains. A negative or normal result of which of the following tests makes the diagnosis of SLE most dubious?
A. Erythrocyte sedimentation rate
B. Anti–double-stranded DNA antibody
C. Anti-Smith antibody
D. Antinuclear antibody
E. Rheumatoid factor

The response items for statements 84–88 are the same. You will be required to select one answer for each statement in the following set.

A. Naloxone
B. Oxygen
C. Deferoxamine
D. Ethanol
E. Atropine
F. Pyridoxine
G. Diphenhydramine
H. N-acetylcysteine
I. Flumazenil
J. Glucagon
K. d-Penicillamine
L. Pralidoxime

For each poisoning, select the appropriate antidote.

1. A 15-month-old boy has severe vomiting and bloody stools 3 hours after swallowing 30 prenatal vitamin tablets.

2. A 15-year-old girl has vomiting and abdominal pain 2 hours after swallowing 18 extra- strength acetaminophen tablets.
3. A 6-year-old boy presents in the winter with acute onset of headache, vomiting, confusion, blurry vision, and slurred speech. On examination, his skin is cherry red in color.
4. An 8-year-old boy who is developmentally delayed presents with seizures. His mother found an empty bottle of isoniazid recently prescribed for tuberculosis infection in his bedroom.
5. A 4-year-old girl develops dystonia while being treated with an antiemetic (a phenothiazine) for acute gastroenteritis.
6. A 15-year-old sexually active girl presents with a 6-month history of frequent painful irregular menstrual periods. She describes her periods as occurring every 2–3 weeks and each lasting 10–12 days. The amount of bleeding is very heavy. Which of the following statements regarding her condition is correct?
A. The term menometrorrhagia describes the vaginal bleeding.
B. Management involves hormonal contraception; pelvic examination is not necessary.
C. The bleeding is likely secondary to excessive progesterone production.
D. Laboratory testing is not indicated because her diagnosis is a clinical diagnosis.
E. Dilation and curettage is indicated.
7. A 5-year-old boy is brought to the emergency department. His parents state that he is unable to walk. On physical examination, the patient is afebrile with normal vital signs. He is very irritable and scared. Neurologic examination shows significant symmetric weakness and diminished deep tendon reflexes in the lower extremities. Strength and deep tendon reflexes of the upper extremities are preserved. Sensation is normal in both the upper and lower extremities. Which of the following features is most consistent with the most likely diagnosis?
A. Symmetric descending paralysis
B. Elevated cerebrospinal fluid protein with low cell counts
C. Normal nerve conduction studies
D. Abnormal spinal magnetic resonance imaging
E. Concurrent diarrheal infection with Clostridium botulinum
8. A 3-year-old boy is brought to your office for evaluation of a limp. For the past 2 days he has been limping and crying intermittently, saying that he is in pain and pointing to his left thigh. The patient has been afebrile, and his parents deny any trauma. Medical history further reveals an upper respiratory infection 2 weeks ago that resolved rapidly. On physical examination, the boy appears healthy but anxious, and he refuses to walk. His temperature is 36.4°C (97.6°F). When lying on the examination table, he holds his left hip in external rotation and abduction. Movement of his left hip results in mild pain. Laboratory evaluation reveals a white blood cell count of 11,000 cells/mm3 and an erythrocyte sedimentation rate of
18 mm/hour. Which of the following is the most likely diagnosis?
A. Septic arthritis of the hip
B. Osteomyelitis of the hip
C. Slipped capital femoral epiphysis
D. Transient synovitis
E. Toddler’s fracture
9. During a routine health maintenance visit, the mother of a 3-year-old boy expresses concern that her son has had “too many infections.” On further questioning, you discover he had pneumonia at 6 months of age and at 20 months of age, and “many, many ear infections” since infancy. Most recently, he was treated with a 2-week course of antibiotics for suspected sinusitis. Despite these infections, the child has been growing and developing normally. Which of the following laboratory studies would confirm the most likely diagnosis?
A. Complete blood count
B. Anergy skin test

C. Quantitative serum IgA level
D. Assessment of T-cell response to mitogens
E. Total hemolytic complement
10. The parents of a 4-year-old boy are concerned about their son’s bowed legs. The bowing has been apparent ever since he began to walk. However, they are a bit concerned now because the bowing in his right leg has become more pronounced, whereas his left leg appears to be improving. On physical examination, the right leg is bowed laterally to a greater degree than his left leg, and when he walks, his body shifts laterally away from midline. Which of the following is the most appropriate management of this patient?
A. Standing anterior–posterior radiograph of the lower extremities
B. Referral for physical therapy to strengthen the muscles of the lower extremities
C. Parental reassurance that the condition is genu varum and will resolve without treatment
D. Parental reassurance that the condition is genu valgum and will resolve without treatment
E. Referral to a podiatrist for orthotic shoes
11. A 9-year-old girl has a 2-week history of restlessness, episodes of excessive crying, and increased difficulty writing her name. Magnetic resonance imaging of her head is normal. Which of the following is the most likely diagnosis?
A. Migraine
B. Complex partial epilepsy
C. Sydenham chorea
D. Hydrocephalus with increased intracranial pressure
E. Tourette syndrome
12. A 12-year-old boy returns from a camping trip in the Arkansas mountains. He has fever and severe headache. As you are examining him, a petechial rash suddenly appears on his hands and feet. Which of the following statements regarding the most likely cause of his illness is correct?
A. The illness is caused by infection with a spirochete.
B. Definitive serologic evidence of infection should be obtained before the initiation of therapy.
C. Thrombocytopenia and anemia may be present.
D. Prophylactic antibiotics are indicated to prevent this infection.
E. Incidence of infection is highest in infants and very young children.
13. A 10-year-old boy presents with fever, hypotension, diarrhea, and diffuse erythroderma of the skin. Nikolsky sign is absent. Laboratory studies reveal thrombocytopenia and an elevated creatine kinase. Cultures of his blood, urine, and cerebrospinal fluid are negative. Which of the following is the most likely cause of the infection?
A. Group A β-hemolytic streptococcus
B. Borrelia burgdorferi
C. Staphylococcus aureus
D. Kawasaki disease
E. Escherichia coli O157:H7

The response options for statements 97–100 are the same. You will be required to select one answer for each statement in the following set.

A. High serum ceruloplasmin
B. Low serum copper
C. High serum copper
D. Low serum zinc
E. Low serum calcium

F. High serum calcium
G. Low serum glucose
H. High serum ammonia

For each patient, select the likely laboratory finding.

1. A 12-year-old boy with ataxia, seizures, abnormal behavior, elevated transaminases, and yellowish discoloration of the peripheral cornea.
2. A 6-year-old boy with myoclonic seizures, mental retardation, and hair that is easily breakable and unmanageable.
3. A 12-month-old boy with vesicles and scales in the diaper area, failure to thrive, and chronic diarrhea.
4. A 2-year-old boy with interrupted aortic arch, recurrent fungal infections, a small chin, and short palpebral fissures.
5. On routine examination in the newborn nursery, a healthy 2-day-old female infant born at 39 weeks’ gestation is incidentally noted to have a palpable abdominal mass. The infant is otherwise asymptomatic. She is feeding, voiding, and stooling appropriately. Which of the following is the most common etiology?
A. Wilms tumor
B. Hepatomegaly
C. Ovarian torsion
D. Hydronephrosis
E. Neuroblastoma
6. A 1-day-old full-term male infant is having difficulty feeding with frequent episodes of emesis immediately after feeding. He is transferred to the neonatal intensive care unit, and passage of an oral gastric tube is attempted. The tube is met with resistance, and a chest radiograph confirms the tube is in the upper part of the thorax. On reviewing the infant’s prenatal course, which of the following would be the most likely finding?
A. Hepatomegaly
B. Oligohydramnios
C. Meconium-stained amniotic fluid
D. Congenital diaphragmatic hernia
E. Polyhydramnios

The response items for statements 103–105 are the same. You will be required to select one answer for each statement in the following set.

A. First-degree atrioventricular (AV) block
B. Second-degree AV block, type I
C. Second-degree AV block, type II
D. Third-degree AV block
E. Long QT Syndrome
F. Wolff–Parkinson–White
G. Supraventricular tachycardia

For each scenario, select the most likely arrhythmia.

1. This heart block is associated with children born to mothers with systemic lupus erythematosus (SLE).
2. Risk for lethal ventricular arrhythmias, particularly torsades de pointes
3. Progressive prolongation of the PR interval leading to failed atrioventricular (AV) conduction
4. A 17-year-old female patient with a strict and restrictive vegan diet for the last 3 years presents

to clinic with increasing symptoms of fatigue, clumsiness and pallor for the last several months. On examination, she is noted to have a smooth red tongue, hyporeflexia and mild gait ataxia. Which of the following abnormalities in the laboratory finding would be most likely?
A. Macrocytic anemia
B. Hyponatremia
C. Low white blood cell count
D. Normocytic anemia
E. Prolonged international normalized ratio (INR)

The response items for statements 107–109 are the same. You will be required to select one answer for each statement in the following set.

A. Down syndrome
B. Trisomy 18
C. Trisomy 13
D. Ataxia telangiectasia
E. Turner syndrome (XO)
F. Klinefelter syndome (XYY)
G. CHARGE syndrome

Select the most likely genetic condition associated with the following hematologic/oncologic disorders.

1. A 17-year-old female with gonadoblastoma.
2. A 4-year-old child with transient myeloproliferative disorder who goes on to develop acute myelogenous leukemia.
3. An 11-year-old with recurrent sinopulmonary infections, chronic lung disease, and newly diagnosed lymphoma.
4. The parents of a 3-year-old boy bring him to clinic because of concern that he is walking “pigeon toed”. The child began to walk at age 12 months and has had no other developmental concerns. He has no pain, limp, or frequent falling. On examination, the patient has a normal gait with mild, symmetric intoeing. His thigh–foot angle has 0° of external rotation. Which of the following is the most likely diagnosis?
A. Femoral anteversion
B. Internal tibial torsion
C. Genu varum
D. Talipes equinovarus
E. Metatarsus adductus
5. Your office receives a call from the newborn screening office that a 1-day-old female infant had an elevated thyroid-stimulating hormone (TSH) level on the newborn screen. What should be your next step?
A. Contact the family next week.
B. Begin levothyroxine.
C. Contact the family and obtain a TSH and thyroxine (T4) immediately.
D. Refer to pediatric endocrinology.
E. Order a thyroid ultrasound.
6. You are performing a well-child examination on a 12-year-old male. The parents express concern that members of the family did not enter puberty until very late. They ask you if their son has started puberty. As you begin your physical examination, which of the following is the most common initial sign of puberty in a male?
A. Voice change

B. Apocrine odor
C. Facial hair
D. Growth spurt
E. Testicular enlargement
7. A 7-month-old male infant presents to your office with a history of fever with temperature up to 103°F over the last 24 hours. He vomited twice, is not taking fluids well, and is irritable. He had prenatal hydronephrosis, and his family history is positive for a kidney transplant owing to vesicoureteral reflux in his paternal uncle. On examination, he is an alert but irritable infant. His temperature is 39°C, and he is tachycardic. He is uncircumcised, and his belly is soft but mildly tender to palpation. Otherwise his examination is normal. White blood cell (WBC) count is 18,000 with a left shift. A bagged urine sample shows 30–40 WBCs and positive nitrites. His serum creatinine is 0.7. What are the next appropriate steps?
A. Obtain a catheterized urine culture, start empiric antibiotics until the culture results are back, and follow up in the office in 2 weeks.
B. Obtain a catheterized urine culture, admit to the hospital for intravenous antibiotics, obtain a renal ultrasound, and schedule a voiding cystourethrogram for 2 weeks later.
C. Obtain a catheterized urine culture, start empiric antibiotics, and schedule a renal ultrasound and voiding cystourethrogram for the next day.
D. Obtain a catheterized urine culture but wait for the results before starting antibiotics.
E. Obtain a bagged urine culture, treat with antibiotics, but do no imaging studies, as vesicoureteral reflux is a benign condition.
8. A 7-year-old female is noted to have a low-pitched continuous murmur. The murmur is noted mostly along the superior portion of the right sternal border. The murmur seems loudest when the patient is standing, and it disappears when she is supine. The patient is otherwise healthy and has no cardiovascular symptoms other than some occasional dizziness. Which of the following would you explain to the parents as the source of this murmur?
A. Turbulence within the carotid artery
B. A humming noise resulting from vibration as blood returns to the heart
C. The residua of a fetal connection between the aorta and the pulmonary artery
D. Transmission of heart sounds from the aortic valve
E. A “musical” functional murmur due to turbulence as blood leaves the heart
9. A 15-year-old male with asthma complains of chest pain with running. He visits your office after experiencing one episode of syncope while running. He remembers very little about this episode, just that he “woke up on the ground.” The pain he experiences while running started about 2 weeks ago and is described as sharp and over the left precordium. It has not changed in character, and it gets better when he stops running. His physical examination is normal. Which of the following is the next best step in management of this patient?
A. Increase the frequency and intensity of his asthma treatment
B. Electrocardiogram and immediate consultation with a pediatric cardiologist
C. Referral to a pediatric pulmonary specialist
D. Chest x-ray
E. Reassurance and allow the patient to return to normal activities
10. A 14-year-old female has the onset of sharp chest pain and shortness of breath several days after an upper respiratory infection. The pain is described as “sharp,” and is worse lying down. She feels much better sitting up. On examination, she is in mild respiratory distress and has a fever with a temperature of 38.6°C. Given her presentation and likely diagnosis, which of the following would be the most likely physical finding associated with her diagnosis?
A. Bounding pulses
B. Pulsus paradoxicus
C. Apical click

D. Systolic ejection murmur
E. Crisp heart sounds
11. A 12-year-old gymnast comes to you for “sports clearance.” She is new to your practice. Her past medical history is notable for an appendectomy performed at 5 years of age. Her family history is notable for a maternal grandmother who died suddenly at the age of 74 years with a history of high cholesterol. On examination, her blood pressure is 95/56 mm Hg. She has a grade 1-2/6 vibratory, musical systolic ejection murmur at the mid-left sternal border and apex that is louder when supine and is much softer when she sits up. Her pulses are easily felt in all four extremities. Which of the following would be the next best step for her evaluation and/or management?
A. Schedule an exercise stress test.
B. Check a fasting cholesterol panel.
C. Clear her for all sports without any restrictions.
D. Refer her to a pediatric cardiologist.
E. Restrict her from participating in gymnastics until you reevaluate her in 3 months.
12. A 3-year-old girl presents with low-grade fever and decreased oral intake and activity over the past 24 hours. She is fully immunized and attends daycare. On physical examination, she is noted to have painful blisters on her hands and the bottom of her feet, as well as ulcers on her tongue. Which of the following choices is correct regarding this child’s likely illness?
A. This is typically an illness that will not resolve on its own.
B. She should be prescribed amoxicillin.
C. This is likely caused by an enterovirus.
D. This is not contagious.
E. She should be prescribed acyclovir.
13. A 4-year-old girl presents with a 6-day history of high fevers refractory to antipyretics, tachycardia, and irritable mood. Over the past few days, she has developed a nonpurulent conjunctivitis; dry, red cracked lips; a maculopapular rash on her trunk; and a unilateral
1.5 cm enlarged cervical lymph node. Of the following, what is the next best step in management?
A. Continue treatment with antipyretics and return for medical evaluation if she does not defervesce over the next 72 hours.
B. Prescribe broad spectrum antibiotics to cover Staphylococcus aureus as well as
Streptococcus pyogenes infection.
C. Start treatment with intravenous immune globulin (IVIG) and high-dose aspirin.
D. Obtain a lumbar puncture to rule out meningitis.
E. Initiate high-dose steroid therapy.
14. A 6-year-old female who immigrated from Peru at the age of 4 years is brought in for medical evaluation after a new-onset seizure. Over the past couple of weeks, the parents note that she has appeared tired and has complained of headaches. Computed tomography (CT) of the brain shows a single parenchymal cyst and no obstructive hydrocephalus. Which of the following statements is correct regarding this child’s presentation and likely condition?
A. Neurocysticercosis is one of the leading causes of epilepsy in the developing world.
B. Seizures from this condition do not require anticonvulsant therapy.
C. Taenia eggs on stool ova and parasite (O and P) studies are commonly found.
D. This child requires surgical intervention as part of her treatment.
E. She contracted this infection through ingestion of undercooked fish.
15. An 18-year-old male college student presents to the clinic with acute onset of bilateral parotitis, fever, and headache. Which of the following statements is most consistent with his likely diagnosis?
A. The vaccine administered to prevent this infection is administered at 2, 4 and 6 months.

B. Diagnosis is best made by aspirating the parotid gland.
C. Viral causes of parotitis can include cytomegalovirus (CMV), Epstein–Barr virus (EBV), human immunodeficiency virus (HIV), and influenza.
D. Maternal mumps infection in the first trimester results in congenital mumps malformation.
E. This patient should have received a dose of measles, mumps, and rubella (MMR) vaccine before college entry.
16. During a well-child checkup, a father asks if his 4-year-old needs to be screened for tuberculosis. He recalls that his older daughter was screened when she started preschool years ago. You explain that recommendations have changed. Which of the following would necessitate screening for tuberculosis with a purified protein derivative (PPD) in this 4 year old?
A. Being born in California
B. Taking a 5 day-long vacation in Mexico
C. Living with an asymptomatic grandparent who is PPD-positive
D. Having a family member with tuberculosis with whom the patient has had no contact
E. Being born in Canada
17. A 14-year-old girl from California’s Central Valley was recently diagnosed with juvenile idiopathic arthritis and is being treated with infliximab. She presents with fatigue, weight loss, and low-grade fevers over the past 2 weeks and now is in the emergency room with worsening fever, cough, and chest pain. She rapidly develops respiratory failure. Her complement fixation titer for coccidioidomycosis is >1:16. Which of the following statements is correct regarding coccidioidomycosis?
A. The most common presentation is meningitis.
B. Children younger than 1 year are usually protected by maternal antibody and have low risk of disease.
C. Impairment of T-lymphocyte function increases risk for severe primary infection, disseminated disease, or relapse of disease.
D. In addition to the Southwestern United States, endemic areas for coccidioidomycosis include the Mississippi River Valley.
E. Empiric therapy should be initiated with dicloxacillin.
18. A 7-year-old boy has just returned from a month long visit with his grandparents in the Midwest. He presents to the emergency room with complaints of headache and a fever with a temperature of 103.5°F. You note scattered petechiae on his wrists and ankles, and he appears unwell. Which of the following will be the most appropriate treatment?
A. Intravenous (IV) doxycycline
B. IV ceftriaxone
C. Amoxicillin–clavulanate
D. Supportive care
E. IV ceftriaxone and IV doxycycline
19. A 2-year-old boy is being evaluated for failure to thrive. He was adopted, and his birth history is unknown. You are planning on testing him for human immunodeficiency virus (HIV) infection and are reviewing common infections and the clinical presentation of children with HIV infection. In addition to failure to thrive, which of the following is a typical complication for a child with perinatally acquired HIV?
A. Cytomegalovirus (CMV) retinitis
B. Recurrent common bacterial infections
C. Central nervous system (CNS) lymphoma
D. Shingles (reactivation of varicella zoster virus)
E. Kaposi sarcoma

20. A 6-year-old girl presents with a 3-day history of bloody diarrhea and dehydration. She has decreased urine output and appears very pale. You plan to admit her to the hospital for management. Which of the following should be part of her management?
A. Administration of trimethoprim/sulfamethoxazole (TMP/SMX) for probable infection with Shigella
B. Stool sample for Giardia intestinalis–specific enzyme immunoassay (EIA)
C. Examination of a peripheral smear for evidence of microangiopathic hemolytic anemia
D. Administration of metronidazole empirically for the treatment of Clostridium difficile
E. Rehydration with half normal saline (½ NS) with 20 mEq of KCl
21. During a well-child examination for a 12-year-old female, her mother refuses the human papilloma virus (HPV) vaccine, stating that her daughter “does not need the vaccine now because she is not going to be sexually active for a very long time.” Which of the following statements about HPV and the vaccine is true?
A. HPV is a rare cause of sexually transmitted disease in the United States.
B. Vaccination during the preteen years is recommended to facilitate the development of an immune response before the initiation of sexual activity.
C. HPV serotypes have been associated with cancers of the female reproductive tract but have not been associated with cancers in male patients.
D. The HPV vaccine is universally recommended for females and is optional for males.
E. Although recommended during the preteen years, HPV vaccine may be administered to sexually active individuals regardless of age.
22. A 2-week-old female infant is being fed both breast milk and infant formula. Which one of the following statements about vitamin D supplementation is correct?
A. Vitamin D supplementation is not necessary as long as the mother continues supplementing with formula.
B. The daily recommended intake of vitamin D in the first year of life is 200 IU.
C. Early introduction of cow’s milk can replace the need for vitamin D supplementation.
D. Vitamin D supplementation in the breastfed infant is not necessary as long as the mother takes adequate amounts of vitamin D and calcium in her diet.
E. Vitamin D supplementation is recommended for both breastfed and formula-fed young infants.
23. A 12-month-old female infant who was born at term with a birth weight of 3.5 kg presents now for a well-child visit. Her current weight is 8 kg, and her head circumference has increased by 12 cm since birth. Hemoglobin screening is 12.5 g/dL, and lead screening is
<3 µg/dL. She is taking her first steps and says “mama,” “dada,” and “baba,” which parents understand to be her bottle. Which of the following best describes how you will focus your discussion with her parents during this visit?
A. Discuss routine anticipatory guidance because surveillance of her growth, development, and screening laboratory findings did not reveal concerning findings.
B. Discuss starting ferrous sulfate for abnormal hemoglobin screen with a concern for iron- deficiency anemia.
C. Discuss her abnormal weight gain and obtain a more detailed history to assess for failure to thrive.
D. Discuss her abnormal head growth since birth and consider a diagnostic workup.
E. Discuss her expressive language delay and recommend doing more reading with her.

Questions 130–134: You are on your way down to the emergency room to evaluate an 8-year-old with fever, headache, and emesis. You are worried that this child could have meningitis and are prepared to perform a lumbar puncture. Match each diagnosis in questions 130–134 with the mostly likely cerebrospinal fluid (CSF) profile from the choices below:

1. Enteroviral meningitis
2. Normal CSF profile
3. Tuberculous meningitis
4. Partially treated bacterial meningitis
5. Pneumococcal meningitis
6. A 12-month-old boy presents for a well-child checkup. You note that the child’s vaccinations are not up-to-date. When discussing the recommended vaccines for the 12-month visit, the family expresses hesitation because they read on the internet that measles, mumps, and rubella (MMR) vaccine may cause autism. As you counsel the parents about vaccine administration, which of the following is true?
A. Live attenuated vaccines such as MMR and varicella zoster virus (VZV) can cause symptoms such as fever and rash 1–2 weeks after immunization.
B. Numerous reputable studies have shown an association between vaccines and autism.
C. Thimerosal-containing vaccines are associated with an increased risk of autism, but routine childhood vaccines no longer contain thimerosal.
D. A mild febrile illness is a contraindication to immunization.
E. The MMR vaccine can be safely administered to children with compromised immunity.
7. A 4-year-old male child presents with a 1-week history of increasing facial edema and 4 days of “swollen feet” following an upper respiratory infection. The parents report that he is increasingly fussy with decreased appetite. Past medical history is unremarkable, and family history is noncontributory. Physical examination is notable for an irritable child with a temperature of 39°C. He has significant periorbital and tibial edema with abdominal fullness. Which of the following statements regarding the patient’s likely condition, evaluation, or management are correct?
A. This patient is predisposed to bleeding.
B. This patient’s urinalysis is likely to include red blood cell (RBC) casts.
C. This patient is likely to have been infected with a toxin-producing bacteria.
D. Blood cultures should be obtained, and the patient should be started on antibiotics, given an increased risk of developing spontaneous bacterial peritonitis.
E. This patient is at increased risk of developing hearing loss.
8. A 2-year-old male child is brought in by his parents for itchy, scabbed papules on the ventral wrists and around the waistline. On further questioning, you determine that the mother and father both have itchy bumps on their skin too. What would be the next best step?
A. No treatment is needed as this condition will resolve.
B. Prescribe a topical steroid for everyone in the home.
C. Prescribe permethrin 5% cream to be applied overnight to the skin (for all family members) and repeat the treatment in 1 week.
D. Recommend that they take diphenhydramine to control itching.
E. Treat the papules with cantharidin.
9. A 30-month-old girl was recently diagnosed with autism spectrum disorder. She is presently receiving speech and occupational therapy once a week. The diagnostic team recommended that she receive additional intensive, evidence-based intervention. Of the following, which is the intervention with the strongest evidence to support its use in this case?
A. Applied behavior analysis
B. Auditory integration therapy
C. Developmental play therapy
D. Sensory integration therapy
E. Iron chelation
10. An 18-month-old girl presents with an erythematous papular rash that affects the elbows, knees, and cheeks and was preceded by a mild upper respiratory infection. Her mother is

concerned because this has been ongoing for over 2 weeks. What is the most likely diagnosis in this case?
A. Roseola
B. Pityriasis rosea
C. Gianotti–Crosti syndrome
D. Varicella
E. Fifth disease (erythema infectiosum)
11. An otherwise healthy sexually active 17-year-old girl comes in to your office for her annual health maintenance examination. She has had two male lifetime partners, occasionally uses condoms, and is not interested in birth control because she just broke up with her boyfriend. Her last menstrual period was 1 month ago. She currently denies abdominal pain, dysuria, and vaginal discharge. Which of the following screening tests should be performed?
A. Vaginal swab for Trichomonas vaginalis
B. Vaginal swab for bacterial vaginosis
C. Urethral swab for herpes simplex virus
D. Urine nucleic acid amplification test for gonorrhea and chlamydia
E. Cervical cytology for human papilloma virus
12. A 6-week-old female infant presents with increased tearing noted from her right eye. There is no color to the eye discharge, and conjunctival erythema and swelling are absent. The infant is without fever and, on examination, has a normal red reflex in each eye and normal extraocular movements. What is your next step in evaluation and management of this infant?
A. Begin topical erythromycin ointment.
B. Begin oral erythromycin for suspected Chlamydia trachomatis infection.
C. Refer to ophthalmology for probing of the nasolacrimal duct.
D. Inform parents that this condition is unlikely to resolve without treatment.
E. Observation only

WBC Glucose Protein Gram Stain
CSF Profile A 10,000 15 120 Gram-positive diplococci
CSF Profile B 6 50 35 Negative
CSF Profile C 240 55 45 Negative
CSF Profile D 145 55 180 Negative
CSF Profile E 5600 20 120 Negative
CSF = cerebrospinal fluid; WBC = white blood cell.
The response options for statements 142 and 143 are the same. You will be required to select one answer for each statement in the following set.

A. Quantitative measurement of serum immunoglobulins
B. Dihydrorhodamine (DHR) flow cytometry test
C. Complete blood count with differential
D. Total hemolytic complement (CH50)
E. T-cell subsets

For each patient, select the best initial screening test that may help lead to a diagnosis.

1. A 4-year-old girl with a history of recurrent skin abscesses, osteomyelitis, and pneumonia due to Aspergillus.
2. A 6-year-old boy with disseminated infection due to Neisseria meningitides.
3. A 6-year-old boy has hives, lip swelling, vomiting, and shortness of breath after being stung by an insect that his mother identifies as a honeybee. What is the next best step in management of this child once he recovers from this reaction?
A. Prescribe oral diphenhydramine and develop an emergency plan for any future

reactions.
B. Prescribe oral corticosteroids and advise the parents to have him avoid wearing bright clothing and scented lotion in the future.
C. Prescribe intramuscular epinephrine, review an emergency plan, and refer him to an allergist for possible venom immunotherapy.
D. Teach him how to appropriately remove an insect stinger and provide reassurance that this is likely not an allergic reaction.
E. Discuss management of local reactions to insect stings, including cool compresses and topical steroids.
4. During the evaluation of a 17-year-old boy for a preparticipation sports physical for football, you note asymmetry of his testes. His sexual maturity rating is 5 for both pubic hair and genital development. His left scrotum is nontender, appears larger when he is standing, and diminishes in size when supine. On palpation, the superior portion of the left testicle feels like a “bag of worms.” The right testicle is uniformly soft and smooth to palpation. He denies sexual activity. Which of the following statements is correct?
A. The adolescent should be referred to a urologist for surgical intervention.
B. The adolescent should be reassured that no intervention is needed at this time.
C. A urine nucleic acid amplification test should be sent for Chlamydia trachomatis and
Neisseria gonorrhoeae.
D. Transillumination of the scrotum will reveal a cystic mass.
E. An additional physical examination finding for this condition includes painless inguinal swelling.
5. You are meeting with the family of a 4-year-old boy who has attention deficit/hyperactivity disorder, combined type, to discuss treatment options. His parents are interested in treatment and would like to know which options are available. Of the following treatments, which would you first recommend to the parents?
A. Trial of nonstimulant medication
B. Organizational skills training
C. Behavioral management training with parents
D. Play therapy
E. Trial of stimulant medication
6. A 7-year-old girl is brought in by her parents for a 3–4 month history of thickened, yellow fourth and fifth toenails with pink scaling skin between the toes. Which would be the best treatment for this problem?
A. Oral terbenafine
B. High-potency topical steroids
C. Topical clotrimazole
D. Surgical removal of the affected nails
E. Topical vitamin D analogues
7. A 16-year-old boy is brought to your office by his mother because she is concerned he may be depressed. The adolescent seems to have become more withdrawn over the previous 3 weeks and is sleeping more than usual. This is affecting his ability to attend school. Of the following, what additional symptoms would you expect the boy to display for a diagnosis of major depression?
A. Cough from the initiation of tobacco use
B. Decrease in his appetite
C. Increase in extracurricular activities
D. Epigastric abdominal pain
E. Headache with visual aura
8. A 2-year-old boy is brought into the emergency department by paramedics. He was found

hiding in a closet within his apartment during a fire in his building. You notice that he appears quiet but with increased work of breathing. He has soot around his nares and singed nasal hairs, and it appears that his lips are cherry red. What is the next step in management of this child?
A. Intubate with rapid sequence induction.
B. Obtain a stat chest x-ray.
C. Obtain an arterial blood gas.
D. Arrange for transfer to a center that provides hyperbaric oxygen.
E. Determine body surface area (BSA) burned.
9. You are asked to counsel new parents of a male infant who are very anxious because they lost their first child to sudden infant death syndrome (SIDS). Your recommendation to the parents would include which of the following?
A. Recommend that the infant sleep in bed with parents.
B. Avoid any tobacco smoke in the home.
C. Prescribe a cardiorespiratory monitor to prevent SIDS.
D. Put the infant to sleep in the prone position.
E. Ensure the room is very warm to encourage rest and sleep.

Answers and Explanations
1. The answer is A [, section X.E.4, Figures 4-3 and 4-4]. The differential diagnosis of direct (conjugated) hyperbilirubinemia includes obstructive jaundice secondary to a choledochal cyst, obstructive jaundice secondary to biliary atresia, neonatal hepatitis, cystic fibrosis, and inborn errors of metabolism. Hepatic ultrasound is able to diagnose choledochal cyst and is also useful in the evaluation of biliary atresia. Sepsis (diagnosed with a blood culture) should be considered in the differential diagnosis of both direct and indirect hyperbilirubinemia; however, it is very unlikely in an otherwise well-appearing infant who is alert and feeding well. ABO incompatibility (diagnosed with Coombs testing), hypothyroidism (diagnosed with thyroid hormone studies), and hemolytic disorders such as hereditary spherocytosis (diagnosed with an osmotic fragility test) all are causes of indirect (unconjugated) hyperbilirubinemia.
2. The answer is E [, section III.E]. There are three major contraindications to administering the diphtheria, tetanus, and pertussis (DTaP) vaccine. These contraindications include a severe allergic reaction (i.e., anaphylaxis) after a previous vaccine dose or to a vaccine component, encephalopathy within 7 days of administration of a previous dose of DTaP, and a progressive neurologic disorder, which includes progressive encephalopathy and uncontrolled epilepsy. A mild respiratory illness with a low-grade fever is not a contraindication to administering the DTaP vaccine, nor is a history of moderate swelling at the injection site after a previous dose. Administration of intravenous immune globulin within the preceding 3–11 months is a precaution to immunization with live virus vaccines, such as measles, mumps, and rubella (MMR) and varicella. Immunocompromised status or the presence of an immunocompromised sibling in the household is not a contraindication to administration of DTaP.
3. The answer is A [, section V.D]. This child’s presentation is most consistent with dermatomyositis. Characteristic cutaneous manifestations include a periorbital heliotrope rash that may cross the nasal bridge and an erythematous and hypertrophic rash over the metacarpal and proximal interphalangeal joints (Gottron papules). Characteristic laboratory and diagnostic studies include abnormal electromyography findings, abnormal muscle biopsy findings, and increased muscle enzymes, including creatine kinase. A positive antinuclear antibody is not specific and would be more suggestive of systemic lupus erythematosus (SLE) or juvenile idiopathic arthritis (JIA), although this patient fails to meet 4 of the 11 diagnostic criteria required for the diagnosis of SLE and has no arthritis, which is characteristic of JIA. Although both Epstein–Barr virus infection and cytomegalovirus infection may present with chronic fatigue, neither infection would present with the characteristic rashes described in this patient.
4. The answer is B [, Table 20-1]. The Glasgow coma score (GCS) is used to assess a patient’s level of neurologic impairment. It is based on the physical examination and includes assessment for motor response, verbal response, and eye opening. In a young child, a modified scoring system can be used, which has been adapted for the nonverbal patient. The patient described has a GCS score of 5 because he does not open his eyes to pain or to voice (scored 1), has decerebrate posturing (scored 2), and moans with stimulation with incomprehensible words (scored 2). Because his GCS is <8, he has likely suffered severe head injury.
5. The answer is B [, section II.A]. The most common presenting symptoms of acute lymphocytic leukemia (ALL) are fever and bone or joint pain, as in this boy. The peak incidence of ALL is between 2 and 6 years of age, and ALL is more common in males. Pallor, hepatosplenomegaly, and lymphadenopathy are among the most common presenting findings on physical

examination. The white blood cell count varies on presentation and is elevated in only one- third of patients.
6. The answer is A [, section IV.B.2]. Hemophilia A, a deficiency of factor VIII, is an X-linked inherited disorder that occurs only in males. Typical features include deep soft-tissue bleeding and hemarthroses. Laboratory findings include a prolonged aPTT (activated partial thromboplastin time) in all but very mild cases, but prothrombin time and bleeding time are normal. Management includes replacement of factor VIII, which is not a vitamin K–dependent coagulation factor.
7. The answer is C [, section VI]. This patient’s presentation is consistent with nephrotic syndrome, which is characterized by heavy proteinuria, hypoalbuminemia, hypercholesterolemia, and edema. Most children present with edema, which can range from mild periorbital edema to scrotal or labial edema and to widespread edema; this edema often follows an upper respiratory infection. Patients with nephrotic syndrome are at an increased risk for infection with encapsulated organisms, such as Streptococcus pneumoniae. Because of this risk, if a patient with active nephrotic syndrome becomes febrile, evaluation should include a blood culture, urine culture, and chest radiograph, and broad-spectrum empiric intravenous antibiotics should be initiated immediately. Patients with nephrotic syndrome are predisposed to thrombosis (stroke, renal vein thrombosis, deep vein thrombosis, and sagittal sinus thrombosis) secondary to hypercoagulability. Renal biopsy is rarely indicated in a child with nephrotic syndrome, unless the creatinine clearance is impaired or initial management with corticosteroids is ineffective. Most patients with nephrotic syndrome respond to corticosteroid therapy.
8. The answer is C [, section VII.B.1.b]. Bacterial vaginosis is the most common cause of vaginitis in adolescents. It results from a change in the patient’s normal vaginal flora secondary to a reduction in lactobacilli. It presents with gray-white, malodorous discharge, a fishy odor, and little vaginal or vulvar inflammation. Wet-mount saline preparation demonstrates the presence of “clue cells.” Bacterial vaginosis is not thought to be transmitted sexually, so partners do not need treatment. Neither antibiotics nor antiyeast medications are indicated.
9. The answer is D [, section V.C and Table 8-6]. This clinical presentation is consistent with bacterial endocarditis. Ophthalmologic examination is warranted to evaluate for Roth spots and retinal hemorrhages, which can help confirm the diagnosis. Diagnosis is often made on confirmation of vegetations by echocardiography. Transesophageal echocardiography is more sensitive than transthoracic echocardiography in detecting vegetations. Rheumatoid factor and other acute-phase reactants, as well as the white blood cell count and the erythrocyte sedimentation rate, are often elevated. Gram-positive cocci, including Streptococcus and Staphylococcus species, are the most common infecting organisms. Gram-negative organisms are uncommon causes of endocarditis. Management includes appropriate intravenous antibiotics, but antibiotics can be safely withheld until at least three blood cultures are obtained in an attempt to identify the infecting organism and to use appropriate antibiotic therapy.
10. The answer is C [, section X.E]. Myasthenia gravis is an immune-mediated condition caused by antibodies that form against the acetylcholine receptor at neuromuscular junctions. Most patients first come to medical attention with eye problems, especially bilateral ptosis. One of the hallmarks of patients with myasthenia gravis is that their weakness progresses as the day progresses. Fasciculations (i.e., spontaneous, small twitches of muscle fibers caused by denervation) are commonly seen in Guillain–Barré syndrome but not in myasthenia gravis. Botulism is associated with honey ingestion and would more likely present acutely with constipation followed by weakness. Deep tendon reflexes are normal in myasthenia gravis. The Tensilon test is a useful test that can confirm the diagnosis of myasthenia gravis. It is performed by intravenously injecting edrophonium chloride, a rapidly acting cholinesterase

inhibitor, which produces transient improvement of ptosis.
11. The answer is E [, Table 7-2, sections III.B, and III.C]. The majority of children with fever of unknown origin (FUO), defined as unexplained fever lasting longer than 8 days to 3 weeks, have a common infection with an unusual presentation. Infectious diseases are the most common causes of FUO, whereas rheumatologic causes (e.g., Kawasaki disease, systemic juvenile idiopathic arthritis) are the second most common cause of FUO. Hospitalization is often indicated to facilitate the evaluation and to document the fever and identify any coexisting signs and symptoms. A cause is identified in approximately 75% of patients after an extensive workup; no cause is identified in 25% of children, despite a thorough evaluation.
12. The answer is C [, section VI.C.1]. The most frequent cause of respiratory distress in preterm infants is respiratory distress syndrome (RDS) (also known as hyaline membrane disease or surfactant deficiency syndrome), which occurs in as many as 50% of newborns born before 30 weeks’ gestation (0.5% of all neonates). Meconium aspiration syndrome, neonatal sepsis, and intraventricular hemorrhage may all present with respiratory distress, but these conditions occur less frequently than RDS. Bronchopulmonary dysplasia is a chronic complication of RDS and does not occur in the immediate newborn period.
13. The answer is B [, section II.D.6]. The patient’s disorder, short bowel syndrome, occurs after surgical resection of small bowel or damage to the small intestine from Crohn disease or from radiation. The loss of small bowel reduces the absorptive capacity of the gut, leading to malabsorption and electrolyte abnormalities. Total parenteral nutrition (TPN) is commonly used to maintain adequate nutrition; however, TPN causes cholestatic liver disease and, in many patients, gallstones. Other complications of this disorder include bacterial overgrowth within the remaining intestine, nutritional deficiencies, poor bone mineralization, renal stones, and secretory diarrhea. Patients on TPN or those with short bowel syndrome generally do not have gastrointestinal bleeding, constipation, pancreatic insufficiency, or inflammatory bowel disease.
14. The answer is D [, section V.A.4.b, Table 17-6, and Figure 17-5]. A physeal fracture that extends through the physis and into both the epiphysis and metaphysis is a grade IV Salter– Harris fracture. A grade I fracture is within the physis. A grade II fracture is within the physis and extends into the metaphysis. A grade III fracture is in the physis and extends into the epiphysis. A grade V fracture describes a crush injury of the physis.
15. The answer is D [, section II.B.2.d.(2)]. The patient described has craniosynostosis, a premature closure of a cranial suture. Eighty to ninety percent of cases of craniosynostosis are sporadic in occurrence with an unknown cause. Ten to twenty percent of cases are familial or are a component of a genetic syndrome (e.g., Apert syndrome, Crouzon syndrome). Sleeping on the back and congenital muscular torticollis may cause plagiocephaly, which is skull asymmetry without premature suture closure. Perinatal asphyxia causes microcephaly as a result of inadequate brain growth, rather than premature suture closure.
16. The answers are B and C, respectively [, section I.K]. Sturge–Weber syndrome is a neurocutaneous syndrome characterized by a nevus flammeus (“port wine stain”) in the area innervated by the ophthalmic branch of the trigeminal nerve (V-I). It is associated with seizures, intracranial and intraspinal malformations, and intracranial calcifications (question 16). Erythema toxicum neonatorum is a benign rash of infancy characterized by erythematous macules, papules, and pustules on the trunk and extremities. The lesions are filled with eosinophils, and no treatment is required (question 17).
17. The answer is D [, section VI.A]. Patients with homocystinuria are hypercoagulable due to increased homocysteine levels. The tendency toward thromboembolism leads to significant morbidity, including deep vein thrombosis, myocardial infarction, and stroke. Betaine reduces homocysteine levels and greatly decreases the risk of thromboembolic events. Propranolol, a
β-blocker, is useful for patients at risk for aortic dissection, such as patients with Marfan

syndrome. Vitamin C may be useful in transient tyrosinemia of the newborn as it can assist in tyrosine elimination. d-Penicillamine increases the clearance of copper and is useful in Wilson disease. Zinc is useful for children who have problems with healing, such as patients with acrodermatitis enteropathica. Propranolol, vitamin C, d-penicillamine, and zinc do not play useful roles in the management of patients with homocystinuria.
18. The answers are E, C, and B, respectively [, sections II.B.2.d.(3).(c).1 and II.C.3.f; , sections
IV.E.2 and IV.C.1]. McCune–Albright syndrome (question 19) is characterized by endocrine dysfunction (peripheral precocious precocity, hyperthyroidism), irregularly bordered hyperpigmented macules (“coast of Maine” spots), and fibrous dysplasia of the bones that may lead to fractures. Kallmann syndrome (question 20), a form of hypogonadotropic hypogonadism, is associated with anosmia (absent sense of smell). Prader–Willi syndrome (question 21) is characterized by hypotonia, hypogonadism, and small hands and feet, with growth problems in the first year of life (secondary to feeding problems), followed by hyperphagia and obesity later in childhood. Turner syndrome is characterized by ovarian dysgenesis, short stature, webbing of the neck, and left-sided congenital heart disease, with an increased incidence of hypothyroidism. Laurence–Moon–Biedl syndrome is characterized by hypogonadism, retinitis pigmentosa, obesity, and polysyndactyly.
19. The answer is B [, section VI.B.2; , section V.G]. Findings consistent with child abuse include specific types of fractures, such as metaphyseal (“corner” or “bucket handle”) fractures. These fractures occur as a result of torsional force placed on an extremity (i.e., pulling and twisting) or from violent shaking. Other fractures highly suggestive of abuse include posterior and first rib fractures; fractures of the sternum, scapula, and vertebral spinous processes; and fractures in different stages of healing. Bruises on exposed areas, such as the knees, shins, elbows, and forehead, are common during childhood and are typical of accidental injury. Burns that have an irregular splashlike configuration are generally accidental. Toddler’s fracture (i.e., spiral fracture of the distal tibia) can occur with little trauma, such as a fall when running and alone are not suggestive of abuse. Supracondylar fractures are common during childhood and usually occur when a child falls on an outstretched arm and alone are not suggestive of abuse.
20. The answer is B [, section IV.A.7 and Table 9-5]. Asthma is the most likely diagnosis because of the symptoms of recurrent cough and wheezing. This patient likely has persistent asthma, given the frequency and persistence of symptoms. Because asthma is an inflammatory disorder, anti-inflammatory medications, such as cromolyn sodium or inhaled corticosteroids, are recommended for persistent asthma symptoms. In as many as 50% of patients with asthma, symptoms may develop during the first year of life. Smoking should not be allowed in any place that a child with asthma may visit, including the home and automobile. Passive smoke exposure is an airway irritant and a trigger of asthma. Although albuterol would be useful for the management of acute symptoms, it has no effective role in prevention, except for preventing symptoms of exercise-induced asthma. Foreign body aspiration is less likely in a child with both persistent and recurrent symptoms. In addition, inspiratory and expiratory films could not be obtained in an 11-month-old infant; in this case decubitus films would be indicated if foreign body aspiration was suspected.
21. The answer is D [, section I.A.1]. The most common cause of cardiac arrest in a child is a lack of oxygen supply to the heart. Respiratory problems that result in a lack of oxygen supply include choking, airway disease, lung disease, suffocation, and brain injury. The end result of any of these processes is decreased oxygen tension within the blood (hypoxia). Cardiac arrest can often be prevented if assisted breathing is administered rapidly. Heart disease is an uncommon cause of cardiac arrest during childhood. Seizures, poisonings, and trauma are also less common causes of cardiac arrest than is hypoxia.
22. The answer is D [, section VII.B]. Peripheral tissue resistance to insulin is seen in type 2 diabetes mellitus (DM). All of the other choices listed are more commonly associated with

type 1 DM. The evolution of type 1 DM is thought to have numerous causes. It begins with a genetic predisposition (human leukocyte antigen [HLA] haplotypes DR3/DR4), a viral infection acting as an environmental triggering event, and lastly an autoimmune process that ultimately results in β-cell destruction. Autoimmune antibodies, such as islet cell and insulin antibodies, may be found in type 1 DM.
23. The answer is B [, section II.C.2–3]. Patients with celiac disease may present with vomiting, bloating, foul-smelling stools, or failure to thrive. Infants also may be irritable. This infant developed his symptoms after the introduction of wheat cereal that contains gluten. Gluten is found in wheat, rye, barley, and oats (if the oats are harvested in fields that also grow wheat). Celiac disease is a gluten-sensitive enteropathy in which antibodies to gluten cross-react and damage the mucosa of the small intestine, which results in flat, atrophic villi. Lactase deficiency causes bloating and watery stools but not failure to gain weight. Crohn disease is uncommon during infancy. Gastroesophageal reflux disease may be associated with vomiting and irritability; however, stools are not foul-smelling. Cow’s milk protein intolerance may be associated with diarrhea, irritability, and vomiting, but stool blood and mucus are often present, and symptoms often begin before 6 months of age.
24. The answer is B [, section IV.D.2]. The 100% oxygen test helps distinguish whether cyanosis is caused by cardiac or respiratory disease. Patients with cyanotic congenital heart disease associated with reduced pulmonary blood flow, such as tetralogy of Fallot, do not respond with any increase of significance in the PaO2 level when given 100% oxygen. In contrast, infants with cyanotic congenital heart disease associated with normal or increased pulmonary blood flow, such as truncus arteriosus, may have some increase in the PaO2 level, but not as much of an increase as seen in patients with primary pulmonary disease. Patients with primary lung disease, such as neonatal pneumonia, respiratory distress syndrome, or meconium aspiration syndrome, demonstrate a very significant increase in the PaO2 levels when administered 100% oxygen.
25. The answer is D [, section II.G]. This patient presents with diagnostic criteria consistent with Kawasaki disease, and treatment includes high-dose intravenous immune globulin together with high-dose aspirin therapy. Oral penicillin is not an appropriate course of management in this patient because there is no evidence of the fine sandpaper-like rash consistent with scarlet fever, a sequela of group A β-hemolytic streptococcal infection. Oral dicloxacillin can be used for suspected acute bacterial cervical adenitis; however, the lack of improvement on amoxicillin–clavulanic acid and the other findings on physical examination make cervical adenitis alone less likely. Admission for an evaluation for invasive bacterial disease (rule-out sepsis workup) is not indicated because this patient is neither toxic-appearing nor has meningeal signs. The presentation is also not consistent with high fever without a source, which would require empiric broad-spectrum antibiotic coverage. Steroids are controversial and currently not routinely indicated for the initial treatment of Kawasaki disease.
26. The answers are B and A, respectively [, section V.F]. Cardiomyopathy may be categorized as dilated, hypertrophic, or restrictive. Hypertrophic cardiomyopathy (question 29) may be inherited in an autosomal dominant manner or may occur in infants of diabetic mothers. Infants of diabetic mothers have transient septal hypertrophy and have a systolic ejection murmur because of obstruction of left ventricular outflow. Causes of dilated cardiomyopathy (question 30) include viral myocarditis, carnitine deficiency, anomalous origin of the left coronary artery, and hypocalcemia. Anomalous origin of the left coronary artery from the pulmonary artery may present with evidence of myocardial infarction. As a result of infarction, cardiac function may be impaired, and an echocardiogram shows a dilated ventricle with poor ventricular function. Causes of restrictive cardiomyopathy include inherited infiltrative disorders (such as hemochromatosis and Gaucher disease) and amyloidosis.

27. The answer is E [, section X.C.2]. Melena, a term used to describe dark and tarry stools, is most often caused by upper gastrointestinal (GI) bleeding proximal to the ligament of Treitz. The most common causes of melena in a 3-year-old girl include gastritis, ulcers, swallowed blood from epistaxis, mechanical injury to the mucosa from vomiting (Mallory–Weiss tear) or a foreign body, and varices. To best evaluate the cause of upper GI bleeding, upper intestinal endoscopy is used to visualize the source of bleeding and, if indicated, to perform biopsies and treat active bleeding. Computed tomography of the abdomen is not useful in the diagnosis of upper GI bleeding. A Meckel scan and colonoscopy have more utility in the investigation of lower, rather than upper, GI bleeding. Plain film radiography is generally not useful in diagnosis unless a foreign body or intestinal perforation is suspected.
28. The answer is B [, section VII]. This patient’s clinical presentation of a prodrome of bloody diarrhea followed by the onset of anemia, thrombocytopenia, and acute renal failure is most consistent with hemolytic uremic syndrome (HUS). The most common subtype of HUS seen in childhood is Shiga toxin–associated HUS resulting from an intestinal infection with a toxin- producing bacteria. The treatment of Shiga toxin–associated HUS is supportive, and antibiotics are not indicated. Vascular endothelial injury by the Shiga toxin is the key to the pathogenesis of injury in Shiga toxin–associated HUS. The toxin binds to endothelial cells, causing endothelial cell injury, especially in the renal vasculature, leading to platelet thrombi formation and renal ischemia. Although the patient’s bloody diarrhea may have contributed to his anemia, patients with HUS have hemolytic anemia. The prognosis for children with Shiga toxin–associated HUS is usually favorable but depends on the severity of the initial episode. In North America, the most common pathogen associated with Shiga toxin–associated HUS is Escherichia coli O157:H7.
29. The answer is D [, section I.D.2.d.(2)]. Although iron-deficiency anemia is the most common cause of anemia during childhood, other causes of anemia should be considered in patients who do not respond to iron therapy for suspected iron-deficiency anemia. In this case, this patient may have β-thalassemia minor (trait), a heterozygous condition that causes mild asymptomatic anemia. Because she did not respond to iron, she should undergo further testing, including an hemoglobin (Hgb) electrophoresis, blood smear, lead level, and an iron level (which is normal or elevated in β-thalassemia minor). Continuing to treat her with iron may not be beneficial at this point. Red blood cell transfusion is not indicated for this Hgb level. Folic acid is not helpful or indicated; folic acid deficiency causes a macrocytic anemia.
30. The answer is A [, section III.D]. This patient is presenting with symptoms of physiologic gastroesophageal reflux, a finding found in up to 60% of infants. Gastroesophageal reflux does not need workup, change in the diet, or medical treatment. The spitting up is benign and resolves with time. Given her normal weight gain and the absence of pain, she does not have gastroesophageal reflux disease (GERD). GERD in some cases merits further evaluation with a barium upper gastrointestinal study to evaluate the anatomy of the stomach, esophagus, and duodenum and a pH probe to confirm the presence of reflux.
31. The answer is E [, section VI.E]. Avulsed primary teeth do not require reimplantation. In the case of an avulsed secondary (permanent) tooth, extraoral time is the most important prognostic factor for successful reimplantation. A tooth that has been stored dry, even after only 30 minutes, has a very poor prognosis. Management of an avulsed secondary tooth includes gentle rinsing with saline or water, placement of the avulsed tooth into the tooth socket or into a liquid such as milk, and emergent referral to a dentist.
32. The answer is E [, section I.C]. The constellation of a nonthrombocytopenic petechial eruption on the thighs and a limp (arthritis) in a toddler, along with upper respiratory symptoms, is most consistent with the diagnosis of Henoch–Schönlein purpura (HSP). The renal manifestations of HSP are variable, ranging from mild hematuria or proteinuria to end-stage renal disease in 1% of patients. Classically, the renal manifestations of HSP may not become

clinically apparent for up to several months after the patient’s initial presentation. HSP is a multiorgan system vasculitis associated with increased serum IgA levels. Recurrences of HSP are quite common, occurring in approximately 50% of patients. Steroids are the mainstay of therapy for severe abdominal pain. The rash of HSP characteristically appears on the lower extremities and buttocks, not the palms and soles.
33. The answer is A [, section IX.B]. The most important initial treatment for any bite is local wound care, which includes copious and prompt wound irrigation. Dog bites may become infected, especially bites to the hand, wrist, and foot and bites that are small puncture wounds, as in this patient. Large wounds, wounds to the face, and wounds less than 12 hours old should all be sutured. However, wounds greater than 12 hours old should be allowed to close on their own (except facial wounds, which may still be safely closed up to 24 hours later as a result of the high vascularity of the face). Aerobic and anaerobic organisms, including Staphylococcus aureus and Pasteurella multocida, are the usual pathogens causing infection. Bartonella henselae is not a usual pathogen in dog bites but rather is a cause of cat scratch disease. Antibiotics should be started empirically, without waiting for clinical signs of infection, because of the high risk of infection. Tetanus prophylaxis is necessary for unimmunized or underimmunized patients who sustain dog bites.
34. The answers are E, C, A, and D, respectively [, Table 18-2 and section III.B.3]. Diagnosis of a red eye can often be made on the basis of history and physical examination. Conjunctivitis that occurs between days 4–10 of life is most likely caused by Chlamydia trachomatis and is best treated with oral erythromycin (question 38). Oral therapy is used to kill Chlamydia organisms in both the conjunctiva and nasopharynx simultaneously. If Chlamydia organisms within the nasopharynx are not killed, pneumonia may develop between 1 and 3 months of age. Burning, crusting, and scales at the eyelash margin suggest blepharitis, a condition treated with baby shampoo scrubs of the eyelashes (question 39). Purulent conjunctivitis in a young child is typically bacterial in origin and would prompt administration of topical antibiotics, such as tobramycin (question 40). Itching is a hallmark of allergy and suggests allergic conjunctivitis. Allergic disease of the eyes is best treated with topical antihistamines or topical mast cell stabilizers (question 41).
35. The answer is C [, section V.D.2–3]. Because the clinical features of viral pharyngitis and group A β-hemolytic streptococcal (GABHS) pharyngitis overlap, children presenting with pharyngitis should undergo laboratory testing to confirm GABHS infection before the institution of antibiotic therapy. Such testings may include rapid antigen testing or a throat culture (the gold standard). Although infectious mononucleosis should be considered in patients with pharyngitis, this patient’s acute presentation is unlikely to be caused by infectious mononucleosis, and therefore, a monospot test is not indicated. Supportive care with analgesics and lozenges is helpful for pharyngitis, but it is not the most important initial management step. A lateral neck radiograph is not indicated given the absence of clinical features consistent with upper airway obstruction.
36. The answer is C [, section II.B–C]. Communicating hydrocephalus is caused by decreased absorption of cerebrospinal fluid (CSF) or, less commonly, by increased CSF production. Meningitis causes inflammation and scarring of the arachnoid membrane, which prevents the normal absorption of CSF and results in communicating hydrocephalus. Brain atrophy does not cause hydrocephalus but rather a condition called hydrocephalus ex vacuo, in which the ventricles are enlarged because of loss of surrounding brain tissue. Chiari type II malformation produces a noncommunicating hydrocephalus caused by blockage of flow in the ventricular system. Aqueductal stenosis may cause noncommunicating hydrocephalus for the same reason. A Dandy–Walker malformation is characterized by an absent or hypoplastic cerebellar vermis and cystic enlargement of the fourth ventricle, which blocks the flow of CSF, thereby causing noncommunicating hydrocephalus.

37. The answers are E and A, respectively [, Table 10-1]. Scurvy (question 44) is caused by vitamin C deficiency and causes symptoms that include hematologic abnormalities, edema, poor wound healing as a result of impaired synthesis of collagen, and a characteristic spongy swelling of the gums. Vitamin A deficiency (question 45) presents with xerophthalmia (dry conjunctiva and cornea) and night blindness.
38. The answer is B [, section VII.B.4]. This patient’s presentation with dysuria and increased urinary frequency are consistent with urethritis. A urinary tract infection is possible, but it is less likely in a male. Because of his sexual activity and the intermittent nature of his condom use, a sexually transmitted disease is most likely. Urethritis may be the result of Neisseria gonorrhoeae or nongonococcal causes, such as Chlamydia trachomatis. Diagnosis may be made presumptively by finding greater than five white blood cells per high-power field on a Gram stain of the patient’s urethral secretions or by finding evidence of pyuria on first morning- voided urine. Urethritis is more common in males than in females. Unlike this patient, many patients have asymptomatic infection. Definitive diagnosis can be made either by culture of the urethral secretions (via a urethral swab) or by nonculture tests on urine or on discharge (direct fluorescent antibody staining, polymerase chain reaction, ligase chain reaction, enzyme-linked immunoassay, or nucleic acid hybridization).
39. The answer is A [, section VIII.B]. On the basis of this patient’s clinical features, parotitis is the most likely diagnosis. Unilateral parotitis is generally caused by an infection with Staphylococcus aureus, Streptococcus pyogenes, or Mycobacterium tuberculosis. Testing for tuberculosis should be considered for patients with unilateral parotid infection. Mumps, Epstein–Barr virus, and cytomegalovirus generally result in bilateral parotitis. Management of bacterial parotitis rarely requires surgical drainage unless there is an abscess.
40. The answer is B [, section IV.B]. The clinical presentation is consistent with tetralogy of Fallot. Children with tetralogy of Fallot may be cyanotic or pink at rest. The degree of cyanosis depends on the amount of resistance to flow through the obstructed pulmonary outflow tract and on the systemic vascular resistance, both of which affect the degree of right-to-left shunting across the ventricular septal defect (VSD). Crying can be a cause of hypercyanotic, or “tet” spells in a child with tetralogy of Fallot. Crying increases the child’s heart rate. The elevated heart rate increases right ventricular outflow tract resistance, which increases right- to-left shunting, causing increasing cyanosis. Patients with transposition of the great arteries, tricuspid atresia without a VSD, and total anomalous pulmonary venous connection are expected to be cyanotic at all times (including at rest). Patients with truncus arteriosus may not only be mildly cyanotic but also commonly manifest signs and symptoms of congestive heart failure, with poor weight gain because of excessive pulmonary blood flow.
41. The answers are C, A, F, E, and D, respectively [, sections V.A.2.1, III.C.3, III.E.3, III.F.2, and III.D.3]. Tinea corporis (ringworm) is a fungal infection characterized by oval or circular patches with central clearing (question 49). Seborrheic dermatitis in an adolescent may present with greasy scales and crusts in the nasolabial folds, beard areas, chest, and scalp (question 50). Psoriasis is characterized by scaling papules and plaques, and lesions may demonstrate the Koebner phenomenon, in which new lesions appear at the site of skin trauma (question 51). Miliaria rubra, or heat rash, occurs in association with heat and occlusive clothing. The sweat irritates the skin, causing pruritic papules, often on the neck, chest, inguinal area, or axilla (question 52). Pityriasis rosea is characterized by a large single scaly erythematous lesion (“herald patch”) that is followed by oval erythematous macules and papules that follow skin lines in a “Christmas tree” distribution (question 53).
42. The answer is D [, section VI.C.1]. Latex allergy is seen in health care workers and in patients with myelomeningocele more frequently than in the general population because of more frequent exposure to latex (patients with myelomeningocele often require intermittent bladder catheterization, which exposes them to latex repeatedly). Latex exposure can cause acute

urticaria and anaphylaxis in patients with latex allergy. Patients with neural tube defects do not have increased risks of developing allergies to penicillin, cow’s milk protein, dust mites, or wool.
43. The answer is B [, section XIII.D.4.c]. This infant most likely has congenital rubella syndrome. Cataracts, congenital heart disease (most often patent ductus arteriosus), and sensorineural hearing loss are the hallmarks of this syndrome. Presenting features may also include thrombocytopenia and extramedullary hematopoiesis manifesting as a “blueberry muffin” appearance to the skin. Congenital toxoplasmosis presents with hydrocephalus, intracranial calcifications, and chorioretinitis. Congenital varicella is uncommon but may present with a dermatomal rash or scarring. Perinatally acquired herpes simplex virus infection presents with a vesicular rash or encephalitis but not with structural defects. Congenital cytomegalovirus infection typically presents with microcephaly, hepatosplenomegaly, and cerebral calcifications and usually not with cataracts or congenital heart disease.
44. The answer is B [, Tables 2-1, 2-3 and 2-4]. A 6-month-old infant should be able to sit with support and may be able to sit independently. Language development in a normal 6-month- old includes babbling, which is placing consonants and vowels together. Another 6- to 7- month milestone is transferring objects from hand to hand. At 4 months of age, infants are learning to bring their hands to midline and to mouth but would not be able to transfer objects. By 8 months of age, infants should be sitting without support. At 9 and 10 months of age, infants should be able to use jargon, which integrates babbling with the intonational patterns, and would have moved beyond sitting with support to crawling, creeping, and pulling up to stand.
45. The answer is C [, section IV.B.5–6]. This patient’s presentation with chronic cough, recurrent pneumonia, and failure to thrive is most consistent with cystic fibrosis. Patients with cystic fibrosis may also present with chronic diarrhea as a result of steatorrhea, and may have a history of meconium ileus at birth. Diagnosis of cystic fibrosis may be confirmed by documenting an elevated sweat chloride level or by documenting mutations in the gene associated with cystic fibrosis. Although a chest radiograph may help identify pneumonia, chest radiographic findings are not specific for cystic fibrosis. Sputum culture is difficult to obtain in a young child, and even if it could be obtained, it would not diagnose her underlying disorder. Foreign body aspiration, diagnosed by decubitus radiographs, should be considered in patients with cough, choking, stridor, and hoarseness, but it would not typically cause failure to thrive. Mycoplasma pneumoniae infection, suggested by elevated cold agglutinins, would not cause failure to thrive, recurrent pneumonia, or cough for 4–5 months.
46. The answers are B, C, and A, respectively [, Table 7-3]. Bacterial meningitis (question 58) is characterized by high white blood cell (WBC) count on cerebrospinal fluid (CSF) analysis, with a polymorphonuclear leukocyte (PMN) predominance, low glucose, and high protein. Tuberculous meningitis (question 59) is characterized by very high protein, low glucose, and a WBC count generally <500 cells/mm3 with a lymphocyte predominance. Viral meningitis (question 60) may also have an early PMN predominance, but the CSF WBC count is usually lower than in bacterial meningitis, and the glucose is normal.
47. The answer is B [, section III.F.4.c]. Chlamydia trachomatis is a common cause of afebrile pneumonia in an infant 1–3 months of age. It presents with a staccato cough, respiratory distress, and a history of conjunctivitis (50%). On examination, patients often have tachypnea and wheezing. Infection with Mycoplasma pneumoniae is more common in older children and adolescents, and it typically presents with a chronic nonproductive cough and headache. Respiratory syncytial virus is a common cause of pneumonia and bronchiolitis during the late fall and winter (November through April). It should be considered in any infant with wheezing; however, upper respiratory symptoms such as rhinorrhea and low-grade fever generally develop before cough and wheezing. Bacterial infection with Streptococcus

pneumoniae can occur in young infants, although they would be expected to have fever. Asthma is a chronic disorder characterized by repeated episodes of wheezing and therefore is not present in a patient of this young age.
48. The answer is E [, section IX.B.1]. This patient’s presentation with sudden onset of scrotal pain, a tender and swollen testicle, and absent cremasteric reflex on the affected side is consistent with torsion of the spermatic cord, which is a urologic emergency. Emergent surgical consultation and intervention are necessary. Detorsion of the torsed spermatic cord must be performed within 6 hours to preserve testicular function, and at the time of the procedure, the opposite testicle is also fixed to the scrotum because it has an increased incidence of torsion in the future. The diagnosis of torsion of the spermatic cord is a clinical diagnosis. Although not always necessary, confirmation of the torsion can be made by finding decreased uptake on radionuclide imaging or absent pulsations on Doppler ultrasound. Elevation of the torsed testicle can relieve pain in some instances; however, this is not a reliable finding in testicular torsion. This patient’s presentation is inconsistent with epididymitis because the entire testicle, not just the epididymis, is tender and swollen.
49. The answer is C [, section IV.B.5]. An abnormal anal wink reflex raises concern that there may be a neurologic reason for enuresis, and imaging studies of the spine and brain should therefore be considered. Family history is frequently positive in uncomplicated nocturnal enuresis. Psychosocial factors such as parental divorce or the birth of a sibling are frequently associated with uncomplicated secondary enuresis, which does not require imaging studies if the neurologic examination is normal. Constipation is a frequent comorbid factor that should be treated because it may cause impingement on the bladder. Children with small bladder capacities often present with a history of frequent small voids and can sometimes be cured through bladder stretching exercises.
50. The answers are D, E, and B, respectively [, Table 13-3]. Most patients with sickle cell disease have one or more crises during their lifetime. A hyperhemolytic crisis (question 64) is characterized by rapid hemolysis that leads to anemia. Patients present with fatigue, pallor, and jaundice. Laboratory evaluation reveals decreased hemoglobin (Hgb), elevated bilirubin, and an elevated reticulocyte count. An aplastic crisis (question 65) is characterized by cessation of red blood cell (RBC) production, most commonly as a result of infection with parvovirus B19. Patients present with pallor and fatigue. Laboratory evaluation reveals decreased Hgb and decreased reticulocyte count. Treatment is transfusion of RBCs. Sequestration crisis (question 66) is characterized by the acute accumulation of blood within the spleen or liver. Patients may present with pallor, fatigue, shortness of breath, and shock. Laboratory findings include a low Hgb and very elevated reticulocyte count. Urgent transfusion of RBCs is indicated.
51. The answer is C [, Table 2-1]. When 4-month-old infants lie on their stomach, they can push up onto their elbows and look around with their head held up. When 4-month-old infants are pulled from a supine to a sitting position, they should also be able to keep their head even with their body. Two-month-old infants have head lag when pulled from a supine to a sitting position and can pull the shoulders up slightly and move the head from side to side when placed in the prone position. Six-month-old infants can lead with their head when pulled from a supine to a sitting position. Some 4-month-old infants can roll from a supine to a sitting position but are not able to get into a sitting position at this age.
52. The answer is B [, section III.A]. This patient’s physical examination is consistent with developmental hip dysplasia (DDH). Patients with DDH whose diagnosis is delayed may present with a limb length discrepancy. Examination may also reveal asymmetric thigh or buttock folds and diminished abduction of the affected hip. Transient synovitis, a postinfectious inflammatory response within the joint, also causes a limp, but a limb length discrepancy would not be present. Legg–Calvé–Perthes disease is idiopathic avascular

necrosis of the femoral head that occurs most commonly in children 4–9 years of age. Femoral anteversion is an inward angulation of the femur that results in intoeing, and it is not manifested by a limb length discrepancy or asymmetric skin folds. Slipped capital femoral epiphysis is slippage of the femoral head off the femoral neck and is most common in overweight adolescent males.
53. The answers are B, A, and B, respectively [, section V.A–B and Figure 14-1]. Both Wilms tumor and neuroblastoma commonly present with an abdominal mass in the first 5 years of life. Clinical features of neuroblastoma vary based on the tumor location. Two percent of patients with neuroblastoma have acute cerebellar atrophy with ataxia, opsoclonus, and myoclonus (so-called “dancing eyes and dancing feet”; question 69). Clinical features of Wilms tumor include hemihypertrophy, aniridia, and genitourinary malformations (question 70). The prognosis for neuroblastoma is very good for infants younger than 1 year, and spontaneous regression is possible, even with metastatic disease (stage IV-S disease; question 71).
54. The answer is D [, section X.D.1 and Table 10-5]. Age is an important factor in determining the cause of lower gastrointestinal (GI) bleeding. In neonates and infants, an allergy to protein antigens in their food source can cause intestinal inflammation with GI bleeding, which may be occult or frank. Mucus may also be present in the stool. Juvenile polyp is the most common cause of lower GI bleeding after the neonatal period. Meckel diverticulum and infectious enterocolitis are important causes of bleeding, especially after infancy. Inflammatory bowel disease typically occurs later in childhood.
55. The answers are D and B, respectively [, sections IV.C.1 and IV.H.2]. Turner syndrome is characterized by short stature, a webbed neck with a low posterior hairline, a shield chest, and ovarian dysgenesis that manifests as delayed puberty. Complications of Turner syndrome include scoliosis, left-sided cardiac lesions, and hypothyroidism. Therefore, screening for hypothyroidism is indicated (question 73). Osteogenesis imperfecta type I is characterized by blue sclerae, fragile bones that result in frequent fractures, and easy bruisability. Patients are at risk for skeletal deformities and for early conductive hearing loss. In addition, they may have dentinogenesis imperfecta which leads to increased breakage and loss of teeth (question 74).
56. The answer is D [, section IV.C.4]. Irregular respirations with no particular pattern are also termed ataxic respirations and result from severe brainstem injury. This respiratory pattern often suggests impending brain death. Cerebellar injury and cerebral strokes do not usually result in changes in the respiratory pattern. Acute drug overdose with narcotics or sedatives causes hypoventilation.
57. The answer is E [, section I.D.2.b]. Pathologic short stature (child’s height is greater than 3 standard deviations below the mean with abnormal growth velocity) may be characterized as proportionate or disproportionate. Disproportionate short stature is defined as short stature in patients who are very short-legged with an increased upper-to-lower (U/L) body segment ratio, suggesting either a skeletal dysplasia or rickets (the latter would also be expected to present with bowing). In contrast, malnutrition, child neglect, growth hormone deficiency, and fetal alcohol syndrome are all causes of proportionate short stature, in which the patient has a normal U/L body segment ratio.
58. The answer is B [, section III.B.5]. Osteomyelitis is characterized clinically by fever, irritability, localized bone pain, and swelling and erythema of the infected site. Osteomyelitis during childhood is most commonly acquired by hematogenous seeding. Direct inoculation and contiguous spread are less common ways of acquiring this infection. A plain radiograph is not a good study to assess early osteomyelitis because it is usually normal early in the course of disease. The elevation of periosteum as a result of infection does not usually become evident on a plain radiograph until after 10–14 days. Staphylococcus aureus and Streptococcus pyogenes are the two most common causative pathogens. Salmonella typhi infection occurs most commonly in a child with underlying sickle cell disease. Blood culture is positive in only 50%

of patients with osteomyelitis. All children with osteomyelitis require initial management with intravenous antibiotics.
59. The answer is C [, section I.H.1]. The normal umbilical cord should contain two arteries and one vein. The presence of only one umbilical artery should prompt suspicion of an associated underlying congenital cardiac or renal anomaly. Neither an umbilical hernia nor diastasis recti (the separation of the rectus abdominis muscles at the midline of the abdomen) is associated with underlying structural problems. Patients with hypospadias, in which the urethral meatus is located on the ventral surface of the penis, are not likely to have associated urinary malformations. This is in contrast to epispadias (in which the urethral meatus is located on the dorsal surface of the penis), in which associated problems, such as bladder extrophy, may be present. A nevus simplex (or “salmon patch”) is the most common vascular lesion of infancy and is also not associated with underlying pathology. This is in contrast to a nevus flammeus, or “port wine stain” in the distribution of the ophthalmic branch of the trigeminal nerve, which may be associated with Sturge–Weber syndrome.
60. The answer is B [, section II.C]. This patient’s age and his presentation with splenomegaly are most consistent with adult-type chronic myelogenous leukemia (CML). Most patients with adult-type CML are older children and adolescents, and they may be diagnosed with splenomegaly picked up as an incidental finding during a routine health maintenance visit. Adult-type CML is characterized by the presence of the Philadelphia chromosome. Juvenile monomyelocytic leukemia (JMML), another type of CML, often presents with fever, a chronic eczematous-type facial rash (not found in adult-type CML), lymphadenopathy, petechiae, and purpura. JMML occurs in young children and infants. Radiation therapy is ineffective in adult- type CML; chemotherapy and bone marrow transplantation are more useful. Adult-type CML follows a biphasic course, characterized by easy control of elevated white blood cell (WBC) counts for several years followed by an acute deterioration resembling acute leukemia. Adult- type CML is characterized by extremely high WBC counts, often >100,000 cells/mm3.
61. The answer is B [, section VII.F]. This patient’s clinical presentation, including the geographic location in which she lives, is consistent with Lyme disease. The management of Lyme disease is aimed at eradicating Borrelia burgdorferi, the spirochete that causes the infection. This patient has early localized disease and should be treated with amoxicillin. Doxycycline should be avoided in children younger than 9 years because of concerns regarding permanent tooth discoloration. Patients with complications of carditis or meningitis require intravenous penicillin or intravenous ceftriaxone.
62. The answer is A [, sections III.B.4 and III.B.6]. This patient presents with signs and symptoms consistent with croup (laryngotracheobronchitis). Given the lack of stridor at rest and the absence of respiratory distress, supportive care alone is indicated. Patients with stridor may benefit from a single dose of corticosteroid treatment to reduce airway inflammation. Patients with both stridor at rest and respiratory distress also benefit from nebulized racemic epinephrine to vasoconstrict subglottic tissues. Albuterol can be useful if wheezing is present, but does not have efficacy on the edematous subglottic tissues. Ribavirin is not effective in croup.
63. The answer is B [, section XVI.B]. This patient’s presentation is typical of neurocysticercosis, a helminth infection caused by the ingestion of the eggs of Taenia solium, a pork tapeworm. Treatment includes anticonvulsant medications; surgical excision is not indicated. Ova and parasite evaluation detects the eggs of the organism in only 25% of cases. Diagnosis is often made by findings on computed tomography (CT) or magnetic resonance imaging, which reveal cysts that may be calcified. Calcifications are evidence of old, quiescent infection.
64. The answer is D [, section IV.F.6]. The antinuclear antibody (ANA), although not specific, is almost universally elevated in children with systemic lupus erythematosus (SLE) (>99%). If a patient has a negative ANA, it is very unlikely that he or she has SLE. Erythrocyte

sedimentation rate and rheumatoid factor are often, but not always, elevated in SLE but are nonspecific. Anti–double-stranded DNA antibodies are relatively specific for SLE, and uniquely, their levels can be used to monitor progression of disease, especially renal disease. Antibodies to the Smith antigen are elevated in only 30% of patients with SLE, but they are strongly suggestive of SLE when present.
65. The answers are C, H, B, F, and G, respectively [, Table 20-3; sections VIII.D.1, VIII.D.3, and VIII.D.6]. Antidotes exist for a relatively small number of toxins. Iron is commonly found in prenatal vitamins (question 84). The antidote for iron is deferoxamine, an iron chelator. The blood level of acetaminophen in the 15-year-old girl (question 85) should be plotted on the Matthew–Rumack nomogram. If it is elevated, she should be treated with N-acetylcysteine, the antidote for acetaminophen poisoning. Symptoms and signs of carbon monoxide toxicity include headache, vomiting, auditory and visual changes, slurred speech, and mental status changes (question 86). Cherry-red skin is a characteristic, although uncommon, finding. The antidote is 100% oxygen, which displaces carbon monoxide from the hemoglobin molecule. Isoniazid can cause seizures (question 87), and the antidote is pyridoxine (vitamin B6). Dystonia can occur from several types of medications, including phenothiazines (question 88), and antidotes for dystonia include diphenhydramine and benztropine.
66. The answer is A [, section VIII.D]. Bleeding that is prolonged and that occurs at irregular intervals, as found in this patient, is termed menometrorrhagia. The bleeding may be secondary to dysfunctional uterine bleeding (DUB), the most common cause of vaginal bleeding during adolescence, but the presence of painful bleeding is inconsistent with DUB. Because she is having painful bleeding and because her bleeding is heavy, both a pelvic examination and laboratory studies to evaluate for pregnancy, anemia, and blood dyscrasias are indicated. Bleeding secondary to DUB occurs because of unopposed estrogen production. Dilation and curettage may be required for bleeding that does not stop with hormonal therapy, but it is not the first-line treatment.
67. The answer is B [, section VII.D]. The patient’s presentation is consistent with Guillain–Barré syndrome, an acute inflammatory demyelinating polyneuropathy and a well-described cause of symmetric ascending weakness in childhood. Albuminocytologic dissociation in the cerebrospinal fluid (CSF), usually present 1 week after the onset of symptoms, is an important hallmark of Guillain–Barré syndrome. Patients classically have increased CSF protein levels but normal cell counts. Demyelination results in slowing of the nerve conduction velocity. Magnetic resonance imaging of the spine would be expected to be normal. Guillain–Barré syndrome is associated with many infectious agents, most commonly a recent infection with Campylobacter jejuni.
68. The answer is D [, section III.B.2]. The patient’s presentation is most consistent with transient synovitis, a postinfectious inflammatory response in the hip joint that is a common cause of limp in toddlers. Transient synovitis commonly follows an upper respiratory infection or diarrheal illness, and is characterized by limp, irritability, low-grade fever or absence of fever, and a normal or slightly elevated white blood cell (WBC) count and erythrocyte sedimentation rate (ESR). Examination may reveal hip flexion, external rotation, and abduction, similar to that seen in septic arthritis, although usually less pain is elicited on movement of the hip joint compared with septic arthritis, in which pain is very significant. Septic arthritis and osteomyelitis also have an elevated WBC count and ESR, and patients are usually more ill- appearing. Slipped capital femoral epiphysis is a slipping of the femoral head off the femoral neck and typically presents with a painful limp, although this is more common in adolescence. A toddler’s fracture is a spiral fracture of the tibia, which may occur after only mild trauma. A patient with a toddler’s fracture may suddenly refuse to walk or bear weight. However, examination of the hip would be normal, although mild erythema, swelling, and tenderness may be found over the tibial fracture site.

69. The answer is C [, section X.A]. IgA deficiency is the most common immune deficiency and is associated with an increased incidence of sinusitis, otitis media, bronchitis, and pneumonia. Low quantitative IgA levels confirm the diagnosis. A complete blood count is usually part of the workup for immune deficiency but would not confirm IgA deficiency. Anergy skin testing and the assessment of T-cell response to mitogens evaluate the competency of T cell–mediated immunity, and patients with cellular-mediated immunodeficiency are more likely to present with recurrent viral, fungal, or protozoan infections. A normal total hemolytic complement demonstrates that all components of the complement pathway are present and functional. Patients with terminal complement deficiency usually present with recurrent meningococcal and gonococcal infections.
70. The answer is A [, section IV.A.2.b]. Blount disease, a progressive angulation of the proximal tibia, should be suspected in any child with progressive bowing, unilateral bowing, or persistent bowing after 2 years of age. In this patient with progressive bowing that is more prominent in one leg, Blount disease would be a concern. Blount disease is diagnosed with a standing anterior–posterior radiograph of the lower extremities that reveals an exaggerated metaphyseal–diaphyseal angle. Physical therapy is not effective. Genu varum (bowed legs) is not likely because the bowing is more prominent in one extremity, and because it is still present after 2 years of age. Genu valgum (knock-knees) is characterized by knees angulated toward the midline. Orthotic shoes are not useful in Blount disease.
71. The answer is C [, section VIII.A]. Sydenham chorea is an autoimmune response of the central nervous system to group A β-hemolytic streptococcal infection. The major clinical features include chorea and emotional lability. The choreic movements of the arms and legs can interfere with daily activities such as eating, writing, and walking. Migraine does not present with motor difficulties. In complex partial epilepsy, motor, psychomotor, or sensory findings may occur, but consciousness is decreased during the seizure. Hydrocephalus with increased intracranial pressure most commonly presents with vomiting and early morning headache rather than with emotional lability, restlessness, and clumsiness. Tourette syndrome presents with motor or phonic tics.
72. The answer is C [, section XVII.A.2]. The patient’s clinical features are consistent with Rocky Mountain spotted fever, which is caused by infection with Rickettsia rickettsii, a gram-negative bacteria transmitted by the bite of a tick. Clinical features include fever, a petechial rash that begins on the hands and feet, myalgias, hypotension, and hepatosplenomegaly. Thrombocytopenia and anemia also occur. Treatment should be started empirically on the basis of the clinical and epidemiologic features rather than waiting for confirmation by serologic testing. Prophylactic antibiotics to prevent infection are not indicated. The incidence is highest in school-aged children.
73. The answer is C [, section IX.A.6 and Table 7-5]. The patient’s clinical presentation is most consistent with toxic shock syndrome (TSS). The most common organism associated with TSS is Staphylococcus aureus, although some cases may be associated with group A β-hemolytic streptococcus. This patient’s signs and symptoms are inconsistent with Kawasaki disease, hemolytic uremic syndrome (caused by Escherichia coli O157:H7), or Lyme disease (caused by Borrelia burgdorferi).
74. The answers are C, B, D, and E, respectively [, sections XI.A.2, XI.B, IV.D.1, and Chapter 10, Table 10-1]. Patients with Wilson disease (question 97) have a high serum copper level that leads to copper deposition in the brain, eyes, and liver because of a copper excretion defect. These patients present with neurologic findings such as behavioral changes, seizures and ataxia, hepatic dysfunction, and Kayser–Fleisher rings in the peripheral cornea that represent copper deposition in Descemet membrane. Patients with Menkes kinky hair disease (question
98) present with myoclonic seizures, progressive neurologic degeneration, mental retardation, and pale kinky friable hair. Menkes kinky-hair disease is caused by a defect in copper

transport, and patients have a low serum copper level. Patients with acrodermatitis enteropathica (question 99) present with a symmetric dry vesiculobullous scaly rash, failure to thrive, and chronic diarrhea. Acrodermatitis enteropathica is caused by zinc deficiency, and low serum zinc is found on laboratory evaluation. Patients with DiGeorge syndrome (question
100) have hypocalcemia (caused by parathyroid hypoplasia), cellular-mediated immunodeficiency, which may result in recurrent fungal infections (caused by thymic hypoplasia), and cardiac findings that may include aortic arch abnormalities. DiGeorge syndrome is caused by a defect in the structures derived from the third and fourth pharyngeal pouches.
75. The answer is D [, section I.G.6]. Hydronephrosis is the most common cause of an abdominal mass in healthy newborns. Although the differential diagnosis includes Wilms tumor and neuroblastoma, both are less common than hydronephrosis as an explanation for abdominal masses in infancy. Wilms tumor rarely presents in the immediate newborn period. Ovarian cysts are more likely to present with an abdominal mass than ovarian torsion.
76. The answer is E [, section XII.A.2]. The infant likely has a tracheoesophageal fistula. The most common type, involving >90% of cases, is comprised of proximal esophageal atresia with a distal tracheoesophageal fistula. This is commonly associated with polyhydramnios due to the fetus’ inability to swallow amniotic fluid. Feeding difficulty is observed immediately after birth. Infants with tracheoesophageal fistula should be examined carefully for anomalies associated with VACTERL (see Chapter 5 section IV.I.1.2.). Hepatomegaly is not typically associated with esophageal atresia. Meconium-stained fluid can cause meconium aspiration syndrome and is more likely to present with respiratory distress than feeding intolerance. Congenital diaphragmatic hernia would likely have been apparent on chest radiographs.
77. The answers are D, E, and B, respectively [, sections VI.B and VI.C]. Babies born to mothers with SLE are at risk for congenital third-degree atrioventricular (AV) block due likely to immune-mediated damage to the conducting system through maternally passed autoantibodies. Babies often present with bradycardia (question 103). Long QT syndrome is a disorder of myocardial repolarization. Long QT syndrome may be congenital or acquired from electrolyte abnormalities (hypokalemia, hypomagnesemia) or medication side effects. This syndrome places children at risk for a lethal ventricular arrythmia known as torsades de pointes (question 104). Type I second-degree AV block (also known as Wenckebach) is characterized by progressive prolongation of the PR interval leading to failed AV conduction (question 105). First-degree AV block is characterized by PR prolongation without failed AV conduction. Type II second- degree AV block is an abrupt failure of AV conduction without progressive prolongation of the PR interval.
78. The answer is A [, section I.E.2]. The patient’s restrictive vegan diet places her at risk for vitamin B12 deficiency. Clinical findings of B12 deficiency include hyporeflexia, ataxia, and a smooth red tongue, in addition to the classic symptoms and signs associated with anemia (fatigue and pallor). Vitamin B12 deficiency leads to a macrocytic anemia with a mean corpuscular volume (MCV) >95. Diagnosis is confirmed with low serum vitamin B12 levels. Other causes of vitamin B12 deficiency include pernicious anemia, inflammatory bowel disease, and short gut sydrome. This patient would not be expected to have a low white blood cell count or prolonged international normalized ratio (INR) based on the described symptoms and signs. Hyponatremia can produce fatigue and neurologic symptoms; however, symptomatic hyponatremia usually presents more acutely than the several month history noted in this case.
79. The answers are E, A, and D, respectively [, Figure 14-1]. Females with Turner syndrome are at risk for developing gonadoblastomas, a neoplasm that occurs in dysgenetic gonads (question 107). Risks for other types of cancer in patients with Turner syndrome are likely similar to that of the general population. Patients with Down syndrome are at risk for both

transient myeloproliferative disorder, and subsequently for acute myelogenous leukemia (question 108). Ataxia telangiectasia is characterized by immunodeficiency which may cause recurrent sinopulmonary disease, leading to chronic lung disease with bronchiectasis and pulmonary fibrosis. Patients with ataxia telangiectasia will develop malignancy, the majority of which are lymphomas (question 109).
80. The answer is B [, section IV.A.1, Figure 17-3 and Table 17-5]. This child has benign intoeing due to tibial torsion. Tibial torsion is distinguished from the other common causes of intoeing
—metatarsus adductus and femoral anteversion—based on age and the thigh–foot angle with decreased external rotation. Metatarsus adductus is typically identified in children younger than 1 year, and femoral anteversion is typically identified in children older than 3 years.
Talipes equinovarus (“club foot”) is present at birth and has a fixed inversion and equinus position of the foot with minimal flexibility. Genu varum presents with bowed legs rather than intoeing.
81. The answer is C [, section IX.B.4.a]. The correct answer is to recheck the thyroid-stimulating hormone (TSH) and thyroxine (T4) levels. These results should be available rapidly, and levothyroxine can be initiated when hypothyroidism is confirmed. No delay should occur in repeat testing, and it would not be appropriate to wait 1 week to contact the family. Referral to a pediatric endocrinologist for additional workup may well be indicated once hypothyroidism is confirmed. Thyroid imaging may be useful once the diagnosis is confirmed.
82. The answer is E [, section II.A.5]. Testicular enlargement is the first sign of puberty in boys. Puberty refers to the activation of hypothalamic–gonadal axis. Puberty generally begins after the age of 9 years and is not considered delayed until after the age of 14 years. Typically, patients develop more classical signs of puberty (voice change, growth spurt, and facial hair) at the later stages of puberty. It is important to realize that the growth spurt in boys occurs late in puberty as compared with girls in which the growth spurt occurs early in puberty. Apocrine odor is a sign of adrenarche. The timing of adrenarche is not linked to activation of the hypothalamic–gonadal axis. Therefore, the presence of apocrine odor does not indicate that a patient is in puberty.
83. The answer is B [, section XIV.E-F]. This infant has a febrile urinary tract infection (UTI) consistent with pyelonephritis. Bagged urine cultures, especially in uncircumcised infants, are unreliable. This infant is also at risk for having significant vesicoureteral reflux because of the positive family history of kidney failure from reflux nephropathy and because of his history of prenatal hydronephrosis. Infants with suspected pyelonephritis should be admitted to the hospital for intravenous antibiotics for 2–3 days, pending the results of the catheterized urine culture and sensitivities, and then complete a 14-day course of antibiotics as an outpatient. Uncircumcised male infants under six months of age are at a greater risk for UTI than circumcised infants. An imaging evaluation is important in this infant because of both the pyelonephritis and the positive family history. It is useful to obtain the renal ultrasound in the first day or two in the hospital to assess for severe hydronephrosis that might require urgent urologic consultation. The voiding cystourethrogram (VCUG) should be postponed until the urine culture is subsequently shown to be negative. It will be important for the radiologist to assess the posterior urethra during the VCUG to rule out a posterior urethral valve. Although vesicoureteral reflux may not be as serious as previously thought, the highest risk group for renal parenchymal damage is the child younger than 2 years with vesicoureteral reflux and pyelonephritis.
84. The answer is B [, Table 8-1]. The murmur described in the vignette is a venous hum that results in a vibratory hum throughout the cardiac cycle as blood returns from the head through the internal jugular vein. The murmur disappears either when the child is supine or turns his or her head or when placing a finger over the internal jugular vein. Turbulence within the carotid artery would typically produce a bruit that would be a sign of obstruction

within the carotid artery, usually from plaques. A bruit would be unusual in a 7-year-old, but there are rare circumstances where a bruit may be appreciated. A patent ductus arteriosus (PDA) would be a continuous machinelike murmur, which should not be affected by position. Choice D refers to extra heart sounds, such as clicks transmitted to the aorta in aortic stenosis or bicuspid aortic valves. These murmurs are unaffected by the patient’s position. Benign, functional cardiac murmurs that are “musical” in nature are typically systolic and are heard over the precordium.
85. The answer is B [, sections V.F.3.d and VI.C.3]. This vignette describes a very worrisome episode of syncope that occurred when the patient was exercising. The most concerning cardiac pathologies associated with this presentation include long QT syndrome and hypertrophic cardiomyopathy. Syncope with exercise is a “red flag” and should be fully investigated immediately by a pediatric cardiologist before any of the other interventions may be considered.
86. The answer is B [, section V.D.4]. The patient’s history and findings, especially sharp pain that worsens when lying down, in the face of a pulsus paradoxicus, are classic for pericarditis with a pericardial effusion. The remainder of the physical findings would not be characteristic of pericarditis. Other common findings of pericarditis include a pericardial rub and muffled heart sounds. Bounding pulses are typically seen when there is significant reversal of blood flow, such as a large patent ductus arteriosus (PDA) or severe aortic regurgitation. An apical click is associated with a bicuspid aortic valve.
87. The answer is C [, Table 8-1]. The examination describes a healthy child with a benign family history. Her murmur as described is typical for a benign Still’s (left ventricular outflow tract) murmur.
88. The answer is C [, section V.D.2]. This presentation is consistent with hand–foot–mouth disease, a contagious viral infection caused by coxsackievirus (an enterovirus). Painful blisters typically develop on palms, on soles, and in the mouth. This is typically a self-limited illness and does not required specific treatment. Amoxicillin is not recommended given that this is a viral infection. Acyclovir, an antiviral that has activity against herpes viruses, is not indicated for enteroviral infection.
89. The answer is C [Chapter 16, section II]. The constellation of findings is consistent with Kawasaki disease. She meets criteria with at least 5 days of fever, conjunctivitis, mucous membrane changes, rash, and a unilateral lymph node. This is an inflammatory condition, not infectious, and so antibiotic therapy is not indicated. While patients with Kawasaki disease may develop aseptic meningitis, the next best step in management is to treat with intravenous immune globulin (IVIG) and high-dose aspirin to prevent cardiac sequelae. Although steroids have been used in treatment of refractory Kawasaki disease, they have no role in primary therapy. Her risk of developing coronary artery aneurysms if left untreated is approximately 20%.
90. The answer is A [, section XVI.B]. This child’s presentation and computed tomography (CT) findings (a parenchymal cyst that may be enhancing) are consistent with neurocystercercosis. New-onset seizures in a child from a developing country should include evaluation for parasite infection and neuroimaging. Anticonvulsant therapy may be indicated, but discontinuation of antiepileptic medication after 2 years of being seizure free can be considered. Patients usually acquire neurocystercercosis from ingestion of Taenia eggs, not from ingesting raw pork, but via the fecal oral route after contact with someone infected with the adult tapeworm, excreting eggs in their stool. Patients with neurocysticercosis have Taenia eggs or adult tapeworm identified in their stools only 25% of the time. Surgical intervention, and placement of a ventriculoperitoneal shunt, is only required if there is concomitant obstructive hydrocephalus, which is not present in this case.
91. The answer is C [, section VIII.B.1 and Chapter 1, section III.C.6]. Mumps virus is the most

common cause of viral parotitis, but other viruses can be implicated. Mumps is a vaccine- preventable disease, and the vaccine is recommended for administration at 12 months of age and again at kindergarten entry, but can be given any time with at least 4 weeks following the first vaccine. Diagnosis of mumps is made by culturing the virus from urine or by serologies, but not from parotid gland aspiration. Congenital infection with mumps has not been described. There is no recommendation to vaccinate with a measles, mumps, and rubella (MMR) vaccine before college entry, unless an individual has not yet received two vaccines.
92. The answer is C [, section IV.I]. Although universal tuberculosis screening was required for school entry in the past, targeted screening based on risk factors is now recommended. Indications for tuberculosis testing include birth in a high-risk area outside of the United States, travel for >1 week (cumulative) in a high-risk area outside of the United States, history of exposure to someone with active tuberculosis, or close contact with anyone with a positive skin test for tuberculosis (TB).
93. The answer is C [, section XIV.C]. The patient in this scenario is immunosuppressed and is taking an anti–tumor necrosis factor (TNF) drug, which will impair her T-cell immunity. She is therefore at risk for severe and disseminated disease. Other patients at high risk include young infants, pregnant women, and the elderly. Most infections are asymptomatic, although severe pulmonary infections can occur in healthy persons. Complications of systemic disease include meningitis, osteomyelitis, and lymphadenitis. Coccidioidomycosis is endemic in the Southwestern United States, northern Mexico, and areas of Central and South America. Histoplasmosis, another endemic fungal infection, is endemic in other parts of the country, including the Mississippi River Valley. Coccidioidomycosis is treated with antifungal medications (fluconazole).
94. The answer is E [, section XVII.A.2]. This boy likely has Rocky Mountain spotted fever (RMSF). He has fever, headache, and a typical rash and has just returned from an endemic area. Given that he is ill-appearing and has a petechial rash, sepsis with typical bacterial pathogens should also be considered. Empiric coverage of both rickettsial and typical bacterial pathogens includes doxycycline and a third generation cephalosporin. RMSF is a life- threatening infection and empiric treatment should be given rather than only supportive care.
95. The answer is B [, section XIII.A.3.b]. Children with perinatally acquired human immunodeficiency virus (HIV) infection are frequently asymptomatic in the first year of life but then develop recurrent common bacterial infections, including otitis media and pneumonia. Failure to thrive is a common presenting symptom as well. Other findings include difficult to treat thrush, Pneumocystis jirovecii pneumonia (PCP), and loss of developmental milestones. These symptoms may overlap with an adult presentation of HIV infection, but it would be very unusual for a young child with HIV infection to present in the first few years of life with cytomegalovirus (CMV) retinitis, shingles, or central nervous system (CNS) lymphoma, all which are acquired immunodeficiency syndrome (AIDS)–defining illnesses in adults with HIV infection. Children with perinatally acquired HIV infection do not develop Kaposi sarcoma in childhood, which may be a presenting sign in adults with a new diagnosis of HIV infection.
96. The answer is C [, section XI.C.2 and Table 7-6]. This child may have enterohemorrhagic E. coli (EHEC) and hemolytic uremic syndrome (HUS), given her decreased urine output and pallor. In this case, further examination should be done to confirm HUS. A complete blood count (CBC) should be obtained to examine for thrombocytopenia, as well as a peripheral smear to look for microangiopathic hemolytic anemia. She should not be given antibiotics, as this may increase the risk of HUS. Rehydration solutions should not contain KCl until her urine function is evaluated. Given that she has bloody diarrhea, she is not likely to have Giardia lamblia. Although Clostridium difficile can be a cause of bloody diarrhea in children, empiric treatment with antibiotics is not indicated in this child.

97. The answer is B [, section III.C.11]. Human papilloma virus (HPV) is one of the most common sexually transmitted disease in the United States. The vaccine is ideally administered before the onset of sexual activity to develop protective immunity but may be administered to males through age 21 years and to females through age 26 years. Vaccination is recommended for both males and females to decrease the contagious risk and to lower the risk of complications, including cervical, anal, oropharyngeal and penile cancers. Along with hepatitis B vaccine, HPV vaccine is one of the few cancer-preventive immunizations.
98. The answer is E [, Table 1-5]. Vitamin D supplementation is essential for bone health in the pediatric population, especially to prevent conditions such as rickets in infants and toddlers. Vitamin D facilitates the dietary absorption of calcium from the gastrointestinal (GI) tract. During the first year of life, the recommended daily dose is 400 IUs. To meet the daily recommended dose, formula-fed infants would need to ingest 1 L of formula/day. Most infants will not ingest this volume until 4 months of age or older. Maternal supplementation with vitamin D will not prevent vitamin D deficiency in infants, as breast milk contains low levels of vitamin D. Introduction of cow’s milk before 12 months of age has been associated with iron-deficiency anemia and is therefore not recommended.
99. The answer is C [, Tables 1-1 and 1-3]. This 12-month-old patient is demonstrating inadequate weight gain. While this patient’s current weight is approximately twice her birth weight, on average, infants triple their birth weight by 12 months of age. The clinician should take a detailed history about possible causes of failure to thrive, focusing in particular on caloric intake. Other aspects of her health screening are normal, including her head circumference, language and ambulation skills, as well as her hemoglobin and lead levels.
100. The answers are C, B, D, E, and A, respectively [, section IV and Table 7-3]. Enteroviral meningitis is characterized by low-level pleocytosis, with normal chemistries and a negative Gram stain. A normal cerebrospinal fluid (CSF) profile usually has less than 10 white blood cells (WBCs). Note that this can be higher in neonates but is typically <10 in healthy infants and children, with protein and glucose in the normal range. Tuberculous meningitis is characterized by moderate pleocytosis, with slightly low glucose, but markedly elevated protein, and a negative Gram stain. The CSF in partially treated bacterial meningitis typically shows an elevated WBC count with low glucose and high protein (similar to untreated bacterial meningitis), but negative Gram stain owing to antibiotic administration. The CSF profile in pneumococcal meningitis typically has a very elevated WBC count, depressed glucose, elevated protein, and a positive Gram stain (in this case gram-positive diplococci suggest pneumococcal meningitis, and gram-negative diplococci would be suggestive of Neisseria meningitidis).
101. The answer is A [, sections III.D-E]. Although most vaccines cause local pain and redness in the immediate period, some live attenuated vaccines such as measles, mumps, and rubella (MMR) and varicella zoster virus (VZV) vaccines can cause a fever and rash 1–2 weeks after administration. Mild febrile illness (e.g., upper respiratory infection) is not a contraindication for immunizations. No reputable studies have shown a link between vaccines and autism or developmental disabilities. Thimerosal has also not been linked with developmental disability, but as a precautionary measure to reassure the public, thimerosal has been removed from almost all pediatric immunizations.
102. The answer is D [, section VI.E.4]. A history of upper respiratory infection preceding periorbital and peripheral edema suggests the possible diagnosis of nephrotic syndrome, the most common form of which is minimal change disease. Diagnostic criteria include heavy proteinuria, hypoalbuminemia, hypercholesterolemia, and edema. Patients with nephrotic syndrome are at increased risk for infection with encapsulated organisms such as Streptococcus pneumoniae, and may therefore present with spontaneous bacterial peritonitis, pneumonia, or overwhelming sepsis. In this case, given the fever on presentation, after a blood culture is

drawn, the patient should be started on empiric broad-spectrum antibiotic coverage. Patients with nephrotic syndrome also have a predisposition to thrombosis and not bleeding. The presence of red blood cell (RBC) casts would suggest a diagnosis of glomerulonephritis and not nephrosis. Infection with a toxin-producing bacteria may predispose to hemolytic uremic syndrome. Hearing loss is associated with Alport syndrome, most commonly inherited in an X-linked dominant fashion, and characterized by hypertension, hematuria, progressive hearing loss that starts in childhood, and ocular abnormalities in a subset of patients.
103. The answer is C [, section V.D.2]. This child’s presentation is most consistent with scabies infestation’s given the location of the papules on the wrists and along the waistline, as well as the presence of similar symptoms in the parents. The most appropriate treatment is overnight application of permethrin 5% cream for the whole family, with a repeat treatment in 1 week to catch the newly hatched mites. Topical corticosteroids and oral diphenhydramine could be used to help treat the pruritus associated with scabies infestation, but they are not effective as primary treatments. Cantharidin is indicated for the treatment of molluscum contagiosum, not for scabies infestation.
104. The answer is A [, section II.C.5]. Applied behavior analysis (ABA) has the most evidence to support its use in the treatment of young children with autism spectrum disorder. Developmental play–based interventions such as floor time, sensory integration therapy, and auditory integration therapy are frequently used but have not yet been shown to be as effective for the core symptoms in children with autism spectrum disorder. Iron chelation is a highly controversial biomedical treatment that carries a significant risk for physical harm to the child.
105. The answer is C [, section V.C.5]. This toddler’s presentation with a papular rash affecting the extremities and cheeks that has lasted >2 weeks is most consistent with Gianotti–Crosti syndrome (GCS). Also known as papular acrodermatitis, GCS most often occurs in children between the ages of 6 months and 12 years and is associated with hepatitis B, Epstein–Barr virus, enterovirus, and echovirus infections. Roseola presents as a pink papular rash that lasts only a few days and is preceded by a high fever, and varicella is associated with vesicular and crusted skin lesions as well as fever. The rash of fifth disease is a “slapped cheek” rash followed by a lacy reticulate rash on the extremities. Although the rash of pitryiasis rosea can also last several weeks, it is more common in older children presenting with oval pink scaling plaques on the trunk and extremities.
106. The answer is D [, section II.E.6]. Annual screening tests for a sexually active girl who is not consistently using a barrier method with each sexual encounter should include nucleic acid amplification tests (NAATs) of urine or a vaginal swab for Chlamydia trachomatis and Neisseria gonorrhoeae, serologic test for syphilis, and human immunodeficiency virus (HIV) testing. Routine screening of adolescents who are asymptomatic for Trichomonas vaginalis, bacterial vaginosis, herpes simplex, and human pailloma virus (HPV) is generally not recommended.
107. The answer is E [, section IV.A and Table 18-2]. This infant is presenting with nasolacrimal duct (NLD) obstruction. NLD obstruction is characterized by watery eye with increased tear lake, matted eyelashes, and often mucous in the medial canthal area. It resolves spontaneously without treatment in over 80% of cases by age 9–12 months, and so observation only would be the most appropriate management at this time. NLD probing by ophthalmology may be needed in older infants if the obstruction does not resolve. Because conjunctivitis is not present, topical antibiotics would not be indicated. Chlamydia trachomatis infection presents at 4–10 days of life, often with purulent or serous discharge.
108. The answers are B and D, respectively [, sections XI.A.4 and XII.D]. Chronic granulomatous disease (CGD) is characterized by defective neutrophil oxidative metabolism that results in impaired intracellular killing of catalase-positive bacteria and some fungal pathogens. Presentation classically includes abscess formation, and history of infection of the bones, skin,

liver, spleen and lungs. Aspergillus species is one of the major pathogens associated with CGD. Diagnosis is made by performance of the dihydrorhodamine (DHR) flow cytometry test (question 142). Deficiency of the late components of the classic pathway of complement (C5, C6, and C8) is associated with susceptibility to meningococcal infection. Testing for total serum hemolytic complement (CH50) will evaluate for normal functioning of the classic complement pathway (question 143).
109. The answer is C [, sections I and V.D]. The child in the description has suffered anaphylaxis, a severe, potentially life-threatening, systemic reaction. Any child who has had a severe systemic reaction to insect venom should be prescribed intramuscular epinephrine, and an emergency plan should be developed, providing instruction on what to do in the event of a future reaction. As allergy immunotherapy can significantly reduce the rate of reactions in those sensitive to insect venoms, all patients should be referred to an allergist for consideration of venom immunotherapy. Although diphenhydramine and corticosteroids may be useful in severe allergic reactions, epinephrine is the principal treatment. Instruction on management of local reactions and removal of a stinger may be helpful but are not as important as prescribing epinephrine.
110. The answer is B [, section IX.C.4]. The physical examination of this adolescent boy, in which the left testicle feels like a “bag of worms” that diminishes in size when the patient is supine, is consistent with a diagnosis of a left varicocele. In most cases, no surgical intervention is needed for the treatment of a varicocele and reassurance only may be given. An emergent urology referral is indicated if the examination was consistent with a diagnosis of testicular torsion. If the patient were sexually active, a nucleic acid amplification test (NAAT) for gonorrhea and chlamydia would be indicated. A cystic mass on transillumination of the scrotum is consistent with the diagnosis of a hydrocele and painless inguinal swelling with a diagnosis of inguinal hernia.
111. The answer is C [, section II.D.6]. The current recommendations for the treatment of attention deficit/hyperactivity disorder (ADHD) vary based on the age of the patient. For preschool children, first-line treatment is parent- and school-based behavior management. If behavioral strategies are ineffective and moderate to severe impairments are seen in the child’s functioning at home and preschool/child care and with peer interaction, then treatment with a stimulant medication is suggested. Organizational skills training and play therapy have not been shown to be effective as a first-line treatment at this age. A trial of nonstimulant medication is also not a first-line treatment.
112. The answer is A [, section V.A.3]. This patient’s clinical presentation with nail dystrophy and skin changes between the toes is most consistent with tinea unguium (onychomycosis) and tinea pedis. Topical agents (clotrimazole) are not very effective for nail infection, but rather, systemic therapy (terbenafine) is indicated. Topical corticosteroids and topical vitamin D analogues are not appropriate for this condition but might be tried for psoriasis of the nails. Surgical removal of the nails is a last resort for chronic nail infection over many years.
113. The answer is B [, section III.B.2].The signs and symptoms of depression in an adolescent include behavioral, psychological, and physical manifestations that impair the teen’s ability to function normally. A decrease in appetite, which eventually could lead to significant weight loss, fulfills one of the criteria listed in the Diagnostic and Statistical Manual of Mental Disorders (DSM-5) to make a diagnosis of depression. Diminished interest in activities, fatigue, and energy loss are other general symptoms. A patient must fulfill five or more of nine specific criteria almost every day for at least 2 weeks to make a definitive diagnosis of major depressive disorder. Other symptoms such as cough, abdominal pain, and headache with visual aura are not part of the DSM-5 criteria, although abdominal pain and headache may be present in a depressed teen. In addition, teens who are depressed usually diminish their extracurricular activities, not increase them.

114. The answer is A [, sections IV.C.1.a and VIII.D.6]. This patient has suffered a major burn injury with inhalation of hot gases. Management of any trauma victim begins with management of the airway. Any evidence of burns to the lips, nose, or tongue, such as singed nasal hairs or presence of soot in the nasal/oral airways, indicates a risk for progressive airway compromise. Management would therefore include immediate endotracheal intubation. Delay in securing the airway emergently could potentially lead to airway obstruction secondary to edema and inflammation. The finding of cherry-red lips is a late manifestation of carbon monoxide poisoning. Although obtaining a chest x-ray and arterial blood gas and determining body surface area burned are important management steps, they are not as important initially as airway management.
115. The answer is B [, sections IV.E.2.b and IV.E.6.c]. Tobacco smoke exposure is a known risk factor for sudden infant death syndrome (SIDS). Other risk factors include sleeping in the prone position and overheating. Infants should sleep in their own crib with a firm mattress and no pillow or excessive blankets. Cardiorespiratory monitors have not been shown to prevent SIDS.

Index

Note: Page numbers followed by “f” indicate figures and “t” indicate tables.

A
Abdominal wall defects, 90 Abnormal vaginal bleeding, 63–65 Absence epilepsy, of childhood, 308 Absence seizures, 305, 305f Abstract thinking, 27
Acetaminophen toxicity, 471–472, 471t Achondroplasia, 107
macrocephaly due to, 4 Acne, 452
Acne vulgaris
clinical features, 452–453
management, 453
pathophysiology, 452
Acquired adrenal insufficiency, 136 Acquired aplastic anemia, 332 Acquired heart disease, 201–206
acute rheumatic fever, 201 cardiomyopathy, 204–206
infective endocarditis, 201–202, 202t Kawasaki disease, 201
myocarditis, 204
pericarditis, 203–204, 203f Acquired microcephaly, 3, 4t Acquired torticollis, 404
Acuity assessment, 427 Acute abdominal pain
causes, 245–247
definition, 244 etiology, 245f evaluation, 244
Acute cerebellar ataxia, of childhood, 312 Acute kidney injury (AKI), 283–284
Acute lymphocytic leukemia (ALL), 348–350
causes, 348
classification, 348
clinical features, 348
diagnosis, 348–349
prognostic factors at time of, 349, 349t epidemiology, 348
etiology, 348

management
bone marrow transplant, 349 complications, 349–350
consolidation stage, 349
delayed intensification, 349
induction stage, 349
maintenance therapy, 349
prognosis, 350
Acute myelogenous leukemia (AML)
Auer rods, 350
causes, 350
classification, 350
clinical features, 350
diagnosis, 350
disorders associated with, 350 epidemiology, 350
etiology, 350
management, 350
prognosis, 350
Acute otitis media (AOM), 160–161 Acute pancreatitis, 246–247
Acute rheumatic fever, 201
Acyanotic congenital heart disease, 188–195, 189f, 189t, 190f, 190t, 191f, 193f, 194f aortic stenosis, 195
atrial septal defect, 188–191, 189f, 189t, 190t
clinical and diagnostic features, 188 coarctation of the aorta, 194–195, 194f patent ductus arteriosus, 193–194, 194f pulmonary stenosis, 195
ventricular septal defect, 191, 191f, 194
Adaptive responses, 373
Additional disorders, 108
Adenosine deaminase deficiency, 375
ADHD Attention deficit/hyperactivity disorder (ADHD)
Adolescent
growth and development, 45–46, 46t health screening, 46–51, 47f–49f, 50t
Adrenal gland disorders, 134–137
acquired adrenal insufficiency, 136 CAH, 135–136
classification, 134–135
general principles, 134
glucocorticoid excess, 136–137
β-Adrenergic agonists, for anaphylaxis, 367 Adrenocorticotropic hormone (ACTH), for infantile spasms, 308 Afebrile seizures, 305, 305f
α-Fetoprotein (AFP), 102
Alagille syndrome, 257–258

Alanine aminotransferase (ALT), 255 Alcoholism, 53
Allergen skin testing, 368 Allergic colitis, 254
Allergic conjunctivitis, 430, 431 Allergic contact dermatitis, 443 Allergic rhinitis, 367–369, 368f
clinical features, 368
defined, 367
diagnosis, 368
epidemiology, 367
etiology, 367 management
allergen avoidance, 368–369
immunotherapy, 369
patient education, 369
pharmacotherapy, 369
pathophysiology, 368
perennial, 367
seasonal, 367 Allergy
allergic rhinitis, 367–369, 368f
atopic dermatitis, 369–370, 370t drug, 372
food, 370–371
insect venom, 371
latex, 371
urticaria, 371, 372t
Alloimmune hemolytic anemia, 324f, 329 Alopecia areata
clinical features, 451–452
epidemiology, 450
etiology, 450
management, 452
α-fetoprotein (AFP), in spina bifida detection, 302 Ambiguous genitalia
differential diagnosis of, 133 evaluation, 133–134, 134f
Amblyopia
clinical features, 428
defined, 428
diagnosis, 428
epidemiology, 428
etiology, 428
management, 428
Amebiasis, 174
Amenorrhea, 63, 64t
Amino acid metabolism defects, 110–114, 112t, 113t

homocystinuria, 111–113, 112t–113t
organic acidemias, 113–114
urea cycle disorders, 112t–113t, 113 Amniocentesis, 102
Amniotic band syndrome, 109 Amniotocele (dacryocele), 432
Anaphylactic shock, 460
Anaphylactoid, 372 Anaphylaxis
clinical features, 367
defined, 367
diagnosis, 367
etiology, 367
insect venom allergy, 371 management, 367
Anemia, 323–331
classification, 323, 324f clinical features, 324t concepts, 323
defined, 323
epidemiology, 323
fetal hemoglobin, 323
folic acid deficiency, 326–327
glucose-6-phospate dehydrogenase deficiency, 327–328, 328t hemoglobin and age, 323
hemolytic, 327
alloimmune, 329
autoimmune, 328–329
microangiopathic, 329
hereditary elliptocytosis, 328
hereditary spherocytosis, 327
iron-deficiency, 324–325
lead poisoning, 326
macrocytic, 323, 326–327
microcytic, hypochromic, 323–326
normocytic, normochromic, 323, 327–331, 328t, 330t, 331t RBC aplasias, 327, 331, 332t
RBC defects, 327–329
sickle cell, 330–331, 330t, 331t sideroblastic, 326
thalassemias, 325–326
vitamin B12 deficiency, 327 Angelman syndrome, 106 Angulation of knee, 415 Ankylosing spondylitis, 395 Ann Arbor system, 352 Anorexia nervosa, 54–55, 55t

Anticipatory guidance, 16, 17t–18t topics for, age-appropriate, 16, 17t–18t
Antidotes, 471t
Anti-double-stranded DNA (anti-dsDNA) antibodies, in SLE, 390 Antihistamines
for allergic rhinitis, 369
for anaphylaxis, 367
for Atopic dermatitis, 370 for urticaria (hives), 371
Antinuclear antibody (ANA), 387 Antiphospholipid antibodies, in SLE, 390 Anti-Smith (anti-Sm) antibodies, in SLE, 390 Antithrombin III deficiency, 339
Anxiety, 36–37
Aortic stenosis, 195
Apnea, apparent life-threatening event (ALTE), 229–230 Apnea of prematurity, 84–85
Appendicitis, 245–246 Arterial blood gas (ABG), 217 Arthralgias, 447
Arthritis, 392
inflammatory bowel disease associated, 395 psoriatic, 395
reactive, 395
rheumatic fever, 392
Aseptic meningitis, 158–159, 158t Aspartate aminotransferase (AST), 255 Aspergillosis, 173
Aspiration pneumonia, juvenile dermatomyositis, 391 Asthma, 222–225, 223f, 225f, 372
Astrocytoma, 353
Astrocytomas, 354
Ataxia, 311
Ataxia telangiectasia, 376
Atlantoaxial instability, 404–405
Atopic dermatitis, 369–370, 370t, 444
acute vs. chronic manifestations, 369, 370t clinical features, 369–370
defined, 369
diagnosis, 370
epidemiology, 369
management, 370
Atresias, 243
Atrial septal defect, 188–191, 189f, 189t, 190t Attention deficit/hyperactivity disorder (ADHD)
assessment and diagnosis, 31
clinical features, 30 differential diagnosis, 31t

epidemiology and etiology, 30 management, 31
medications, 31–32, 32t
potential consequences, 31
Atypical hemolytic uremic syndrome (aHUS), 280 Autism spectrum disorder, 29–30
associated conditions, 30
clinical features, 30
epidemiology, 29
etiology, 29
management, 30
Autoimmune hemolytic anemia (AIHA), 324f, 328–329 Autoimmune hepatitis, 260–261
Autosomal dominant mode of inheritance, 100
Autosomal recessive mode of inheritance, 100

B
Back pain, 407–408, 407t Bacterial infections, 163–165
Bacterial meningitis, 156–158, 157t
Bacterial pneumonia, 222
Bacterial tracheitis, 219 Bacterial vaginosis (BV), 58–59
BAER (brainstem auditory evoked response), in hearing screening, 11 Balanitis, 14
Becker muscular dystrophy (BMD), 314–315
clinical features, 315
defined, 314
diagnosis, 315
epidemiology, 315
etiology, 315
management, 315
pathology, 315
pathophysiology, 315
prognosis, 315
Beckwith-Wiedemann syndrome, 356
Beckwith–Wiedemann syndrome, 106
Behavioral concerns, 33–38
anxiety, 36–37
breath-holding spells, 37
colic, 33–34
discipline issues, 38
encopresis, 35
enuresis, 34–35
selective eating, 36
sibling rivalry, 37
sleep problems, 35–36
temper tantrums, 37

toilet training, 38
Benign epilepsy with centrotemporal spikes (BECTS), 308
Benign rolandic epilepsy Benign epilepsy with centrotemporal spikes (BECTS)
Berger disease, 277
Bernard-Soulier syndrome, 338
Bilateral conjunctivitis, 384
Biliary atresia, 257
Biliary enzymes, 255
Bilirubin, 86, 255
Binge drinking, 54
Binocular vision, 428 Biologic poisonings
black widow spider, 475–476 brown recluse spider, 476 coral snakes, 477
pit viper snakes, 476
Black widow spider, 475–476 Bleeding, gastrointestinal
definitions, 253
laboratory confirmation, 253
lower, 254
UGI bleeding, 253–254
Blepharitis, 432
Blood vessel, disorders of, 337 Blount disease, 415
Body mass index, 1
Body surface area (BSA), 464 Bone and joint infections, 166 Bone marrow transplant
for acute lymphocytic leukemia, 349 for acute myelogenous leukemia, 350
Bone marrow transplantation, chronic granulomatous disease, 377 Bone mineral disorders, 142–144
Bone tumors, 357–359
Ewing sarcoma, 358–359
osteogenic sarcoma, 357–358 Borrelia burgdorferi, Lyme disease, 394 Brachycephaly, 4
Bracing prevents progression, 405 Brain tumors, 353–354
classification, 353
clinical features, 353
diagnosis, 354
epidemiology, 353
grade, 353
histology, 353
location, 353
management, 354

prognosis, 354
signs and symptoms, 353–354
Brainstem auditory evoked response (BAER), in hearing screening, 11 Brainstem gliomas, 354
Breath-holding spells, 37
Bronchiolitis, 220–221
Bronchopulmonary dysplasia, 227–228 Brown recluse spider, 476
Bruton tyrosine kinase (BTK) gene, 377
Buccal cellulitis, 164 Bulimia nervosa, 54–55, 55t Burkitt lymphoma, 352
Burns, 464–465
body surface area, 464 classification, 464
epidemiology, 464
first-degree, 464
management, 464–465
second-degree, 464
third-degree burns, 464

C
CAGE questionnaire, 54
Calcaneal apophysitis, 416
Calcium, for juvenile dermatomyositis, 391 Caloric irrigation, in comatose patients, 304 Campylobacter jejuni, Guillain-Barré syndrome, 312 Cancer
bone pain, 347
bone tumors, 357–359
brain tumors, 353–354
bruising, 347
causes, 347
early morning headache with vomiting, 347 environmental factors and, 347
etiology, 347
fever, 347
genetic disorders, 347, 348t germ cell tumors, 359–360 hypertension, 348
immunodeficiency diseases and, 347 infectious diseases and, 347 Langerhans cell histiocytosis, 360–361 leukemias, 348–351
leukocoria, 348
liver tumors, 359
lymphomas, 351–353, 351t
neuroblastoma, 354–356

pallor, 347
palpable mass, 347
petechiae, 347
presenting features, 347 renal tumors, 354–357, 355t retinoblastoma, 359
soft tissue tumors, 357
supraclavicular lymphadenopathy, 347 suprarenal tumors, 354–357, 355t Wilms tumor, 356–357
Wiskott-Aldrich syndrome, 347
X-linked lymphoproliferative disease, 347 Candidal vulvovaginitis, 59
Candidiasis, 173
CAP (chronic abdominal pain) clinical features, 248
definitions, 247–248
evaluation, 248
management, 249
prognosis, 249
risk factors, 248
Caput succedaneum, 3
Carbohydrate metabolism defects, 114–115 Carbohydrates, 15
Carbon monoxide, 474
Cardiac chest pain, 208–209, 209f Cardiogenic shock, 460 Cardiology
acquired heart disease, 201–206
acyanotic congenital heart disease, 188–195, 189f, 189t, 190f, 190t, 191f, 193f, 194f chest pain, 208, 209f
congestive heart failure, 187–188
cyanotic congenital heart disease, 196–201, 196t, 197f, 198t, 199f, 200f, 201f dysrhythmias, 206–208, 206t, 207f
innocent cardiac murmurs, 188 Cardiomyopathy, 204–206 Cardiovascular system
systemic lupus erythematosus, 389 Caries, nursing, 15–16
Cartilage-hair hypoplasia syndrome, 340 Cat bites, 475
Cat scratch disease, 178–179 Celiac disease, 238–239
Cellulitis, 163–164
Cellulitis, important variants of, 164 Central hypotonia, 296
Central nervous system (CNS), systemic lupus erythematosus, 389 Central precocious puberty (CPP), 150

Central (or isosexual) precocious puberty (CPP), 129 Cephalohematoma(s), 3
Cerebral palsy, 27–28, 28t Cervical adenopathy, 384
Cervical hydrosyringomyelia, and myelomeningocele, 301 Cervical lymphadenitis, 162
Cervical spine disorders, 404–405 Cervicitis, 59
Cervicitis trachomatis, 59–60
CHARGE Syndrome, 108
Chédiak-Higashi syndrome, 340, 378 Chemotherapy
for brain tumors, 354 for neuroblastoma, 356
Chest pain, 208, 209f
Chiari type II malformation, 300 Child abuse, 466–468
concepts, 466
physical, 466–467
sexual, 467–468
Child cardiopulmonary resuscitation (CPR), 459–460 airway, 459
breathing, 459
circulation, 459
concepts, 459
cycles, 460
sequence, 459
Child with possible infection, 151 Chlamydia trachomatis infection, 33 Cholecystitis, 247
Cholestatic diseases of infancy, 256–258 Cholestatic jaundice, 255
Chorionic villus sampling (CVS), 102
Chromosomal (micro) deletion syndromes, 104–105 Chronic abdominal pain (CAP)
clinical features, 248
definitions, 247–248
evaluation, 248
management, 249
prognosis, 249
risk factors, 248
Chronic benign neutropenia (CBN), of childhood, 339 Chronic granulomatous disease (CGD), 377
Chronic kidney disease (CKD)
clinical features, 284
etiology, 284
evaluation, 284
management, 285

Chronic myelogenous leukemia (CML), 350–351 adult-type, 350, 351
classification, 350
clinical features, 351
epidemiology, 350
juvenile, 350, 351
management, 351
prognosis, 351
Circumcision, 13–14
CKD (chronic kidney disease) clinical features, 284
etiology, 284
evaluation, 284
management, 285
Clavicular fractures, 417–419
Cluster headaches, 311
Coarctation of the aorta, 194–195, 194f Cobb angle, 405, 406f Coccidioidomycosis, 173–174 Cognitive deficits, 28–29, 29t Cognitive development, 26–27
Cognitive-behavioral therapy (CBT), 37 Colic, 33–34
Collagen synthesis, inherited disorders of, 337 Coma
assessment, 302–304
causes, 302, 303t
in childhood and adolescence, 303t defined, 302
etiology, 302
evaluation, 304
Comatose patient, 302–303, 303t Coma
assessment, 302–304
evaluation, 304
Common acute lymphocytic leukemia antigen (CALLA), 348 Common variable immunodeficiency
clinical features, 375
defined, 374
diagnosis, 375
epidemiology, 374
etiology, 375
management, 375
Compensated shock, 460 Complement, decreased, in SLE, 390 Complement system, disorders of, 378 Concrete thinking, 27
Congenital aplastic anemia, 332 Congenital aqueductal stenosis, 300

Congenital cataract, 435
Congenital diaphragmatic hernia, 89–90 Congenital glaucoma, 429, 434
clinical features, 434
defined, 434
etiology, 434
management, 434
prognosis, 434
Congenital heart block, systemic lupus erythematosus, 389 Congenital microcephaly, 3, 4t
Congenital muscular torticollis, 6, 404
Congenital myasthenia, 316 Congenital myotonic dystrophy, 299
Congestive heart failure (CHF), 187–188
Conjunctivitis allergic, 430, 431
bacterial, 430
causes, 430
chemical, 429
with contact lens use, 430 hemorrhagic, 431 neonatal
clinical features, 429, 429t, 430t defined, 429
differential diagnosis, 429
etiology, 429
unilateral, 430
viral, 431
Connective tissue disorders, 107 Constipation
definitions, 249
epidemiology, 249
etiology, 249–250
evaluation, 250
management, 250
normal stool patterns, 249 Contact dermatitis
allergic, 443
defined, 443
primary irritant, 443 Contraception
abstinence, 56
barrier methods, 56
etonogestrel implant, 56–57
injections, 57
intrauterine devices, 56 methods, 57t
oral, 57

vaginal ring, 57
Coral snakes, 477
Corneal abrasion, 433
Cornelia de Lange syndrome, 108 Coronary artery aneurysms
Coronary artery aneurysms, Kawasaki disease and, 385
Corticosteroids
for anaphylaxis, 367
for juvenile dermatomyositis, 391 malnutrition and, 337
for myasthenia gravis, 316 for rheumatic fever, 393
CRAFFT screening tool, 54 Cranial irradiation, 349
Craniopharyngioma, 354
Craniopharyngiomas, 353
Craniosynostosis, 4–6 clinical features of, 4–5 defined, 4
definition, 4
diagnosis of, 5
etiology, 4
management of, 6
CREST syndrome, 396
Cri du chat syndrome, 105 Crohn disease, 252
Crotalidae polyvalent antivenin, 476 Croup syndromes, 218–219, 218t Cryptococcal infection, 174
Cryptorchidism, 66 Cyanosis
clinical significance, 80
definition, 80
etiology, 80
evaluation, 81
management, 81
Cyanotic breath-holding spells, 37
Cyanotic congenital heart disease, 196–201, 196t, 197f, 198t, 199f, 200f, 201f general concepts, 196, 196t
TAPVR, 200–201, 201f
tetralogy of fallot, 197–198, 197f, 197t TGA, 198
tricuspid atresia, 198–199, 199f truncus arteriosus, 199–200, 199f
Cyclic neutropenia, 340
Cyclophosphamide, systemic lupus erythematosus, 390 Cystic fibrosis (CF), 225–227, 226t, 227f
Cysticercosis, 177

Cysts, 441

D
Dacryocystitis, 429
Dandy-Walker malformation, 300
Decompensated shock, 460 Decongestants, for allergic rhinitis, 369 Dehydration, 268–269
Delayed dental eruption, defined, 14 Delayed puberty, 130–131
Dental care, 14–16
dental abnormalities, 15–16
dental hygiene, 14–15
dental trauma, 16
tooth eruption, 14 Dental eruption
delayed, defined, 14
early, defined, 14
Dental floss, 15
Dental hygiene, 14–15
Dental trauma, 16
Depression, clinical features of, 52 Dermatology
acne vulgaris, 452–453
concepts, 441–443
hair disorders, 450–452 hypersensitivity disorders, 445, 445t inflammatory disorders, 443–445 newborn skin diseases, 443 pigmentary disorders, 450, 451t skin infections, 446–450, 448t vascular skin lesions, 453
Desmopressin acetate (DDAVP), 35 Development
disorders
attention deficit/hyperactivity disorder, 30–32, 31t, 32t autism spectrum disorder, 29–30
cerebral palsy, 27–28, 28t cognitive deficits, 28–29, 29t intellectual disability, 28–29, 29t learning disabilities, 29
motor deficits, 27–28, 28t normal
assessment, 23–24
cognitive development, 26–27 domains, 24–27, 24t, 25t, 26t general principles, 23 language skills, 25–26, 26t

milestones, 23–27, 24t, 25t, 26t motor development, 24–25, 24t social–emotional skills, 27
Developmental dysplasia of hip (DDH), 408–409 Developmental quotients (DQs), 24 Developmental screening, 16
Diabetes insipidus, 144–145
Diabetes mellitus, 137
Diabetic embryopathy, 109
Diabetic ketoacidosis (DKA), 139–140 Diarrhea, 166–168, 167t–168t
etiology, 166
evaluation, 166–167, 167t–168t
management, 167
Dietary counseling, for iron deficiency anemia, 325 DiGeorge, 104–105
DiGeorge syndrome, 376
Dihydrorhodamine (DHR), 377 Diphtheria
tetanus, and acellular pertussis (DTaP) vaccine, 7
Diplopia, 436
Discipline, basic premise of, 38 Discitis, 408
Disorders of adrenal gland, 134–137 Disorders of metal metabolism, 116–117 Disorders of puberty, 127–131
delayed, 130–131
normal, 127–128
precocious, 128–130
Disorders of sexual differentiation (DSD), 131–134 differential diagnosis, 132–133, 132f, 133f female sexual differentiation, 132
male sexual differentiation, 131–132 normal sexual differentiation, 131, 131f
Disseminated intravascular coagulation (DIC), acquired clotting factor disorders, 337 Distributive shock, 460
Diurnal enuresis, management of, 35 Dog bites, 475
Doll’s eyes, in comatose patients, 304 Dominant inheritance, 100
Double-blind placebo-controlled food challenge, in food allergy, 371
Down syndrome, 102, 103f, 103t DQs (developmental quotients), 24 Drowning, 465–466
clinical features, 465–466
defined, 465
epidemiology, 465
management, 466

pathophysiology, 465
prognosis, 466
Drug allergy, 372
Duchenne muscular dystrophy (DMD), 314–315 clinical features, 315
defined, 314
diagnosis, 315
epidemiology, 315
etiology, 315
management, 315
pathology, 315
pathophysiology, 315
prognosis, 315
Duodenal hematoma, 464
Dysmenorrhea, 63
Dysrhythmias, 206–208, 206t, 207f heart block, 207–208
long QT syndrome, 208 SVT, 206–207, 206t, 207f
Dysthymic disorder, 52
Dystrophin, 315

E
Early adolescence, 46
Early dental eruption, defined, 14 Ectoparasites, 449
Eczema Atopic dermatitis
Edema, scalp, 3
Edwards syndrome, 104
Ehlers–Danlos syndrome, classic type, 107 Electronic cigarettes (e-cigs), 53–54
Elemental iron, for iron deficiency anemia, 325 Emergency medicine
biologic poisonings, 475–477
burns, 464–465
child abuse, 466–468
child cardiopulmonary resuscitation, 459–460 drowning, 465–466
infant, 459–460
mammalian bites, 474–475 poisonings, 468–474, 470f, 471t, 473t shock, 460–461
sudden infant death syndrome, 468 trauma, 461–464
Encopresis, 35 Endocrinology
bone mineral disorders, 142–144
diabetes insipidus, 144–145

diabetes mellitus, 137
diabetic ketoacidosis, 139–140 disorders of adrenal gland, 134–137 disorders of puberty, 127–131
disorders of sexual differentiation, 131–134 hypoglycemia, 145–146
short stature, 124–127
thyroid disorders, 140–142
type 1 diabetes mellitus, 137–138
type 2 diabetes mellitus, 138–139
Endophthalmitis, 429
Enteral therapy, 252
Enuresis, 34–35
classification, 34
definition, 34
epidemiology, 34
etiology, 34
evaluation, 34
management, 35
Ependymomas, 353
Epidemic keratoconjunctivitis, 431
Epidural hematoma, 462–463
Epiglottitis, 217–218
Epilepsy, 304
classification of, 305
diagnosis, 306
evaluation, 306
management, 306–307
prognosis, 307 Epileptic syndromes
absence epilepsy, 308
benign epilepsy with centrotemporal spikes, 308 classification, 307
clinical features, 308
defined, 307
infantile spasms, 307–308
management, 308 Epinephrine
for anaphylaxis, 367 for food allergy, 371
Erysipelas, 163
Erythema infectiosum, 447
Erythema multiforme (EM), 445, 445t
Esophageal atresia with tracheoesophageal fistula (EA/TEF), 88–89, 89f Esotropia, 436
Etonogestrel implant, 56–57
Ewing sarcoma, 358–359
causes, 358

clinical features, 358, 358t diagnosis, 358
epidemiology, 358
etiology, 358
management, 358
prognosis, 359
Exanthem subitum Roseola infantum
Exome sequencing, 101
Exotropia, 436
Extended oligoarthritis, 387
Eye alignment, 427
Eye misalignment, 428

F
Failure to thrive (FTT), 2–3, 3f classification, 3
defined, 2
differential diagnosis of, 3f evaluation of, 3
Fatty acid oxidation disorders, 115
Febrile seizures, 305, 307
classification, 307
defined, 307
diagnosis, 307
epidemiology, 307
etiology, 307
management, 307
prognosis, 307
risk factors, 307 Female
puberty, 45, 128
reproductive health issues contraception, 56–57
pregnancy, 55–56
Female breast development, for sexual maturity ratings, 48f Femoral anteversion, 415
Femur fractures, 420
Ferritin, 325
Fetal alcohol syndrome (FAS), 109 Fetal exposure to drugs of abuse, 88 Fetal hydantoin syndrome, 109–110 Fetus, genetic evaluation of, 102 Fever in child, 152–154, 152t
Fever of unknown origin (FUO), 154–155, 155t Fluorescence in situ hybridization (FISH), 101 Fluoride, 14
Focal seizures, 305, 305f
Folic acid deficiency, 326–327

Food allergy, 370–371
clinical features, 370
defined, 370
diagnosis, 370–371
etiology, 370
management, 371
Forearm fractures, 420
Foreign body aspiration, 228–229 Fractures
clavicular, 417–419
femur, 420
forearm, 420
juvenile dermatomyositis, 391
supracondylar, 419–420
toddler, 420
types, 417
typical of child abuse, 420 Fragile X syndrome, 106
FTT (Failure to thrive), 2–3, 3f classification, 3
defined, 2
differential diagnosis of, 3f evaluation of, 3
Fungal infections, 173–174
aspergillosis, 173
candidiasis, 173
coccidioidomycosis, 173–174
cryptococcal infection, 174
FUO (fever of unknown origin), 154–155, 155t

G
Galactosemia, 114Galeazzi sign, 409
Gallbladder, hydrops of, Kawasaki disease and, 385 Gastroenterology
acute abdominal pain, 244–247, 245t chronic abdominal pain, 247–249, 248t constipation, 249–250, 250t
diarrhea, 250
gastroesophageal reflux, 240–241 gastrointestinal bleeding, 253–254, 254t hepatitis, 255–261, 259f, 260f inflammatory bowel disease, 251–252, 251t intestinal anatomic obstructions, 241–244 liver abnormalities, 255–261, 259f, 260f malabsorption, 237–239
nutrition, 236, 237t Gastroesophageal reflux (GER)
clinical features, 240–241

definitions, 240
diagnosis, 241
management, 241
pathophysiology, 240
physiology, 240 Gastrointestinal bleeding
definitions, 253
laboratory confirmation, 253
lower, 254
UGI bleeding, 253–254
Gastrointestinal tract, Henoch-Schönlein effects on, 383 Gaucher disease type I, 116
Gene sequencing, 101 Generalized seizures, 305, 305f Genetic disorders, 29
additional disorders, 108–109, 109t
amino acid metabolism defects, 110–114, 112t, 113t carbohydrate metabolism defects, 114–115 chromosomal (micro) deletion syndromes, 104–105 connective tissue disorders, 107
fatty acid oxidation disorders, 115 fetal evaluation, 102
genetic tests, 101
imprinting disorders, 105–106 inheritance patterns, 100–101, 101f lysosomal storage diseases, 115–116 mitochondrial disorders, 116
prenatal diagnosis, 102
sex chromosome syndromes, 104 skeletal disorders, 107
syndromes caused by teratogens, 109–110 terminology, 102
triplet repeat expansion disorders, 106
trisomy syndromes, 102–104, 103f, 103t Genital ulcers, 61, 61t
Genital warts, 61
Genomic imprinting, 105
Genu valgum, 416
Germ cell tumors, 359–360 classification, 359
ovarian tumors, 360
teratomas, 359–360
testicular tumors, 360 Germinomas Germ cell tumors Gianotti-Crosti syndrome, 447
Giardiasis, 174–175
Glanzmann thrombasthenia, 338 Glasgow coma scale, 302

Glasgow Coma Scale (GCS), 462, 462t Glial cell tumors, 353 Glomerulonephritis
classification, 276
clinical features, 276
definition, 275
etiology, 275
evaluation, 276
types, 276–278
Glucocorticoid excess, 136–137
Glucocorticoids, systemic lupus erythematosus, 390
Glucose-6-phospate dehydrogenase deficiency, 327–328, 328t Glycogen storage diseases (GSDs), 114
Gowers sign
juvenile dermatomyositis, 391–392 Granulocytes, disorders of
Chédiak-Higashi syndrome, 378
chronic granulomatous disease, 377 lymphocyte adhesion defects, 378 neutropenia, 378
Shwachman-Diamond syndrome, 378 Granulomatosis, with polyangiitis, 396 Gross motor development, 24–25, 24t Growing pains, 417
Growth
abnormalities of, 1–6, 2t, 3f, 4t, 5f, 6f defined, 2
failure to thrive, 2–3, 3f
head growth—related, 2t, 3–6, 4f head, abnormalities in, 2t, 3–6, 4f
health supervision related to, 1–6, 2t, 3f, 4t, 5f, 6f normal, 1–6, 2t, 3f, 4t, 5f, 6f
Growth abnormalities, neonatology, 79–80, 80t
Guillain-Barré syndrome, 312–313
clinical features, 312
defined, 312
diagnosis, 313
etiology, 312
management, 313
pathophysiology, 312
prognosis, 313
Gynecomastia, 65

H
Haemophilus influenzae type b (HIB), 217 vaccine for, 7
Hair, disorders, 450–452
Hair loss, 451

Hand-foot-mouth disease, 449
Happy spitters, 240
Head growth, abnormalities in, 2t, 3–6, 4f clinical features, 4
craniosynostosis, 4–6, 5f general concepts of, 3 macrocephaly, 4
microcephaly, 3
plagiocephaly, 6, 6f
Headaches, 309–311, 309f
causes, 309, 309f
clinical information, 309
cluster, 311
duration, 309
etiology, 309
location and radiation, 309 migraine, 310
quality of pain, 309 red flags, 309
tension, 310–311
time of onset, 309 HEADDSSS assessment, 50t Health supervision, 1–22
anticipatory guidance in, 16, 17t–18t circumcision, 13–14
dental care in, 14–16 developmental screening in, 16 growth-related, 1–6, 2t, 3f, 4t, 5f, 6f immunizations in, 6–11
well child care in, 1
well child screening in, 11–13 well child visit in, 1
Hearing impairment, 32–33
hearing screening for, 11 Hearing loss
medical evaluation, 33
prognostic factors, 32–33
types, 32
Hearing loss, medical evaluation for, 33 Hearing screening for, 11
Helminth infections, 176–177, 176t–177t
cysticercosis, 177
general concepts, 176
Hemangiomas, 338
Hemarthroses, in hemophilia A, 335 Hematology
anemia, 323–331
hemostasis disorders, 333–339, 334f, 334t, 335f

neutropenia, 339–340
pancytopenia, 331–332
polycythemias, 333
Hematuria, 269–270, 270f clinical significance, 269, 270f definition, 269
differential diagnosis, 271f epidemiology, 269
evaluation, 270
Hemochromatosis, 326
Hemoglobin (Hgb), 323
Hemoglobinopathies, sickle cell, 324f, 330–331, 330t, 331t Hemolytic anemia, 327
alloimmune, 324f, 329
autoimmune, 324f, 328–329
microangiopathic, 324f, 329 Hemolytic uremic syndrome (HUS), 329
atypical, 280
definition, 279
pneumococcal, 280
Shiga-like toxin–associated, 280
subtypes, 279
Hemophilia A, 334
clinical features, 335
inheritance, 335
laboratory findings, 335
management, 335
Hemorrhagic conjunctivitis, 431
Hemostasis, disorders of, 333–339, 334f, 334t, 335f acquired clotting factor, 336–337
blood vessels, 337 clotting cascade, 333, 334f concepts, 333, 334f
congenital clotting factor, 334 hypercoagulability, 339
platelet abnormalities, 337–339
Henoch-Schönlein purpura, 337
Henoch-Schönlein purpura (HSP), 383–384 clinical features, 383
defined, 383–384
diagnosis, 383–384
epidemiology, 383
management, 384
prognosis, 384
Henoch–Schönlein purpura (HSP) nephritis, 277 Hepatitis, 173
A, vaccine for, 9 assessment, 255

autoimmune hepatitis, 260–261 B, vaccine for, 7
cholestatic diseases of infancy, 256–258 general concepts, 255
infant jaundice, 255–256
viral hepatitis, 258–260, 259f, 260f Wilson disease, 258
Hepatitis A infection, 258–259, 259f Hepatitis A vaccine, 51
Hepatitis B infection, 259–260, 260f Hepatitis B vaccine (HBV), 7 Hepatitis C infection, 260 Hepatitis D infection, 260 Hepatitis E infection, 260 Hepatoblastoma, 359
Hepatocellular carcinoma, 359
Hepatocellular enzymes, 255
Hereditary elliptocytosis, 328 Hereditary fructose intolerance, 115
Hereditary hemorrhagic telangiectasia, 337
Hereditary renal diseases alport syndrome, 281
autosomal recessive polycystic kidney disease, 281 general concepts, 281
medullary sponge kidney, 281
multicystic renal dysplasia, 281 nephronophthisis, 282
Hereditary spherocytosis, 327
Herpes simplex virus infection, 448
Heterosexual gonadotropin-independent puberty, 129–130 Hip
developmental dysplasia of hip, 408–409 limp, 409–413, 410t, 411t
Hirschsprung disease, 244, 254 Hodgkin disease, 351–352, 351t clinical features, 351, 351t
diagnosis, 351
epidemiology, 351
management, 352
prognosis, 352
Reed-Sternberg cell in, 351 staging, 352
Homocystinuria, 111–113, 112t–113t
Human bites, 475
Human immunodeficiency virus (HIV), 168–170
Human leukocyte antigen (HLA), for ataxia telangiectasia, 377 Human papilloma virus (HPV) vaccines, 10, 51
Hunter syndrome (MPS type II), 115

Hurler syndrome (MPS type I), 115 HUS (hemolytic uremic syndrome)
atypical, 280
definition, 279
pneumococcal, 280
Shiga-like toxin–associated, 280
subtypes, 279
Hydroceles, 67
Hydrocephalus, 300–301
clinical features, 300
communicating, 300
defined, 300
etiology, 300
evaluation, 300
ex vacuo, 300 macrocephaly due to, 9 management, 300
and myelomeningocele, 301
noncommunicating, 300
prognosis, 300–301
types, 300
Hydrocephalus ex vacuo, 300 Hydroxychloroquine
systemic lupus erythematosus, 390 Hygiene, dental, 14–15
Hypercoagulability, 339
Hyperkeratosis, 442 Hypertension
classification, 272–273
clinical features, 274–275
definitions, 272
etiology, 274, 274t
evaluation, 275
management, 275
measurement, 273–274
Hyperthyroidism, 141–142 Hypertrophic pyloric stenosis, 241–242 Hyphema, 433
Hypocalcemia, 142–143
Hypogammaglobulinemia, 374
Hypoglycemia, 92, 145–146, 146 Hypopigmentation
oculocutaneous albinism, 450
pityriasis alba, 450
postinflammatory, 450
vitiligo, 450
Hypothyroidism, 140–141
Hypotonia, 296–299, 297f

classification, 296
clinical features, 296
defined, 296
differential diagnosis, 296–297, 297f disorders, 298–299 specific disorders evaluation, 297–298
Hypotonic disorders, peripheral congenital myotonic dystrophy, 299 infantile botulism, 298–299
spinal muscular atrophy, 298 Hypovolemic shock, 460
Hypsarrhythmia, 308

I
IBD (inflammatory bowel disease) clinical features, 251–252
definition, 251
epidemiology, 251
evaluation, 252
management, 252
IDMS (infants of diabetic mothers), 92 IgA deficiency, 374
IgA nephropathy, 277
Immune system, components of, 373
Immune thrombocytopenic purpura (ITP), 338 Immune-mediated thrombocytopenias, 338
Immunization(s), 6–11
active, 7
adolescent health screening, 50–51 adverse effects of, 10 contraindications to, 10–11
passive, 7
precautions with, 11
specific, 7–10
types of, 7
Immunocompromised hosts, VZIG in, 7 Immunodeficiency states
categories, 373, 373t
primary, 373, 373t
Immunology, 373, 373t Immunotherapy
for allergic rhinitis, 369
insect venom allergy, 371 Impetigo, 163
Imprinting disorders, 105–106 Inactivated influenza vaccine (IIV), 9–10 Inborn errors of metabolism (IEM), 110 Incisor(s), mandibular central, 15

Indirect inguinal hernia, 66–67 Infant jaundice, 255–256
Infantile botulism, 298–299
Infantile hemangiomas, 453
Infants of diabetic mothers (IDMS), 92 Infectious diseases
bone and joint infections, 166 Cat scratch disease, 178–179 cervical lymphadenitis, 162 child with fever, 152–154, 152t child with possible infection, 151 diarrhea, 166–168, 167t–168t
fever of unknown origin, 154–155, 155t fungal infections, 173–174
helminth infections, 176–177, 176t–177t meningitis, 156–159, 157t, 158t
middle and lower respiratory infections, 161
parasitic infections, 174–176
parotitis, 162–163
Rickettsial infections, 178
skin and soft tissue infections, 163–165 tuberculosis, 179–180
upper respiratory infections, 159–161 urinary tract infections, 168
viral infections, 168–173
Infectious disorders respiratory tract
bacterial tracheitis, 219
bronchiolitis, 220–221
epiglottitis, 217–218
laryngotracheobronchitis, 218–219, 218t
pertussis, 219–220
pneumonia, 221–222
Infectious enterocolitis, 254
Infectious mononucleosis, 170–171 Infective endocarditis, 201–202, 202t Inflammatory bowel disease (IBD)
clinical features, 251–252
definition, 251
epidemiology, 251
evaluation, 252
management, 252
Inheritance patterns, 100–101, 101f Innate responses, 373
Innocent cardiac murmurs, 188 Insect venom allergy, 371 Intellectual disability, 28–29, 29t Internal tibial torsion, 414, 414f

Intestinal anatomic obstructions, 241–244 atresias, 243
Hirschsprung disease, 244 hypertrophic pyloric stenosis, 241–242 intussusception, 244
malrotation and midgut volvulus, 242–243 Intestinal obstruction, 90–91, 91f
Intestinal perforation, juvenile dermatomyositis, 391 In-toeing, 413
Intracranial bleeding, 462–463 Intracranial pressure, 463, 463t
Intranasal cromolyn sodium, for allergic rhinitis, 369
Intranasal steroids, for allergic rhinitis, 369 Intraocular pressure, 434
Intrathecal methotrexate, 349 Intrauterine devices (IUDs), 56 Intravenous immune globulin (IVIG)
for Guillain-Barré syndrome, 313 Kawasaki disease, 386
for myasthenia gravis, 316
Intussusception, 244 Iron deficiency anemia
risk factors for, 12 screening for, 12
Iron toxicity, 472–473, 473t
Iron-deficiency anemia, 324–325
Isoimmune thrombocytopenia, 338

J
Jaundice, 85–88, 86f, 87f classification, 85–86
complications, 87–88
definition, 85
differential diagnosis, 86
evaluation, 86–87, 87f
management, 87
Juvenile ankylosing spondylitis, 398, 401
Juvenile dermatomyositis (JDM), 391–392, 399, 401
clinical features, 391
complications, 391
defined, 391
diagnosis, 391
epidemiology, 391
management, 391
prognosis, 391
Juvenile idiopathic arthritis (JIA), 386–388, 387t classification, 387
clinical features, 387–388, 387t

defined, 386
diagnosis, 386, 387t
epidemiology, 386
laboratory findings, 388
management, 388
oligoarticular, 387
extended, 387
persistent, 387
polyarticular, 388
prognosis, 388
systemic-onset, 388
Juvenile myasthenia, 316
Juvenile myelomonocytic leukemia (JMML), 350, 351
Juvenile polyps, 254

K
Karyotype analysis, 101
Kawasaki disease (KD), 201, 384–386, 384t, 385t atypical, 386
clinical features, 385, 385t
defined, 384
diagnostic criteria, 384–385, 384t epidemiology, 384
incomplete, 386
laboratory findings, 386
management, 386
prognosis, 386
time course of disease, 385 Keratoconjunctivitis, epidemic, 431
Ketogenic diet, 307
Ketotic hypoglycemia, 146, 150 Kidney
Henoch-Schönlein effects on, 383 systemic lupus erythematosus, 389
Klinefelter syndrome (XXY), 104 Klippel–Feil syndrome, 405
Knee angulation, 415
Knock-knees, 416
Koebner phenomenon, 444
Kostmann syndrome, 340
Kyphosis, 407

L
Lactate dehydrogenase (LDH), 255 Langerhans cell histiocytosis, 360–361
clinical features, 360–361
defined, 360
diagnosis, 361

etiology, 360
management, 361
prognosis, 361
Langerhans cell histiocytosis (LCH), 360–361 Language, 25
Language skills, 25–27, 26t
Lap belt injuries, 464 Large cell lymphoma, 352 Laryngeal tumors, 357
Laryngotracheobronchitis, 218–219, 218t
Late adolescence, 46
Latrodectus antivenin, 476
Lead, 473
Lead screening, 13
Learning disabilities, 29
Legg–Calvé–Perthes, 411–412
Leiden deficiency, 339
Leukemias, 348–351
acute lymphocytic leukemia, 348–350, 349t acute myelogenous leukemia, 350
chronic myelogenous leukemia, 350–351 Leukocoria
congenital cataract, 435
retinoblastoma, 435–436
LGA infants, 79–80 Limp, 409–413, 410t, 411t
Lipid screening, 12
Live attenuated influenza vaccine (LAIV), 9
Liver disease, acquired clotting factor disorders, 337 Liver tumors, 359
Long QT syndrome, 208 Louse infestation, 449 Lower extremity
growing pains, 417
Osgood–Schlatter disease, 416 patellofemoral pain syndrome, 416 sever disease, 416
torsional abnormalities, 413–416, 413t, 414f Lower gastrointestinal bleeding, 254
Lyme disease, 394–395
clinical features, 394
constitutional symptoms, 394
defined, 394
diagnosis, 394–395
epidemiology, 394
erythema migrans, 394
etiology, 394
management, 395

prognosis, 395
Lymphoblastic lymphoma, 352 Lymphocyte adhesion defects (LAD), 378 Lymphocytes, disorders of, 374–377, 374f
ataxia telangiectasia, 376
DiGeorge syndrome, 376
IgA deficiency, 374–375
severe combined immunodeficiency disease, 375 Wiskott-Aldrich syndrome, 376–377
X-linked (Bruton) agammaglobulinemia, 377 Lymphomas, 351–353, 351t
Hodgkin disease, 351–352, 351t
Non-Hodgkin lymphoma, 351t, 352–353 Lysosomal storage diseases, 115–116

M
Macrocephaly, 8
Macrocytic anemia, 323, 326–327
Macules, 441
Magical thinking, 27 Malabsorption
celiac disease, 238–239
general concepts, 237–238
protein intolerance leading to, 238 short bowel syndrome, 239
Malaria, 175
Male
puberty, 45, 128 reproductive health issues
gynecomastia, 65
painful scrotal masses, 65–66 painless scrotal masses, 66–67
Male genitalia and pubic hair development, for sexual maturity ratings, 47f Malnutrition, 236
and corticosteroids, 337
Malrotation and midgut volvulus, 242–243 Mammalian bites
cat, 475
dog, 475
epidemiology, 474
human, 475
Mandibular central incisors, 15 Mantoux skin test, 51
Marfan syndrome, 107
Marijuana, 54
MAS (meconium aspiration syndrome), 83–84 Maternal serum markers, 102
Maturity abnormalities, 78–79, 79t

Measles, 171–172, 447
mumps, and rubella (MMR) vaccine, 7 Measles, mumps, and rubella booster (MMR), 50 Meckel diverticulum, 254
Meconium, 83
Meconium aspiration syndrome (MAS), 83–84 Medulloblastoma, 353
Membranoproliferative glomerulonephritis (MPGN), 277–278 Membranous nephropathy (MN), 278
Mendelian inheritance, 100, 101f Meningitis, 156–159, 157t, 158t
aseptic, 158–159, 158t
bacterial, 156–158, 157t
definition, 156
Meningocele, 301
Meningococcal vaccine, 51 Meningococcal vaccine (MCV), 10 Menkes disease, 117
Menstrual cycle, 62–63, 62f Menstrual disorders, 62–65, 62f, 64t
abnormal vaginal bleeding, 63–65 amenorrhea, 63, 64t
dysmenorrhea, 63
normal menstrual cycle, 62–63, 62f Metabolic disorders, 29
Metatarsus adductus, 413
Methylation studies, 101
Microangiopathic hemolytic anemia, 324f, 329 Microarray, 101
Microcephaly, 3–4
acquired, 3, 4t causes of, 3, 4t clinical features of, 4 congenital, 3, 4t
defined, 3
Microcytic, hypochromic anemia, 323–326, 324f Micronutrients, 236
Middle adolescence, 46
Middle and lower respiratory infections, 161 Migraines, 310
with aura, 310
basilar artery, 310
classification, 310
clinical features, 310
defined, 310
diagnosis, 310
epidemiology, 310
etiology, 310

management, 310
ophthalmoplegic, 310
precipitating factors, 310
prognosis, 310
without aura, 310
Miliaria rubra (heat rash), 445 Miller Fisher syndrome, 312
Minimal residual disease (MRD), 349 Mitochondrial disorders, 116
Mitochondrial inheritance, 100
Mitral regurgitation, 392
Molluscum contagiosum, 449
Morbilliform, 447
Moro reflex, 25
Motility assessment, 427 Motor deficits, 27–28, 28t Motor development, 24–25, 24t
differential diagnosis, 25 fine motor skills, 25, 25t gross, 24–25, 24t
red flags in, 25
Movement disorders, 313–314
Mucopolysaccharidoses (MPSs), 115–116
Multifactorial inheritance, 100
Myasthenia gravis, 315–316
causes, 316
classification, 316
clinical features, 316
congenital, 316
defined, 315
diagnosis, 316
epidemiology, 316
etiology, 316
juvenile, 316
management, 316
neonatal, 316
prognosis, 316 Myelomeningocele
clinical features, 301–302
defined, 301
Myocarditis, 204, 392
rheumatic fever, 392
Myotonic dystrophy, 106

N
Naming period, of speech development, 26 Nasolacrimal duct (NLD) obstruction, 432 Nasopharyngeal tumors, 357

Natal teeth, defined, 15
Necrotizing enterocolitis (NEC), 91, 95, 98, 254
Necrotizing fasciitis, 164 Neonatal conjunctivitis
clinical features, 429, 429t, 430t defined, 429
differential diagnosis, 429
etiology, 429
Neonatal hepatitis, 256–257
Neonatal hypoglycemia, 145–146
Neonatal metabolic (state) screening, 11–12 Neonatal myasthenia, 316
Neonatal teeth, defined, 15 Neonatology, 73–99
apnea of prematurity, 84–85 cyanosis, 80–81
fetal exposure to drugs of abuse, 88
growth abnormalities, 79–80, 80t hypoglycemia, 92
infants of diabetic mothers, 92
jaundice, 85–88, 86f, 87f
maturity abnormalities, 78–79, 79t meconium aspiration syndrome, 83–84 newborn evaluation, 73–78, 74t, 75f
persistent pulmonary hypertension of newborn, 84 polycythemia, 92–93
respiratory distress syndrome, 81–83, 82f surgical conditions of newborn, 88–91, 89f, 91f
Nephroblastoma Wilms tumor
Nephrology
acute kidney injury, 283–284 chronic kidney disease
clinical features, 284
etiology, 284
evaluation, 284
management, 285
dehydration, 268–269
electrolytes, 268–269
fluids, 268–269 glomerulonephritis
classification, 276
clinical features, 276
definition, 275
etiology, 275
evaluation, 276
types, 276–278
hematuria, 269–270, 270f clinical significance, 269, 270f

definition, 269
differential diagnosis, 271f epidemiology, 269
evaluation, 270
hemolytic uremic syndrome atypical, 280
definition, 279
pneumococcal, 280
Shiga-like toxin–associated, 280
subtypes, 279 hereditary renal diseases
alport syndrome, 281
autosomal recessive polycystic kidney disease, 281 general concepts, 281
medullary sponge kidney, 281 multicystic renal dysplasia, 281 nephronophthisis, 282
hypertension classification, 272–273
clinical features, 274–275
definitions, 272
etiology, 274, 274t
evaluation, 275
management, 275
measurement, 273–274 nephrotic syndrome
classification, 278
clinical features, 278–279
definition, 278
diagnosis, 279
epidemiology, 278
management, 279
pathophysiology, 278
prognosis, 279
proteinuria, 270–272, 272f
classification, 272
detection, 271
epidemiology, 271 evaluation, 273f
renal tubular acidosis clinical features, 282, 282t definition, 282
etiology, 282, 282t
evaluation, 283
types, 282
structural and urologic abnormalities acquired obstruction, 285
congenital obstructive abnormalities, 285

renal abnormalities, 285–286 vesicoureteral reflux, 286, 287f
urinary tract infection, 288–289 urolithiasis
clinical features, 288
diagnosis, 288
epidemiology, 287
etiology, 287–288
evaluation, 288
management, 288 Nephrotic syndrome
classification, 278
clinical features, 278–279
definition, 278
diagnosis, 279
epidemiology, 278
management, 279
pathophysiology, 278
prognosis, 279 Neural tube defect, 301
Neuroblastoma, 354–356
causes, 354
clinical features, 354, 355t diagnosis, 354
epidemiology, 354
etiology, 354
incidence, 354
management, 356
prognosis, 356
staging, 354
Neurocutaneous syndromes, 451t Neurofibromatosis, macrocephaly due to, 4 Neurofibromatosis type 1, 108–109
Neurogenic shock, 460 Neurology
Becker muscular dystrophy, 314–315
comatose patient, 302–303, 303t Duchenne muscular dystrophy, 314–315 headaches, 309–311, 309f
hydrocephalus, 300–301
hypotonia, 296–299, 297f
movement disorders, 313–314
myasthenia gravis, 315–316
seizure disorders, 304–308, 305f, 305t, 306t spina bifida, 301–302
unsteady gait, 311–313, 311t
Neutropenia, 339–340
autoimmune, 340

causes, 339–340
cyclic, 340
definition, 339
genetic syndromes and, 340 isoimmune, 340
mild, 339
moderate, 339
risk of infection, 339 severe, 339
Nevocellular nevi, 450
acquired, 450
congenital, 450
Nevus simplex/flammeus, 453 Newborn evaluation, 73–78, 74t, 75f
abdominal examination, 76
chest examination, 76
craniofacial examination, 74–75, 75f extremity examination, 77
general appearance and initial assessment, 73, 74t genitalia examination, 77
neck and clavicle examination, 75–76 neurologic examination, 74 pregnancy and maternal history, 73 skin examination, 77–78
spine examination, 77
Niemann–Pick disease types A and B, 116 Night terrors, 36
Nightmares, 35
Nikolsky sign, 445
Nitroblue tetrazolium (NBT) test, 377 Nodules, 441
Noncardiac chest pain, 209
Non-Hodgkin lymphoma, 351t, 352–353 classification, 352
clinical features, 351t, 352 diagnosis, 352
epidemiology, 352
management, 353
prognosis, 353
staging, 352
Noninfectious disorders, respiratory tract
apnea, apparent life-threatening event, 229–230 asthma, 222–225, 223f, 225f
bronchopulmonary dysplasia, 227–228
cystic fibrosis, 225–227, 226t, 227f foreign body aspiration, 228–229
Noninvasive prenatal testing (NIPT), 102 Nonketotic hypoglycemia, 145

Nonstimulant medications, ADHD, 32 Noonan syndrome, 108
Normal puberty, 127–128
Normocytic, normochromic, 323, 324f, 327–331, 328t, 330t, 331t Normocytic, normochromic anemia, 323, 324f, 327–331, 328t, 330t, 331t Nursemaid’s elbow, 403–404
Nutrition, 236, 237t

O
Obesity, 54
Object permanence, 27 Ocular trauma
corneal abrasion, 433
hyphema, 433
orbital floor fracture, 433 retinal hemorrhages, 432–433
Oculocephalic maneuver, in comatose patients, 304 Oculocutaneous albinism, 450
Ointments, 442
Omphalocele, 94, 97 Oncology
bone tumors, 357–359, 358t brain tumors, 353–354 germ cell tumors, 359–360
Langerhans cell histiocytosis, 360–361 leukemias, 348–351
liver tumors, 359
lymphomas, 351–353, 351t
renal tumors, 354–357
retinoblastoma, 359 soft tissue tumors, 357
suprarenal tumors, 354–357 Onychomycosis Tinea unguium Ophthalmology
abnormal tearing, 432
amblyopia, 428
congenital glaucoma, 434 conjunctivitis, 429–432, 429t, 430t leukocoria, 435–436
ocular examination, 427
ocular trauma, 432–433 red eye, 429–432, 430f
retinopathy of prematurity, 434–435 strabismus, 436
vision screening, 427
visual development, normal, 427–428 Optic glioma, 354
Oral and inactivated polio virus (OPV/IPV), vaccines for, 7–8

Oral contraceptives, 57
Oral rehydration therapy (ORT), 269 Orbital floor fracture, 433
Orbital tumors, 357
Organic acidemias, 113–114 Orthopedics
fractures, 417–420, 418f, 419f, 419t hip, 408–413, 411t
lower extremity, 413–417, 413t, 414f
spine, 404–408, 406f, 407t upper extremity, 403–404
Osgood–Schlatter disease, 416
Osteogenesis imperfecta (OI), 107 Osteogenic sarcoma, 357–358
causes, 358
clinical features, 358, 358t diagnosis, 358
epidemiology, 357
etiology, 358
management, 358
prognosis, 358
Osteomyelitis, 412–413
Osteopenia, juvenile dermatomyositis, 391 Otitis externa, 161
Otoacoustic emission (OAE), in hearing screening, 11
Ovarian tumors, 360
Overgrowth syndrome, macrocephaly due to, 4

P
Painful scrotal masses, 65–66 Painless scrotal masses, 66–67 Pallid breath-holding spells, 37 Pancytopenia, 331–332
acquired, 332
congenital, 332
defined, 331
Papular acrodermatitis Gianotti-Crosti syndrome
Papules, 441
Papulovesicular, 447
Parachute reaction, 25
Paraphimosis, 14
Parasitic infections, 174–176
amebiasis, 174
giardiasis, 174–175
malaria, 175
toxoplasmosis, 175–176
Parenteral rehydration, 269
Parotitis, 162–163

Passive autoimmune thrombocytopenia, 338 Patau syndrome, 104
Patellar chondromalacia, 416 Patellofemoral pain syndrome, 416
Patent ductus arteriosus (PDA), 193–194, 194f Pavlik harness, 409
PCV13, 9
PDA (patent ductus arteriosus), 193–194, 194f Penile abnormalities, anatomic, 14 Percutaneous umbilical blood sampling, 102 Perianal cellulitis, 164
Pericarditis, 203–204, 203f
Peripheral hypotonia, 296
Persistent neonatal hypoglycemia, 145 Persistent oligoarthritis, 387
Persistent pulmonary hypertension of newborn (PPHN), 84, 94, 97
Pertussis, 219–220
Pharmacologic anesthetics, during circumcision, 14 Pharyngitis, 159–160
Pharyngoconjunctival fever, 431
Phenylketonuria (PKU), 118, 121
Phimosis, 14
Phototherapy, 87 Physical abuse
clinical features, 467
diagnosis, 467
epidemiology, 466–467
management, 467
Picky eating, 36
Pierre Robin sequence, 109 Pigeon-toe, 413
Pit viper snakes, 476 Pityriasis alba, 450
Pityriasis rosea, 444
Plagiocephaly, 6, 6f Plasmapheresis
for Guillain-Barré syndrome, 313 for myasthenia gravis, 316
Plumbism, 13
Pneumococcal hemolytic uremic syndrome, 280 Pneumococcal vaccine, 9
Pneumonia, 221–222 Poisonings
acetaminophen toxicity, 471–472, 471t
antidotes, 471t
carbon monoxide, 474
caustic agents, 473–474
concepts, 468–469

evaluation, 469
iron toxicity, 472–473, 473t lead, 473
management, 469
physical examination findings, 470f salicylates, 472
Polyarteritis nodosa, 396
Polycythemia, 92–93 Polycythemias
appropriate, 333
complications, 333
defined, 333
inappropriate, 333
primary, 333
relative, 333
secondary, 333
Port wine stain, 95, 98 Port wine stains, 453
Positive Barlow maneuver, 408 Positive Ortolani maneuver, 409
Postinflammatory hypopigmentation, 450
Post-Streptococcal Glomerulonephritis (PSGN), 276–277 Postterm delivery, 79
Postural reactions, 24t, 25 Postural roundback, 407
Potter syndrome, 109
PPHN (persistent pulmonary hypertension of newborn), 84 PPSV23, 9
Prader–Willi syndrome, 105
Precocious puberty, 128–130
Pregnancy, 55–56
newborn evaluation, 73
Premature adrenarche, 128–129
Premature thelarche, 128
Prespeech period, of speech development, 26 Preterm delivery, 78–79, 79t
Primary irritant contact dermatitis, 443 Primary ocular herpes simplex virus, 431 Priming, 368
Primitive neuroectodermal tumors (PNETs), 353, 354 Primitive reflexes, 24t, 25
Problem drinking, 54 Protein C deficiency, 339 Protein S deficiency, 339 Proteinuria, 270–272, 272f
classification, 272
detection, 271
epidemiology, 271

evaluation, 273f Pseudostrabismus, 436
Psoriasis, 444
Psoriatic arthritis, 395
Psychosocial development, 45–46 Puberty
females, 45
males, 45
Puberty disorders, 127–131
Pulmonary disease, 216–217
Pulmonary stenosis, 195 Pulmonology
pulmonary disease, 216–217
respiratory system, 216
infectious disorders, 217–222
noninfectious disorders, 222–230
Pustules, 441
Pyogenic granulomas, 453

Q
22q11.2 deletion syndrome, 104–105

R
Radiation therapy, for brain tumors, 354 Rapid eye movement (REM) sleep, 35 Rashes
periorbital violaceous heliotrope, 391 transient salmon-pink macular rash, 388
RAST tests, in food allergy, 371
Raynaud phenomenon, systemic lupus erythematosus, 389 Reactive arthritis, 395, 399, 401
Recessive inheritance, 100
Red blood cells (RBC) aplasias, 327, 331, 332t
extrinsic defects, 324f, 328–329 intrinsic defects, 324f, 327
Red eye
causes, 430
differential diagnosis, 430f etiology, 429
evaluation, 430
management, 431
Red reflex assessment, 427, 427t
Reed-Sternberg cell, in Hodgkin disease, 351 Reed-Sternberg cells, 362, 365
Renal tubular acidosis (RTA) clinical features, 282, 282t definition, 282

etiology, 282, 282t
evaluation, 283
types, 282
Renal tumors, 354–357, 355t Reproductive health issues
in female
contraception, 56–57
pregnancy, 55–56 in males
gynecomastia, 65
painful scrotal masses, 65–66 painless scrotal masses, 66–67
Respiratory distress syndrome (RDS), 81–83, 82f, 95, 98
clinical features, 82
complications, 83
definition, 81
differential diagnosis, 82f epidemiology, 82
evaluation, 82
management, 83
pathophysiology, 81–82
prognosis, 83
Respiratory system, 216 Respiratory tract
infectious disorders
bacterial tracheitis, 219
bronchiolitis, 220–221
epiglottitis, 217–218
laryngotracheobronchitis, 218–219, 218t
pertussis, 219–220
pneumonia, 221–222 noninfectious disorders
apnea, apparent life-threatening event, 229–230
asthma, 222–225, 223f, 225f bronchopulmonary dysplasia, 227–228 cystic fibrosis, 225–227, 226t, 227f foreign body aspiration, 228–229
Retinal hemorrhages, 432–433
Retinoblastoma, 359, 435–436
clinical features, 435
defined, 435
diagnosis, 435
epidemiology, 435
etiology, 435
management, 435–436
prognosis, 436
Retinopathy of prematurity (ROP) complications, 434

defined, 434
etiology, 434
management, 434–435 screening and prevention, 435
Rhabdomyosarcoma, 364, 366
causes, 357
clinical features, 357
diagnosis, 357
epidemiology, 357
etiology, 357
management, 357
prognosis, 357
sites, 357
Rheumatic fever, 392–393, 393t, 399, 401
clinical features, 392
defined, 392
diagnosis, 393, 393t
epidemiology, 392
etiology, 392
Jones criteria for, 393, 393t laboratory findings, 393
management, 393
prognosis, 393 Rheumatologic markers
juvenile idiopathic arthritis, 388
systemic lupus erythematosus effects on, 389 Rheumatology
Henoch-Schönlein purpura, 383–384
juvenile dermatomyositis, 391–392 juvenile idiopathic arthritis, 386–388, 387t Kawasaki disease, 384–386, 384t, 385t Lyme disease, 394–395
rheumatic fever, 392–393, 393t
systemic lupus erythematosus, 389–390, 389t Rickets, 143–144
Rickettsial infections, 178
Roseola infantum, 447
RTA (renal tubular acidosis) clinical features, 282, 282t definition, 282
etiology, 282, 282t
evaluation, 283
types, 282
Rubella, 172–173, 447
Russell–Silver syndrome, 106

S
St. Vitus dance Sydenham chorea

Salicylates, 472
Salter–Harris Classification, 419f, 419t Scabies, 450
Scalp edema, 3
Scarlet fever, 164–165
Scheuermann kyphosis, 407
Scleroderma, 396 Scoliosis, 405, 406f, 407t Scurvy, 337
Seborrheic dermatitis, 444
Sebum, 452 Seizures
absence, 305, 305f
afebrile, 305, 305f
causes, 304, 305t
classification, 305, 305f
defined, 304
diagnosis, 306
epidemiology, 304
etiology, 304, 305t
evaluation, 306
febrile, 305, 307, 318, 321
focal, 305, 305f
generalized, 305, 305f
management, 306–307
prognosis, 307
tonic-clonic, 305, 305f
Seizure disorders, 304–308, 305f, 305t, 306t, 318, 321
causes, 304, 305t
defined, 304
diagnosis, 306
differential diagnosis, 306, 306t epidemiology, 304
etiology, 304
evaluation, 306
management, 306–307
prognosis, 307
Selective eating, 36
Selective mutism, 36
Sensorimotor period, 26
Sensory impairments, 32–33
hearing impairment, 32–33
visual impairment, 33 Separation anxiety disorder, 36 Septic shock, 460
Serologic testing, Lyme disease, 395 Seronegative spondyloarthropathies, 395 Serum sicknesslike reaction, 445

Sever disease, 416
Severe combined immunodeficiency disease (SCID) autosomal recessive, 375
clinical features, 375
defined, 375
diagnosis, 375
etiology, 375
management, 375
X-linked, 375
Severe congenital agranulocytosis, 340 Sex chromosome syndromes, 104 Sexual abuse
concepts, 467
diagnosis, 467–468
epidemiology, 467
history of, 467
laboratory studies, 468
management, 468
physical examination, 468
Sexual maturity, 1 Sexual maturity ratings
for female breast development, 48f
for female pubic hair development, 49f
for male genitalia and pubic hair development, 47f Sexually transmitted infections (STIs)
cervicitis, 59–60
clinical features, 58–61
epidemiology, 58 genital ulcers, 61, 61t genital warts, 61
PID, 60
risk factors, 58
urethritis, 60–61
vaginitis, 58–59
Shiga-like toxin–associated hemolytic uremic syndrome, 280 Shock, 460–461
cardiogenic, 460
classification, 460
compensated, 460
concepts, 460
decompensated, 460
diagnosis, 460–461
distributive, 460
hypovolemic, 460
management, 461
septic, 460
Short bowel syndrome, 239 Short stature

categorization, 125–126, 126f
definition, 124
endocrinopathies, 127
evaluation, 126–127, 127t
general concepts, 124
history, 124–125, 125f
physical examination, 125
Shwachman-Diamond syndrome, 340, 378, 380, 382
Sibling rivalry, 37
Sickle cell (SS) hemoglobinopathies, 324f, 330–331, 330t, 331t Sideroblastic anemia, 326
Simple URIs, 159
Sinusitis, 159
Sjögren syndrome, 396
Skeletal disorders, 107 Skin
anti-infective agents, 442
anti-inflammatory agents, 442–443
creams, 442
destructive therapies, 442
examination, 441
Henoch-Schönlein effects on, 383 hydration, 442
lotions, 442
Lyme disease, 394
ointments, 442
rheumatic fever, 392
solutions, 442
systemic lupus erythematosus effects on, 389 thickened, 442
topical anti-inflammatory agents, 443 Skin and soft tissue infections, 163–165 Skin infections
bacterial, 447
ectoparasites, 449–450
fungal, 446
viral, 447–449
Skin tests, in food allergy, 371 Sleep problems, 35–36
abnormal patterns, 35
epidemiology, 35
night terrors, 36
nightmares, 35
normal patterns, 35
Slipped capital femoral epiphysis (SCFE), 412 Small-for-gestational-age (SGA) infants, 79, 80t Social phobia, 36
Social skills, 27

Soft tissue tumors, 357 Rhabdomyosarcoma Sotos syndrome, macrocephaly due, 4 Spectrin, 328
Speech, 25
Sphingolipidoses, 116 Spina bifida (SB)
causes, 301
clinical features, 301–302
defined, 301
diagnosis, 302
epidemiology, 301
etiology, 301
management, 302
occulta, 301
prognosis, 302
types, 301
Spinal muscular atrophy (SMA), 298, 317, 320 Spine
back pain, 407–408, 407t
cervical spine disorders, 404–405 kyphosis, 407
scoliosis, 405, 406f, 407t
Splenectomy, for ataxia telangiectasia, 376 Spondylolisthesis, 408
Spondylolysis, 408
Standardized growth curves, 2
Staphylococcal Scalded Skin Syndrome (SSSS), 164 Status epilepticus, 304
Stereopsis, 428 Steroids
Henoch-Schönlein purpura, 384
Kawasaki disease, 386
Stevens-Johnson syndrome (SJS), 445, 445t Stimulant medications, ADHD, 31–32, 32t STIs Sexually transmitted infections (STIs) Stork bites, 453
Strabismus
clinical features, 436
defined, 436
etiology, 436
management, 436
vertical, 436
Streptococcus mutans, nursing caries due, 20, 22
Streptococcus pneumoniae, 409
Streptococcus pyogenes, 409
Structural and urologic abnormalities acquired obstruction, 285
congenital obstructive abnormalities, 285

renal abnormalities, 285–286 vesicoureteral reflux, 286, 287f
Subdural hematoma, 463
Substance abuse, 52–54
alcohol, 53
diagnosis, 52
epidemiology, 52
etiology, 52
marijuana, 54
risk factors, 53
tobacco, 53–54
Sudden infant death syndrome (SIDS), 468 Suicide, epidemiology, 51–52
Supracondylar fractures, 419–420
Supraglottitis Epiglottitis
Suprarenal tumors, 354–357, 355t
Surgical conditions of newborn, 88–91, 89f, 91f abdominal wall defects, 90
congenital diaphragmatic hernia, 89–90
esophageal atresia with tracheoesophageal fistula, 88–89, 89f intestinal obstruction, 90–91, 91f
necrotizing enterocolitis, 91
Survival motor neuron gene (SMN1), 298 SVT, 206–207, 206t, 207f
Sydenham chorea, 313–314, 392
clinical features, 313
defined, 313
diagnosis, 313
differential diagnosis, 313
epidemiology, 313
management, 314
pathophysiology, 313
prognosis, 314
rheumatic fever, 392, 393
Syndromes caused by teratogens, 109–110
Systemic lupus erythematosus (SLE), 389–390, 389t, 399, 401
causes, 389
clinical features, 389
defined, 389
diagnostic criteria, 389, 389t epidemiology, 389
etiology, 389
laboratory findings, 390
management, 390
prognosis, 390
Systemic scleroderma, 396
Systemic-onset juvenile idiopathic arthritis, 399, 401

T
Takayasu arteritis, 395
Talipes equinovarus (clubfoot), 414 Tantrums, temper, 37
TAPVR, 200–201, 201f
Tay–Sachs disease, 116, 118, 122
Tdap (Tetanus, diphtheria, and acellular pertussis booster), 50 Telogen effluvium, 452
Temper tantrums, 37
Tension headaches, 310–311
Teratomas, 359–360, 364, 366
Testicular neoplasms, 66
Testicular tumor, 363, 366
Testicular tumors, 360
Tetanus, diphtheria, and acellular pertussis booster (Tdap), 50 Tetralogy of fallot, 197–198, 197f, 197t
α-Thalassemia, 325–326
β-Thalassemia, 325–326
β-Thalassemia minor, 326
Thrombocytopenia, 337
immune-mediated, 338
neonatal, 338
isoimmune, 338
passive autoimmune, 338
Thrombocytopenia-absent radius (TAR) syndrome, 337 Thrombosis, disease states associated with, 339 Thymectomy, for myasthenia gravis, 316
Thyroid disorders, 140–142
Tinea capitis, 446, 452
Tinea corporis, 446
Tinea cruris, 446
Tinea faciei, 446
Tinea pedis, 446
Tinea unguium, 446
Tinea versicolor, 446
Tobacco, 54–55
Toddler fractures, 420
Toilet training, 38
Tonic-clonic seizures, 305, 305f Tooth (teeth)
natal, defined, 18
neonatal, defined, 18 Tooth brushing, age for, 14 Tooth eruption, 14
Topical antihistamines, 431, 437, 439 Torsional abnormalities, 413–416, 413t, 414f Torticollis, 404
Total body fluid requirement, 268

Tourette syndrome, 314
clinical features, 314
defined, 314
diagnosis, 314
differential diagnosis, 314
epidemiology, 314
etiology, 314
management, 314
prognosis, 314
tics in, 314
Toxic epidermal necrolysis, 445
Toxic shock syndrome (TSS), 165, 165t Toxic synovitis, 410
Toxoplasmosis, 175–176
Traction alopecia, 452
Transient neonatal hypoglycemia, 145 Transient salmon-pink macular rash, 388 Transient synovitis, 410
Trauma, 461–464
abdominal, 464
chest, 464
concepts, 461
dental, 16
Glasgow Coma Scale, 462, 462t head, 462–463
injuries, 462–464
primary survey, 462, 462t adjuncts, 462
secondary survey, 462 spinal cord injury, 464
Trichotillomania, 452 Tricuspid atresia, 198–199, 199f Tricyclic antidepressants, 35
Triggers, 36
Trigonocephaly, 5
Triple/quadruple markers, 102
Triplet repeat expansion disorders, 106 Trisomy 13, 104
Trisomy 18, 104
Trisomy 21, 102, 103f, 103t Trisomy syndromes
trisomy 13, 104
trisomy 18, 104
trisomy 21, 102, 103f, 103t Truncus arteriosus, 199–200, 199f Tuberculosis (TB), 179–180
screening for, 12–13
Tumor lysis syndrome, 350, 362, 365

Tumor necrosis factor (TNF) inhibitors Kawasaki disease, 386
Turner syndrome, 118, 121 Turner syndrome (XO), 104
Type 1 diabetes mellitus, 137–138
Type 2 diabetes mellitus, 138–139

U
UGI bleeding, 253–254 Ulcerative colitis, 251, 251t Ultrasound, 102
Unilateral conjunctivitis, 430 Uniparental disomy (UPD), 105 Universal newborn hearing screening, 11 Unsteady gait, 311–313, 311t
vs. cerebellar dysfunction, 311 vs. Guillain-Barré syndrome, 311 defined, 311
differential diagnosis, 311–312, 311t Upper extremity
anterior shoulder dislocation, 404 brachial plexus injury, 403 nursemaid’s elbow, 403–404
Upper respiratory infections (URIs) acute otitis media, 160–161 general concepts, 159
otitis externa, 161
pharyngitis, 159–160
simple, 159
sinusitis, 159
Urea cycle disorders (UCDs), 112t–113t, 113 Urethritis, 60–61
Urinalysis, 12
Urinary tract infection (UTI), 288–289 Urinary tract infections (UTIs)
in uncircumcised male infants, 13
URIs (upper respiratory infections) acute otitis media, 160–161 general concepts, 159
otitis externa, 161
pharyngitis, 159–160
simple, 159
sinusitis, 159 Urolithiasis
clinical features, 288
diagnosis, 288
epidemiology, 287
etiology, 287–288

evaluation, 288
management, 288
Urology, 268–295
Urticaria, 445
Urticaria (hives), 371, 372t acute, 371
causes, 371, 372t
chronic, 371
classification, 371
defined, 371
management, 371
UTI (urinary tract infection), 288–289

V
Vaccine(s) DTaP, 7
HBV, 7
Hep A, 9
HIB virus, 8
human papilloma virus, 10 influenza, 9–10
live, 8
meningococcal, 10
MMR, 8
non-live, 8
OPV/IPV, 7–8
pneumococcal, 9
rotavirus, 8
varicella, 8
VACTERL (VATER) association, 108 Vagal nerve stimulator, 307
Vaginal ring, 57
Vaginitis, 58–59
Valvulitis, 392
rheumatic fever, 392
Varicella, 173
Varicella (chickenpox), 447, 448t Varicella vaccine, 51
Varicella (chickenpox) virus
vaccine for, 8
Varicella zoster immune globulin (VZIG), for immunocompromised patients, 7 Varicoceles, 67
Vascular skin lesions, 453 Velocardiofacial syndrome, 104–105
Ventricular septal defect (VSD), 191, 191f, 194 Vertical strabismus, 436
Vesicles, 441
Vigabatrin

for infantile spasms, 308 Viral conjunctivitis, 431
Viral exanthem, 447
Viral hepatitis, 258–260, 259f, 260f Viral infections
hepatitis, 173
human immunodeficiency virus, 168–170 infectious mononucleosis, 170–171
measles, 171–172
rubella, 172–173
varicella, 173
Viral pneumonia, 222
Vision screening, 11, 427
Visual acuity, development, 427–428 Visual impairment, 33
Vitamin D, for juvenile dermatomyositis, 391
Vitamin K deficiency, acquired clotting factor disorders, 336 Vitiligo, 450
von Willebrand disease, 334, 336, 342, 345 von Willebrand factor (vWf), 336
VSD (ventricular septal defect), 191, 191f, 194
VZIG (varicella zoster immune globulin), for immunocompromised patients, 7

W
Warts, 449
Well child care, general concepts of, 1 Well child screening, 11–13
cholesterol screening, 12
hearing screening, 11
iron-deficiency anemia screening, 12 lead screening, 13
lipid screening, 12
neonatal metabolic (state) screening, 11–12 tuberculosis screening, 12–13
urinalysis screening, 12
vision screening, 11
Werdnig-Hoffman disease, 298, 317, 320
West syndrome, 308
Wheals, 441
Williams syndrome, 105
Wilms tumor, 356–357
clinical features, 356
diagnosis, 356
epidemiology, 356
management, 356
prognosis, 356
staging, 356
Wilson disease, 116–117, 118, 121, 258

Wiskott-Aldrich syndrome, 337, 376–377, 380, 382 Word combination period, of speech development, 26

X
X-linked (Bruton) agammaglobulinemia, 377 X-linked dominant mode of inheritance, 100 X-linked–recessive mode of inheritance, 100

Y
Yoslk sac tumor, 360, 364, 366